BASES ET RVISIONS NCESSAIRES POUR LE TOEIC · 2019. 12. 2. · BASES ET RVISIONS NCESSAIRES POUR LE...

185

Transcript of BASES ET RVISIONS NCESSAIRES POUR LE TOEIC · 2019. 12. 2. · BASES ET RVISIONS NCESSAIRES POUR LE...

Page 1: BASES ET RVISIONS NCESSAIRES POUR LE TOEIC · 2019. 12. 2. · BASES ET RVISIONS NCESSAIRES POUR LE TOEIC 3 CONSIGNES Ā APPLIQUER AVANT LE STAGE Révisez les temps présentés dans
Page 2: BASES ET RVISIONS NCESSAIRES POUR LE TOEIC · 2019. 12. 2. · BASES ET RVISIONS NCESSAIRES POUR LE TOEIC 3 CONSIGNES Ā APPLIQUER AVANT LE STAGE Révisez les temps présentés dans
Page 3: BASES ET RVISIONS NCESSAIRES POUR LE TOEIC · 2019. 12. 2. · BASES ET RVISIONS NCESSAIRES POUR LE TOEIC 3 CONSIGNES Ā APPLIQUER AVANT LE STAGE Révisez les temps présentés dans

3BASES ET RÉVISIONS NÉCESSAIRES POUR LE TOEIC

CONSIGNES Ā APPLIQUER AVANT LE STAGE

Révisez les temps présentés dans ce fascicule. Seuls les temps simples y sont abordés. Cela est volontaire : nous verrons ensemble les temps progressifs.

Si vous avez des difficultés avec les temps simples, complétez ces excercices par des exercices sur une plateforme d’entrainement en ligne comme anglaisfacile.com.

De nombreuses vidéos sont disponibles sur Youtube pour le «Listening». Ecoutez-en une ou deux parties en vous concentrant sur les réponses. Vous pouvez aussi écouter les dialogues en regardant les réponses pour entrainer votre oreille à reconnaître la bonne réponse. Il est essentiel de le faire régulièrement.

Dès le premier jour, vous serez interrogés sur les verbes irréguliers. Il est impératif de tous les connaître.

Cela est une des conditions contractuelles pour le remboursement de votre TOEIC si vous n’améliorez pas votre score.

Le second jour, vous serez interrogés sur les mots de liaison. Un maximum de deux fautes sera toléré.

Enrichissez votre vocabulaire grâce à des articles de presse comme «The Economist» ou «The Times», vous en trouverez beaucoup sur Internet.

Ci-joint à la fin de ce fascicule, un extrait de «The Economist».

Forcez-vous à lire une dizaine de lignes de cet article en cherchant tous les mots de vocabulaire qui vous font défaut.

Listez-les dans un répertoire et apprenez-les. Le lendemain, refaites la même chose. Une fois l’article lu en entier, entraînez-vous à le résumer afin de vérifier que vous avez bien tout compris.

Durant le stage, nous reviendrons ensemble sur le vocabulaire de cet article de presse.

Tout au long de votre stage, vous serez testés sur les éléments étudiés en cours afin de vérifier votre apprentissage.

Good Luck !

Page 4: BASES ET RVISIONS NCESSAIRES POUR LE TOEIC · 2019. 12. 2. · BASES ET RVISIONS NCESSAIRES POUR LE TOEIC 3 CONSIGNES Ā APPLIQUER AVANT LE STAGE Révisez les temps présentés dans
Page 5: BASES ET RVISIONS NCESSAIRES POUR LE TOEIC · 2019. 12. 2. · BASES ET RVISIONS NCESSAIRES POUR LE TOEIC 3 CONSIGNES Ā APPLIQUER AVANT LE STAGE Révisez les temps présentés dans

PREPARATION DES COURS

PARTIE ILa grammaire

Page 6: BASES ET RVISIONS NCESSAIRES POUR LE TOEIC · 2019. 12. 2. · BASES ET RVISIONS NCESSAIRES POUR LE TOEIC 3 CONSIGNES Ā APPLIQUER AVANT LE STAGE Révisez les temps présentés dans

6

A. Les différents temps

1. Présent simpleHabitude ou vérité généraleAffirmationSujet + Verbe ex: I eatShe/he/it + Verbes +s ex: she eats

NégationSujet + DON’T + verbe ex: you don’t eatShe/He /It + DOESN’T + Verbe ex : she doesn’t eat

InterrogationDO + Sujet + Verbe ex: Do you eat?DOES + She/He/It + verbe ex: Does she eat?

2. FuturAffirmationSujet + WILL + verbe ex: She will come NégationSujet + WON’T + verbe ex : She won’t come

Interrogation WILL + Sujet + Verbe ex: Will you play ?

3. PrétéritAction finie qui a eu lieu dans le passé

Pas de lien avec le présentAffirmationSujet + Verbe +ED ou verbes irréguliers (2ème colonne)) ex : She played / I ate NégationSujet + DIDN’T + verbe ex : She didn’t play /I didn’t eat

InterrogationDID + Sujet + Verbe ex: Did you play?/ Did she eat?

Page 7: BASES ET RVISIONS NCESSAIRES POUR LE TOEIC · 2019. 12. 2. · BASES ET RVISIONS NCESSAIRES POUR LE TOEIC 3 CONSIGNES Ā APPLIQUER AVANT LE STAGE Révisez les temps présentés dans

7BASES ET RÉVISIONS NÉCESSAIRES POUR LE TOEIC

4. Present PerfectOn se sert du present perfect pour une action qui a commencé dans le passé et qui continue dans le présent.Peter studied English for 10 years: l’action est terminéePeter has studied English for 10 years: Peter a commencé à étudier il y a 10 ans mais il continue de le faire

AffirmationSujet + Have/has + Participe Passé (verbe-ed) ou Verbe Irrégulier (3ème colonne) ex: She has played/ I have studied

NégationSujet + Have/Has+ NOT + Participe passé ex : We have not liked/She hasn’t come

InterrogationHave/has + Sujet + Participe Passé ex: Has you played ? / Have you looked ?

5. Past PerfectCe temps est utilisé pour une action passée qui est antérieure à une autre action. Exemple: I had eaten when you arrived: J’avais mangé avant quand tu es arrivé.

AffirmationSujet + HAD + Participe Passé ex: I had eaten

NégationSujet + HAD NOT + Participe Passé ex: She had not played

InterrogationHAD + Sujet + Participe Passé ex: Had you understood?

6. Les ModauxAffirmationSujet + Modal + Base Verbale ex: I can eat

NégationSujet + Modal + NOT + Base Verbale ex: She must not come

InterrogationModal + Sujet + Base Verbale ex: Should I call her?

Page 8: BASES ET RVISIONS NCESSAIRES POUR LE TOEIC · 2019. 12. 2. · BASES ET RVISIONS NCESSAIRES POUR LE TOEIC 3 CONSIGNES Ā APPLIQUER AVANT LE STAGE Révisez les temps présentés dans

8

Les différents modauxCan = capacité ou savoir faire Should= ConseilCould = suggestion, demande Would = conditionelMay = permission, éventualité (10-15% Will = futurMight = éventualité (5%) Shall= suggestionMust/ Have to = obligation Ought to= Conseil moral

7. ConditionnelConditionel 1If I eat, you will be => If + present est suivi du futur

Conditionel 2If I ate, you would be => If + Prétérit est suivi du conditionel present

Attention, If I WERE => on conjugue le verbe être uniquement à sa forme pluriel, peu importe le sujet!

Conditionel 3If I had eaten, you would have been => If + Past perfect est suivi du conditionel passé

8. SubjonctifPrésentLe subjonctif présent est l’infinitif sans ‘to’ pour tous les verbes y compris ‘be’ et ‘have’. A l’exception du verbe ‘to be’, il ressemble au présent de l’indicatif avec la particularité que la 3ème personne du singulier n’a pas de ‘s’.

It is important that Sue not take this job, it is too dangerous. Fred conseille à Sue de ne pas accepter ce travail, il est trop dangereux.

PasséLe subjonctif passé a la même forme que le prétérit pour tous les verbes à l’exception du verbe ‘to be’ qui fait ‘were’ à toutes les personnes

Page 9: BASES ET RVISIONS NCESSAIRES POUR LE TOEIC · 2019. 12. 2. · BASES ET RVISIONS NCESSAIRES POUR LE TOEIC 3 CONSIGNES Ā APPLIQUER AVANT LE STAGE Révisez les temps présentés dans

9BASES ET RÉVISIONS NÉCESSAIRES POUR LE TOEIC

B. Les comparatifs et les superlatifs

1. Le comparatif de supérioritéAdjectifs courts : Adjectif +ER THANLes adjectifs courts sont des adjectifs d’une syllabe (comme tall, small…) ou de deux syllabes qui se terminent en -y (comme funny, happy…)

Ex : I am taller than you : je suis plus grande que toiYou are happier than her (le « y »se transforme en « i »)

Adjectifs longs : More + Adjectif + THANEx : I am more intelligent than youI am more clever than you (clever est un adjectif long car il ne se termine pas par un “y”)

Les irréguliersgood better bad worse

far farther/furthermany more

2. Le comparatif d’inférioritéLESS + adj +THANEx : I travel less often than you. Your sister is less tall than me.

3. Le comparatif d’égalitéAS + adj +ASEx : I don’t swim as fast as you. She didn’t go to London as regurlarly as I did.

Après THAN et AS: on ne met pas les pronoms sujet mais :- Me - It (chose)- You - Us- Her (fille) - You- Him (garçon) - Them

Page 10: BASES ET RVISIONS NCESSAIRES POUR LE TOEIC · 2019. 12. 2. · BASES ET RVISIONS NCESSAIRES POUR LE TOEIC 3 CONSIGNES Ā APPLIQUER AVANT LE STAGE Révisez les temps présentés dans

10

4. Le superlatif de supérioritéAdjectifs courts : THE adj+ EST + NomEx: He is the smallest man of the class

Adjectifs longs : THE + MOST+ Adj+ NomEx: It is the most surprising thing I have ever heard.

5. Le superlatif d’inférioritéTHE LEAST + Adj + NomEx: It is the least interesting class I have ever been to

C. Les pronomsAdjectifs possessifs

Traduction Pronoms possessifs

Traduction

My mon, ma, mes Mine Le/les mien(s), la/les mienne (s), à moi

Your Ton, ta, tes Yours Le/les tien(s), la/les tienne(s), à toi

Her Son, sa, ses (à elle) Hers Le/les sien(s) féminin, la/les sienne(s) fémi-nin, à elle

His Son, sa, ses (à lui) His Le/les sien(s) masculin, la/les sienne(s) masculin, à lui

Its son, sa, ses (neutre) Its Le/les sien(s) neutre, la/les sienne(s) neutre

Our Notre, nos Ours Le nôtre, la nôtre, les nôtres, à nous

Your Votre, vos Yours Le vôtre, la vôtre, les vôtres, à vous

Their leur, leurs theirs Le leur, la leur, les leurs, à eux

Page 11: BASES ET RVISIONS NCESSAIRES POUR LE TOEIC · 2019. 12. 2. · BASES ET RVISIONS NCESSAIRES POUR LE TOEIC 3 CONSIGNES Ā APPLIQUER AVANT LE STAGE Révisez les temps présentés dans

11BASES ET RÉVISIONS NÉCESSAIRES POUR LE TOEIC

Les adjectifs possessifs Ils se placent toujours devant un nom et sont invariables.On les choisit selon le genre (masculin, féminin, neutre) du possesseur.Exemples: Its branch = sa branche (à l’arbre, genre neutre) Her hat = son chapeau (à elle) His hat = son chapeau (à lui)

Les pronoms possessifsIls remplacent un nom. Ils sont invariables et dépendent du genre du possesseur (masculin, féminin, neutre).Les pronoms possessifs, selon le contexte, peuvent être traduits de différentes manières en français. Exemples:Mine : le mien, la mienne, les miens, les miennes, à moi

D. Les plurielsEn règle générale, la forme plurielle du nom est construite en ajoutant un ‘-s’ à la forme singulièreshoe shoes book books river rivers

Les mots se terminant en ‘s’ ou ‘z’ prendront généralement la terminaison ‘-es’ bus buses kiss kisses

Les mots se terminant en ‘-y’ prendront généralement la terminaison ‘-ies’ party parties supply supplies Certains mots sont irréguliers au plurielone man two men one knife two knivesone woman two women one thief two thievesone foot two feet one dwarf two dwarves (ou: dwarfs)one mouse two mice one potato two potatoesone goose two geese one leaf two leavesone tooth two teeth one life two livesone wife two wives one loaf two loaves one child two children one half two halves

Page 12: BASES ET RVISIONS NCESSAIRES POUR LE TOEIC · 2019. 12. 2. · BASES ET RVISIONS NCESSAIRES POUR LE TOEIC 3 CONSIGNES Ā APPLIQUER AVANT LE STAGE Révisez les temps présentés dans

12

Certains mots rarissimes ne changent pas du tout au pluriel One moose two moose One sheep two sheep One aircraft two aircraft One fish two fish Les mots d’origine latine ou grecqueone alumnus two alumni

one syllabus two syllabi

one alumna two alumnae

one alga many algae

one criterion many criteria

one forum many fora (ou : forums)

one thesis two theses

one hypothesis two hypotheses

one phenomenon two phenomena

one cactus --> two cacti (ou : cactuses)

one diagnosis --> two diagnoses

one oasis --> two oases

one analysis --> two analyses

Page 13: BASES ET RVISIONS NCESSAIRES POUR LE TOEIC · 2019. 12. 2. · BASES ET RVISIONS NCESSAIRES POUR LE TOEIC 3 CONSIGNES Ā APPLIQUER AVANT LE STAGE Révisez les temps présentés dans

13BASES ET RÉVISIONS NÉCESSAIRES POUR LE TOEIC

PARTIE II

Le vocabulaire A. Les verbes irréguliers

Infinitif Prétérite Participe Passé Traductionto fly flew flown Voler

to forbid forbade forbidden Interdireto forget forgot forgotten Oublierto forgive forgave forgiven Pardonnerto freeze froze frozen Geler

to get got got Obtenirto give gave given Donnerto go went gone Aller

to grow grew grown Grandirto have had had Avoirto hear heard heard Entendreto hit hit hit frapper, atteindre

to hold held held Tenirto hurt hurt hurt Blesserto keep kept kept Garderto know knew known savoir, connaîtreto lean leant leant s’appuyerto learn learnt learnt Apprendreto leave left left laisser, quitterto lend lent lent Prêterto let let let permettre, louerto lie lay lain être étenduto let let let permettre, louerto lie lay lain être étendu

to lose lost lost Perdreto make made made faire, fabriquerto mean meant meant Signifierto meet met met (se) rencontrer

Page 14: BASES ET RVISIONS NCESSAIRES POUR LE TOEIC · 2019. 12. 2. · BASES ET RVISIONS NCESSAIRES POUR LE TOEIC 3 CONSIGNES Ā APPLIQUER AVANT LE STAGE Révisez les temps présentés dans

14

Infinitif Prétérite Participe Passé Traductionto pay paid paid Payerto put put put Mettreto quit quit quit cesser (de)to read read read Lireto ring rang rung Sonnerto rise rose risen s’élever, se leverto run ran run Courirto say said said Direto see saw seen Voirto sell sold sold Vendre

to send sent sent Envoyerto set set set Fixer

to shoot shot shot Tirerto show showed shown Montrerto shut shut shut Fermerto sing sang sung Chanterto sit sat sat être assis

to sleep slept slept Dormirto smell smelt smelt sentir (odorat)to speak spoke spokento speed sped sped aller à toute vitesseto spell spelt spelt Epeler

to spend spent spent dépenserto stand stood stood être deboutto steal Stole stolen voler, déroberto swear swore sworn Jurerto swell swelled swollen Enflerto swim swam swum Nagerto take took taken Prendreto teach taught taught Enseignerto tell told told dire, raconter

to think thought thought Penserto thrust thrust thrust Enfoncer

to understand understood understood Comprendreto wake woke woken (se) réveillerto wear wore worn porter (des vêtements)to win won won Gagner

Page 15: BASES ET RVISIONS NCESSAIRES POUR LE TOEIC · 2019. 12. 2. · BASES ET RVISIONS NCESSAIRES POUR LE TOEIC 3 CONSIGNES Ā APPLIQUER AVANT LE STAGE Révisez les temps présentés dans

15BASES ET RÉVISIONS NÉCESSAIRES POUR LE TOEIC

B. Les mots de liaisons- again : De plus, En outre; D’ailleurs- all the same : malgré tout, quand même, tout de même, au demeurant- Also, ... : 1/ De plus - 2/ De même- as a consequence : de ce fait, en conséquence (de quoi)- as a matter of fact : en fait- as a result : de ce fait, en conséquence (de quoi)- at least : 1/ au moins - 2/ du moins- besides : d’ailleurs, du reste- but : mais - by the way : à propos, soit dit en passant- despite this : malgré cela- e.g./eg (du latin exempli gratia) : par ex. / p.e. (par exemple) (lu « for example » en G.B. mais souvent « e g » aux E.U.)- either ... or ... : soit ... soit ..., ou bien ... ou bien ...- even better : bien mieux, mieux encore- first, ... second, ... third, ... : primo, ... secundo, ... tertio, ...- for : car, en effet, « :»- for example : par exemple- for instance : par exemple- for one thing : une raison, c’est que ..., entre autres raisons- neither ... nor ... : ni ... ni ...- nevertheless : néanmoins, toutefois, toujours est-il que, il n’empêche que- nonetheless : néanmoins, toutefois- not just ... but also ... : non pas simplement ... mais aussi ...- not only ... but also ... : non seulement ... mais aussi/également/encore ...- Not only that, ... : Il n’y a pas que cela, ..., Bien plus, ...- on the contrary : au contraire- on the one hand ... on the other hand ... : d’une part ..., d’autre part - furthermore : de plus, en outre- however : cependant, toutefois, pourtant, du reste- i.e./ie (du latin id est) : c.-à-d. (c’est-à-dire), à savoir (lu « that is to say » en G.B. mais souvent « i e » aux E.U.)- if not : sinon- in addition : en sus, de surcroît, en outre- in any case : en tout cas- in/by comparison : en comparaison- in/by contrast : par opposition- in effect : en fait, en réalité- in fact : en fait- in other words : en d’autres termes, autrement dit- in spite of that : malgré cela- in that case : en ce cas là- in the first place, ... in the second place, ... : en premier lieu, ... en second lieu, ...- in the same way : de même- in this case : dans ce cas

Page 16: BASES ET RVISIONS NCESSAIRES POUR LE TOEIC · 2019. 12. 2. · BASES ET RVISIONS NCESSAIRES POUR LE TOEIC 3 CONSIGNES Ā APPLIQUER AVANT LE STAGE Révisez les temps présentés dans

16

- in which case (après virgule) : auquel cas, et dans ce cas- instead (général. en tête de phrase, + virgule) : au lieu de cela- moreover : en outre, de plus- More than that, ... : Bien plus- on top of that (général. en tête de phrase, + virgule) : qui plus est, en plus de cela- or : ou - or else : sinon- otherwise : sinon - Rather, ... : Mieux, Plus exactement- therefore : donc, par conséquent- this is why : c’est pourquoi- thus : ainsi (au sens de donc, par conséquent)- to this end : à cet effet- yet : pourtant, cependant, malgré tout

C. Phrasal verbs- to ask someone out : inviter quelqu’un à sortir

- to back something up : faire une copie de sauvegarde- to back up : faire marche arrière- to back someone up : soutenir quelqu’un

- to break down ; tomber en panne, fondre en larmes- to break into : entrer ar effraction

- to bring someone up : élèver un enfant- to bring something up : évoquer un sujet

- to call someone back : rappeler quelqu’un- to call sthg off : annuler qqchose

- to catch up with someone : se mettre au courant

- to come across : tomber sur qqchose- to come down with sthg : attraper (une maladie)- to cut back on sthg : rédurie- to dress up : bien s’habiller, se mettre sur son 31- to fall out with someone : se disputer avec quelqu’un

- to get along/ to get on with someone : bien s’entendre avec qq’un - to get over sthg : se remettre de qqch

- to give something up : renoncer à qqchose - to give in : cèder, craquer

Page 17: BASES ET RVISIONS NCESSAIRES POUR LE TOEIC · 2019. 12. 2. · BASES ET RVISIONS NCESSAIRES POUR LE TOEIC 3 CONSIGNES Ā APPLIQUER AVANT LE STAGE Révisez les temps présentés dans

17BASES ET RÉVISIONS NÉCESSAIRES POUR LE TOEIC

- to hang up on some one : raccrocher au nez de qq’un- to keep out of sthg : se pas se méler de qqch- to let someone down : laisser tomber qq’un

- to look for sthg : chercher qqch- to look forward to sthg (+ verbe ing) : être impatient de- to look sthg up : chercher- to look after someone : s’occuper de qq’un- to look up to someone : admirer qq’un

- to make up with someone : se réconcilier- to make something up : inventer qqchose- to put up with somethhing or someone : supporter qqchose ou qq’un

- to put aside : économiser- to run into: rencontrer par hasard- to run out of sthg : se retrouver à court de qqchose

- to show off : frimer, se vanter

- to stick to sthg : s’en tenir à qqchose, continuer de- to stand in for someone : rempalcer qq’un- to stand up to someone : tenir tête à qq’un- to stand someone up : poser un lapin à qq’un

- to talk someone into doing something : persuader qq’un de farie qqchose- to throw someone out : expulser qq’un- to turn sthg down : rejeter qqchose- tu wear off : diminuer- to work out : faire du sport- to work something out : résoudre un pb

Page 18: BASES ET RVISIONS NCESSAIRES POUR LE TOEIC · 2019. 12. 2. · BASES ET RVISIONS NCESSAIRES POUR LE TOEIC 3 CONSIGNES Ā APPLIQUER AVANT LE STAGE Révisez les temps présentés dans

18

EXERCICES

Les exercices ci-dessous sont à faire avant le début du stage. Il s’agit d’exercices qui vous permettront de réviser vos bases. Ces exercices seront corrigés en partie ensemble (ou elle sera distribuée.) Il est impératif que vous ayez appris la leçon correspondante avant de les réaliser.

GRAMMAIREA. Les Différents temps

1. Présent simple1. The cinema ______________________________ (close) at 7pm. Le cinéma ferme à 7h.

2. They ______________________________ (not/think) you should buy this dress. Ils ne pensent pas que tu devrais acheter cette robe.

3. We usually ______________________________ (take) a taxi to go to work.Nous prenons habituellement un taxi pour aller au travail.

4. How often ______________________________ (you/go) to the swimming pool? A quelle fréquence allez-vous à la piscine?

5. Courses ______________________________ (begin) on the third of September. Les cours commencent le 3 septembre.

6. When ______________________________ (he/arrive) home in the evenings?Quand arrive-t-il à la maison, le soir?

7. She ______________________________ (not/live) in Washington, but in New York.Elle n’habite pas à Washington, mais à New York.

8. He ______________________________ (get up) early on Mondays.Il se lève tôt le lundi.

9. I ______________________________ (not/believe) in witches.Je ne crois pas en la sorcellerie.

10. The Sun’s rays _________________________ (take) eight minutes to reach the Earth.Les rayons du soleil mettent 8 minutes à atteindre la Terre.

11. On Sundays, she ______________________________ (go) to the swimming pool.

Page 19: BASES ET RVISIONS NCESSAIRES POUR LE TOEIC · 2019. 12. 2. · BASES ET RVISIONS NCESSAIRES POUR LE TOEIC 3 CONSIGNES Ā APPLIQUER AVANT LE STAGE Révisez les temps présentés dans

19BASES ET RÉVISIONS NÉCESSAIRES POUR LE TOEIC

12. He ______________________________ (brush) his teeth before going to bed.

13. Our baby ______________________________ (cry) all day long.

14.Jane ______________________________(buy) beautiful clothes.

15. My parents ______________________________ (not/speak) English, sorry

16.Her sister ______________________________(phone) every day.

17.I am very good at German; I ________________________ (understand) it quite well.

18. I am not good at Spanish ; I _______________________ (not/understand) it at all.

19. ______________________________ (understand/she) Italian ? 20.They have a car, they ______________________________ (not/take) the bus.

21. He has a funny dog. His name is Sugar ; he ___________ (play) with him all the time.

22. ______________________________(know/ you) Mr Smith?

23. It’s 10.30 ! They always ______________________________(get up) late.

24. What time ______________________________ (go/your children) to school ?

25.Harry _______________________ (watch) television from 7.00 to 9.00 every evening.

26. She is hard-working, she always ___________(try) to do her best.

27. Jane ______________________________ (study) psychology at university.

2. Futur simple1. I _____________________(go) to the supermarket

2. After my shopping, I _____________________ (cook) dinner

3. After cooking dinner, I ___________________ (call) my friends

4. ______________________________ (we /dance) with them?

5. After dancing we ________________________(talk) together

6. Then we __________________________(not/read) books but...

7. we ______________________________ (go)outside.

8. We ______________________________(not/meet) friends

9. After that we ______________________________ (get) back home

10. No, I think she ___________________ (not/call) the doctor today, she is feeling better!

Page 20: BASES ET RVISIONS NCESSAIRES POUR LE TOEIC · 2019. 12. 2. · BASES ET RVISIONS NCESSAIRES POUR LE TOEIC 3 CONSIGNES Ā APPLIQUER AVANT LE STAGE Révisez les temps présentés dans

20

11. They ______________________________ (buy) a house next week ! It’s so great!

12. It ______________________________ (be) great for me to go to England!

13. Henry ______________________________ (not/call) us! He is too busy!

14. I __________________ (not/clean) the windows! I don’t like it!

15. ______________________________ (my parents/ live) in Paris?

3 PrétériteExercise 1 : Turn these sentences into past sentences. (The first ten sentences use regu-lar verbs, the other ten use irregular ones.) 1. Margaret plays on the computer. (+)2. Mr Brown works in his office. . (-)3. It rains here. (?)4. I wait for her.(+)5. We listen to the news. (-)6. She laughs at her. (?)7. He asks silly questions. (+)8. He writes his report. (-)9. I walk to my office. (?)10. They watch TV. (+)11. She writes to her friend (-)12. We go to the cinema. (?)13. I get up early (+)14. He drives to the shop. (-)15. Bob drinks whisky. (?)16. She speaks to me. (+)17. The children swim in the sea. (-)18. They have lunch at the restaurant. (?)19. She loses her umbrella. (+)20. Ken feels ill. (-)

Exercice 21. Nous avons travaillé ce matin.2. J’ai écouté les nouvelles à la radio à huit heures.3. Quand avez-vous écrit aux Morgans ?4. A quelle heure as-tu pris ton petit déjeuner ?

Page 21: BASES ET RVISIONS NCESSAIRES POUR LE TOEIC · 2019. 12. 2. · BASES ET RVISIONS NCESSAIRES POUR LE TOEIC 3 CONSIGNES Ā APPLIQUER AVANT LE STAGE Révisez les temps présentés dans

21BASES ET RÉVISIONS NÉCESSAIRES POUR LE TOEIC

5. J’ai acheté deux livres la semaine dernière.6. Mon collègue n’est pas venu avec vous à la réunion. 7. J’ai rencontré Jennifer hier. Elle était très occupée.8. Quand avez-vous lavé la voiture ? Je ne sais pas, je n’étais pas à la maison le mois dernier.9. Pourquoi êtes-vous allé dans ce centre hier soir ?10. Nous avons vu notre patron à la télévision hier. 11. Où as-tu acheté ce livre ? Je ne l’ai pas acheté, je l’ai emprunté à la bibliothèque.12. Je suis allé à New York il y a deux ans. C’était vraiment génial !13. Vous ont-ils attendu ? Non, ils sont partis avec Jim.14. Ils se sont levés tôt ce matin.

4. Present PerfectExercise 1: put these sentences in the present perfect1. Peter ______________________________ (steal) my trainers!

2. I ______________________________ (buy/just) this magazine.

3. We ______________________________ (write) three pages.

4. ______________________________ (ever/you/be) to London?

5. I _________________________ (not/finish) my homework yet.

6. My dog ____________________________ (eat) all my cookies!

Exercise 2: choose between present perfect and preterit1. My uncle ______________________________ (go) on a trip to Egypt three years ago.

2. Roy ______________________________ (finish-just) repairing the washing-machine, you can use it now.

3. His worst memory: when he ___________________________ (have) to swim in the North Sea.

4. When I was a child I ________________________ (believe) in ghosts.

5. Last year, we __________________ (visit) the British Museum.

6. What is the most frightening experience __________________ (you-have-ever)?

7. Many students in my class ______________________________ (travel-already) abroad.

8. I ______________________________ (steal-never) anything in my life.

9. I’m afraid, she ______________________________ (leave) the office an hour ago.

10. I’m afraid, she isn’t here. She _____________________ (leave) the office.

Page 22: BASES ET RVISIONS NCESSAIRES POUR LE TOEIC · 2019. 12. 2. · BASES ET RVISIONS NCESSAIRES POUR LE TOEIC 3 CONSIGNES Ā APPLIQUER AVANT LE STAGE Révisez les temps présentés dans

22

5. Past PerfectMettez les verbes entre parentheses au temps qui convient: preterit ou past perfect. On se souvient bien que le past perfect est utilisée pour une action antérieure à une action déjà passée. There _______ (be) six of us at dinner that night: Mike and his wife and daughter, my wife and I, and a man called Richard. Richard _______ a famous gourmet. I _______ (be) to dinner at Mike’s twice before when Richard _______ (be)there, and on each occasion Mike and his wife _______ (go) out of their way to produce a special meal for the famous gourmet. And this one, clearly _______(be) to be no exception. As we _______ (sit) down, I _______ (remember) that on both Richard’s previous visits Mike _______ (play) a little betting game with him over the wine, challenging him to name its vintage. Pratt _______ (reply) that that should not be difficult provided it _______ (be) one of the great years. Mike _______ (tehn bet) him a case of the wine in question that he _______(can) not do it. Richard _______ (accept), and _______(win) both times. Tonight, I _______ (feel) sure that the little would be played over again.

6. Les modauxI. Using MUST, CAN, MAY, or SHOULD , you will reformulate the following sen-tences. Ex. Alex and Axel look like each other. (Brothers – not twins) They must be brothers, but they can’t be twins.1. Paul lived for ten years in Oxford. (Good English)2. William has his leg in plaster. (To break)3. I phoned Betty several times last night but I got no answer. (To unplug)4. They often fly to Miami on weekends. (Well off )5. Mark can’t find his wallet. (Not to lose, though)6. Clive sends Isabel red roses every day. (In love)7. I could have done it myself. (To help – need)8. They just hate English. (German instead)9. It’s Christmas Eve; I can hear a noise on the roof. (Santa Claus)10. Mr Jones killed Mr Smith. ( Jail)

7. Conditionel1. If you answer all the questions correctly, they ____________________ (write) your nickname in the list of winners!

2. If we go to England, we ________________ (visit) the British Museum.

3. If you were nicer to me, I ___________________ (help) you.

4. Karen ______________________________ (come) if you invite her.

Page 23: BASES ET RVISIONS NCESSAIRES POUR LE TOEIC · 2019. 12. 2. · BASES ET RVISIONS NCESSAIRES POUR LE TOEIC 3 CONSIGNES Ā APPLIQUER AVANT LE STAGE Révisez les temps présentés dans

23BASES ET RÉVISIONS NÉCESSAIRES POUR LE TOEIC

5. My son will be allowed to go on holiday with his friends if he _________________________ (pass) his exam.

6. If he ______________________________ (have) more money, he would go to Italy.

7. If you worked harder, you _____________ (have) better marks.

8. He ______________________________ (buy) you a ticket if you want to come.

9. If I were rich, I __________________________ (buy) a house.

10. If I (not be) ______________________________ so ill, I would come with you.

11. If it rains, I ______________________________ (not go) out.

8. SubjonctifComplètez en mettant les verbes au subjonctif présent ou passé It is essential that lucy __________ her multiplication tables (learn)

If only I _____________ able to read Chinese! ( to be)

It is important that you _____________ too much alcohol while driving. (to drink)

Sorry, the cake is burnt ! I wish I _____________ a better cook! (to be)

In Phoebe(s school it is required that every student _________ a uniform! (to wear)

Bob’s grandfather is ill. It is urgent that Bob _____________ at once (to warn)

The monks insisted that the tourists _____________ the temple with their shoes on. (to enter)

It is too cold here. I wish I _____________ in a warmer country. ( to live)

It is recommended that you _____________ after eating a large meal! (to swim)

Mr. Mason demands that the heater _____________immediatley. His room is too cold (to repair)

Page 24: BASES ET RVISIONS NCESSAIRES POUR LE TOEIC · 2019. 12. 2. · BASES ET RVISIONS NCESSAIRES POUR LE TOEIC 3 CONSIGNES Ā APPLIQUER AVANT LE STAGE Révisez les temps présentés dans

24

ARTICLE DE PRESSECharlemagne

Some Europeans fear a surge of Chinese investment. Others can’t get enough of itThere is more to cheer than jeer about Chinese investment in the EU

To this day the trained eye can still spot the occasional boxy Chinese tractor lumbering around rural Albania, a reminder of the time when this Balkan backwater was China’s biggest champion in Europe. In the 1960s Chinese aid and capital propped up Enver Hoxha’s dire regime in return for its support at the UN, where Taiwan still held the Chinese seat. Now some fear that what comes around goes around. Chinese money is pouring into Europe’s heart and its periphery. It sometimes seems to have a political edge.Public investment in the European Union is at its lowest for 20 years. Little wonder some are looking east. The stock of Chinese investment in Europe is low compared with America’s or Japan’s, but it is shooting up. Accor-ding to one study, in 2016 its new FDI in the EU was over 77% above that the year before, at €35bn ($41bn). These days China’s attention is on the innovation hubs of northern Europe as much as on infrastructure, but crisis-hit southern economies have also lapped up its lucre, especially those that have had to divest state assets under euro-zone bail-outs. Last year Cosco, a Chinese state-owned shipping firm, acquired a controlling stake in the main Greek port of Piraeus, providing Chinese maritime exporters with a European foothold. Portugal is rolling in Chinese loot.

But the latest front is further east. This week Li Keqiang, China’s prime minister, swooped into Budapest for the annual meeting of the snappily titled Co-operation Between China and Central and Eastern European Countries (colloquially known as the 16+1 format), bringing promises worth €3bn. There is talk of investment in Estonian dairy, Slovakian freight and a high-speed rail link from Serbia to Hungary (which may violate EU tendering rules). Beata Szydlo, Poland’s prime minister, grumbled about Chinese red tape, but Viktor Orban, her Hungarian counterpart, was more effusive, celebrating China’s economic heft and its agreeable habit of not talking about democracy or human rights

For some western Europeans all this revives old concerns, and sparks new ones. The first is that in their rush for renminbi some European governments will become proxies for Chinese interests. The fear is hardly groundless. In June Greece vetoed a common EU position at the UN on human rights in China. Earlier, pressure from Hun-gary, Greece and others had watered down an EU statement after an international court had condemned China’s mischief-making in the South China Sea

Balkan countries like Serbia, their accession to the EU years away, may be tempted to see China as a geopolitical hedge against Europe, even though most have little to offer beyond their position on the “Balkan Silk Road” between Piraeus and Europe’s rich heartlands.

Page 25: BASES ET RVISIONS NCESSAIRES POUR LE TOEIC · 2019. 12. 2. · BASES ET RVISIONS NCESSAIRES POUR LE TOEIC 3 CONSIGNES Ā APPLIQUER AVANT LE STAGE Révisez les temps présentés dans

25MÉTHODE POUR GAGNER 50 POINTS AU TEST

OBJECTIF : ENTRE 50 ET 100 POINTS en une semaine !

Voici les points que vous devez IMPERATIVEMENT connaître afin d’optimiser votre semaine de stage. Les liens sont là à titre d’exemples, vous en trouverez de nombreux autres sur les sites que j’utilise. Il est essentiel que ces points de grammaire soient acquis avant le début du stage.

Je sais que le site anglaisfacile.com est horrible à regarder, mais il regorge d’exercices utiles pour progresser. Ne vous arrêtez pas à son apparence très archaïque !

N’hésitez pas à faire plusieurs listening. Il y en a de nombreux disponibles sur Youtube (https://www.youtube.com/watch?v=YBQ-irnWrmY) « toeic listening » dans le moteur de recherche.

Ce qui suit est un guide pour vous aider. Si vous rencontrez des soucis dans le début du stage, rendez-vous sur le groupe Facebook, (https://www.facebook.com/groups/280270212437010/)Nous essayerons de vous aider au mieux !

Bon courage !

Page 26: BASES ET RVISIONS NCESSAIRES POUR LE TOEIC · 2019. 12. 2. · BASES ET RVISIONS NCESSAIRES POUR LE TOEIC 3 CONSIGNES Ā APPLIQUER AVANT LE STAGE Révisez les temps présentés dans

26

1. Grammairea. Prétérit simple https://www.anglaisfacile.com/exercices/exercice-anglais-2/exercice-anglais-51877.php https://www.anglaisfacile.com/exercices/exercice-anglais-2/exercice-anglais-57005.php

b. Prétérit progressif https://www.anglaisfacile.com/exercices/exercice-anglais-2/exercice-anglais-71813.php

c. Verbes irréguliers : (désolée, il n’y a pas le choix !) https://www.usingenglish.com/quizzes/31.html

Irregular Verbs

You must of course learn your irregular verbs, but here are the tricky ones to remember!

saw sawed sawn scierdwell dwelt dwelt habiterflee fled fled fuir/s’enfuirlean leant leant s’appuyerleap leapt leapt sautermow mowed mown tondre (la pelouse)shoe shod shod chaussershrink shrank shrunk rétrécirsink sank sunk s’enfoncer/coulerslide slid slid glisserspill spilt spilt renverser (un liquide)spit spat spat crachersplit split split (se)séparerspread spread spread étalerspring sprang sprung bondir/jaillirswell swelled swollen enflerswing swung swung (se)balancerthrust thrust thrust pousser/enfoncerundergo underwent undergone subirwind wound wound enroulerTo creep crept crep tramperTo cast cast cast jeter/lancerTo lay lay lain mettre/poseràplat

Present Perfect httPs://www.usIngenglIsh.com/quIzzes/291.html httPs://www.usIngenglIsh.com/quIzzes/290.html

d. Pronoms https://www.usingenglish.com/quizzes/73.html

e. Countables & uncountables https://www.usingenglish.com/quizzes/432.html

Page 27: BASES ET RVISIONS NCESSAIRES POUR LE TOEIC · 2019. 12. 2. · BASES ET RVISIONS NCESSAIRES POUR LE TOEIC 3 CONSIGNES Ā APPLIQUER AVANT LE STAGE Révisez les temps présentés dans

27MÉTHODE POUR GAGNER 50 POINTS AU TEST

f. Gerunds & infinitives https://www.usingenglish.com/quizzes/420.html

g. Phrasal Verbs https://www.usingenglish.com/quizzes/81.html (il y a des milliers d’exercices sur ce point, il faut réussir à en retenir le plus possible!)

Si certains de ces quizzes vous posent problème, il est important d’attendre les règles associées.

2. Vocabulairea. Question words: https://www.usingenglish.com/quizzes/311.html

b. Leçons de vocabulaire sur Prepmyfutre

c. Exercices de vocabulaire

Les traductions des mots de vocabulaire ne sont volontairement pas inclus afin que vous réfléchissiez d’abord à ceux que vous connaissez sans le dictionnaire. Allez ensuite chercher ceux que vous ne connaissez pas. (bien sûr, il faut ensuite les apprendre !) N’utilisez pas Reverso ou Google traduction ! Utilisez Wordreference.fr, c’est un traducteur très fiable.

fInd the translatIon and learn these words!A. Around the business: Nouns

• advantage• advertisement• advice• agenda• apology• authorization•bill•brand•budget• commission• comparison• competition• competitor• confirmation• costs• creditor• customer•deadline•debt

•debtor•decision•decrease•deficit•delivery•department•description•difference•disadvantage•distribution• employee• employer• enquiry• environment• equipment• estimate• experience• explanation• facilities

• factory• feedback•goal•goods•growth•guarantee• improvement• increase• industry• instructions• interest• inventory• invoice•knowledge• limit• loss•margin•market•message

Page 28: BASES ET RVISIONS NCESSAIRES POUR LE TOEIC · 2019. 12. 2. · BASES ET RVISIONS NCESSAIRES POUR LE TOEIC 3 CONSIGNES Ā APPLIQUER AVANT LE STAGE Révisez les temps présentés dans

28

•mistake•objective•offer•opinion•option•order•output•payment•penalty•permission•possibility•product•production•profit

•promotion•purchase• reduction• refund• reminder• repairs• report• responsibility• result• retailer• rise• risk• salary• sales

• schedule• share• signature• stock• success• suggestion• supply• support• target• transport• turnover•wholesaler

B. Aroundbusiness: Verbs

• accept• add• admit• advertise• advise• afford• approve• authorize• avoid•borrow•build•buy• calculate• cancel• change• charge• check• choose• complain• complete• confirm

• consider• convince• count•decide•decrease•deliver•develop•discount•dismiss•dispatch•distribute•divide• employ• encourage• establish• estimate• exchange• extend•fix• fund• improve

• increase• inform• install• invest• invoice• join• lend• lengthen• lower•maintain•manage•measure•mention•obtain•order•organize•owe•own•pack•participate•pay

Page 29: BASES ET RVISIONS NCESSAIRES POUR LE TOEIC · 2019. 12. 2. · BASES ET RVISIONS NCESSAIRES POUR LE TOEIC 3 CONSIGNES Ā APPLIQUER AVANT LE STAGE Révisez les temps présentés dans

29MÉTHODE POUR GAGNER 50 POINTS AU TEST

•plan•present•prevent•process•produce•promise•promote•provide•purchase• raise• reach

• receive• recruit• reduce• refuse• reject• remind• remove• reply• resign• respond• return

• rise• sell• send• separate• shorten• split• structure• succeed• suggest

C. Job application

• address• age• apply•birthday• certification• children• citizenship• city• college• country• criminal record•date•date of birth•dependents•diploma•divorced• education• educational background• elementary school• employee• employer• experience• family name

• felony record• female•fired•first name•gender•graduate school•handicap•hearing•height•high school•husband’s name• junior college• junior high school• last name•male•marital status•married•middle name•name•next of kin•notify•occupation•phone number

•physical examination•position•present address•previous experience•quit• reason for leaving• relatives• salary• separated• single• skills• social security number• state• street• telephone number• vision•weight•wife’s name• zip code

Page 30: BASES ET RVISIONS NCESSAIRES POUR LE TOEIC · 2019. 12. 2. · BASES ET RVISIONS NCESSAIRES POUR LE TOEIC 3 CONSIGNES Ā APPLIQUER AVANT LE STAGE Révisez les temps présentés dans

30

OBJECTIF : +100 POINTS en une semaine !

Voici les points que vous devez IMPERATIVEMENT connaître afin d’optimiser votre semaine de stage. Les liens sont là à titre d’exemples, vous en trouverez de nombreux autres sur les sites que j’utilise. Il est essentiel que ces points de grammaire soient acquis avant le début du stage.

Je sais que le site anglaisfacile.com est horrible à regarder, mais il regorge d’exercices utiles pour progresser. Ne vous arrêtez pas à son apparence très archaïque !

N’hésitez pas à faire plusieurs listening. Il y en a de nombreux disponibles sur Youtube (https://www.youtube.com/watch?v=YBQ-irnWrmY) « toeic listening » dans le moteur de recherche.

Ce qui suit est un guide pour vous aider. Si vous rencontrez des soucis dans le début du stage, rendez-vous sur le groupe Facebook, (https://www.facebook.com/groups/280270212437010/) Nous essayerons de vous aider au mieux !

Bon courage !

Page 31: BASES ET RVISIONS NCESSAIRES POUR LE TOEIC · 2019. 12. 2. · BASES ET RVISIONS NCESSAIRES POUR LE TOEIC 3 CONSIGNES Ā APPLIQUER AVANT LE STAGE Révisez les temps présentés dans

31MÉTHODE POUR GAGNER 100 POINTS AU TEST

1. Grammairea. Présent simple https://www.anglaisfacile.com/exercices/exercice-anglais-1/exercice-anglais-15.php

b. Present progressif: https://www.anglaisfacile.com/exercices/exercice-anglais-2/exercice-anglais-51877.php

c. Prétérit simple https://www.anglaisfacile.com/exercices/exercice-anglais-2/exercice-anglais-51877.php

d. https://www.anglaisfacile.com/exercices/exercice-anglais-2/exercice-anglais-57005.php

e. Prétérit progressif https://www.anglaisfacile.com/exercices/exercice-anglais-2/exercice-anglais-71813.php

f. Verbes irréguliers : (désolée, il n’y a pas le choix !)

g. https://www.usingenglish.com/quizzes/31.html

Irregular VerbsYou must of course learn your irregular verbs, but here are the tricky ones to remember!

saw sawed sawn scierdwell dwelt dwelt habiterflee fled fled fuir/s’enfuirlean leant leant s’appuyerleap leapt leapt sautermow mowed mown tondre (la pelouse)shoe shod shod chaussershrink shrank shrunk rétrécirsink sank sunk s’enfoncer/coulerslide slid slid glisserspill spilt spilt renverser (un liquide)spit spat spat crachersplit split split (se) séparerspread spread spread étalerspring sprang sprung bondir/jaillirswell swelled swollen enflerswing swung swung (se) balancerthrust thrust thrust pousser/enfoncerundergo underwent undergone subirwind wound wound enroulerTo creep crept crept ramperTocast cast cast jeter/lancerTo lay lay lain mettre/poser à plat

Page 32: BASES ET RVISIONS NCESSAIRES POUR LE TOEIC · 2019. 12. 2. · BASES ET RVISIONS NCESSAIRES POUR LE TOEIC 3 CONSIGNES Ā APPLIQUER AVANT LE STAGE Révisez les temps présentés dans

32

h. Present perfect https://www.usingenglish.com/quizzes/291.html

i. https://www.usingenglish.com/quizzes/290.html

j. Pronoms https://www.usingenglish.com/quizzes/73.html

Si certains de ces quizzes vous posent problème, il est important d’attendre les règles associées.

2. Vocabulairea. Question words: https://www.usingenglish.com/quizzes/311.html

b. Leçons de vocabulaire sur Prepmyfutre

c. Exercices de vocabulaire

Les traductions des mots de vocabulaire ne sont volontairement pas inclus afin que vous réfléchissiez d’abord à ceux que vous connaissez sans le dictionnaire. Allez ensuite chercher ceux que vous ne connaissez pas. (bien sûr, il faut ensuite les apprendre !) N’utilisez pas Reverso ou Google traduction ! Utilisez Wordreference.fr, c’est un traducteur très fiable.

fInd the translatIon and learn these words!A. Around the business: Nouns

• advantage• advertisement• advice• agenda• apology• authorization•bill•brand•budget• commission• comparison• competition• competitor• confirmation• costs• creditor• customer•deadline

•debt•debtor•decision•decrease•deficit•delivery•department•description•difference•disadvantage•distribution• employee• employer• enquiry• environment• equipment• estimate• experience

• explanation• facilities• factory• feedback•goal•goods•growth•guarantee• improvement• increase• industry• instructions• interest• inventory• invoice•knowledge• limit• loss

Page 33: BASES ET RVISIONS NCESSAIRES POUR LE TOEIC · 2019. 12. 2. · BASES ET RVISIONS NCESSAIRES POUR LE TOEIC 3 CONSIGNES Ā APPLIQUER AVANT LE STAGE Révisez les temps présentés dans

33MÉTHODE POUR GAGNER 100 POINTS AU TEST

•margin•market•message•mistake•objective•offer•opinion•option•order•output•payment•penalty•permission•possibility•product

•production•profit•promotion•purchase• reduction• refund• reminder• repairs• report• responsibility• result• retailer• rise• risk• salary

• sales• schedule• share• signature• stock• success• suggestion• supply• support• target• transport• turnover•wholesaler

B. Around business: Verbs

• accept• add• admit• advertise• advise• afford• approve• authorize• avoid•borrow•build•buy• calculate• cancel• change• charge• check• choose• complain• complete

• confirm• consider• convince• count•decide•decrease•deliver•develop•discount•dismiss•dispatch•distribute•divide• employ• encourage• establish• estimate• exchange• extend•fix

• fund• improve• increase• inform• install• invest• invoice• join• lend• lengthen• lower•maintain•manage•measure•mention•obtain•order•organize•owe•own

Page 34: BASES ET RVISIONS NCESSAIRES POUR LE TOEIC · 2019. 12. 2. · BASES ET RVISIONS NCESSAIRES POUR LE TOEIC 3 CONSIGNES Ā APPLIQUER AVANT LE STAGE Révisez les temps présentés dans

34

•pack•participate•pay•plan•present•prevent•process•produce•promise•promote•provide•purchase

• raise• reach• receive• recruit• reduce• refuse• reject• remind• remove• reply• resign• respond

• return• rise• sell• send• separate• shorten• split• structure• succeedsuggest

C. Job application

• address• age• apply•birthday• certification• children• citizenship• city• college• country• criminal record•date•date of birth•dependents•diploma•divorced• education• educational background• elementary school• employee• employer• experience• family name

• felony record• female•fired•first name•gender•graduate school•handicap•hearing•height•high school•husband’s name• junior college• junior high school• last name•male•marital status•married•middle name•name•next of kin•notify•occupation•phone number

•physical examination•position•present address•previous experience•quit• reason for leaving• relatives• salary• separated• single• skills• social security number• state• street• telephone number• vision•weight•wife’s name• zip code

Page 35: BASES ET RVISIONS NCESSAIRES POUR LE TOEIC · 2019. 12. 2. · BASES ET RVISIONS NCESSAIRES POUR LE TOEIC 3 CONSIGNES Ā APPLIQUER AVANT LE STAGE Révisez les temps présentés dans

35TEST BLANC 1.

Ce livre est spécialement dédié au nouveau TOEIC.

Vous retrouverez une partie d’entraînement au TOEIC dans le livre.

Pour la partie listening du test d’entrainement, vous pourrez retrouver la piste audio avec les différents liens proposes en bas.

QR CODE de téléchargement du fichier audio :

Vidéos intéractives des Toeic d’entraînement :

https://www.youtube.com/watch?v=QabanioYj9Q&t=2s

Retrouvez toutes les informations et les tests sur Toeicator.com

Page 36: BASES ET RVISIONS NCESSAIRES POUR LE TOEIC · 2019. 12. 2. · BASES ET RVISIONS NCESSAIRES POUR LE TOEIC 3 CONSIGNES Ā APPLIQUER AVANT LE STAGE Révisez les temps présentés dans

36

Page 37: BASES ET RVISIONS NCESSAIRES POUR LE TOEIC · 2019. 12. 2. · BASES ET RVISIONS NCESSAIRES POUR LE TOEIC 3 CONSIGNES Ā APPLIQUER AVANT LE STAGE Révisez les temps présentés dans

37TEST BLANC 1.

TOEIC L&R PRACTICE TEST 1

Play the file TOEICL&R.PracticeTest1.Listening.mp3

Listening SectionIn the Listening section of the test, you are asked to demonstrate how well you understand spoken English. It consists of four parts, with special directions in each part. Remember, you are not allowed to take notes. This section will last approximately 45 minutes.

Part 1Directions: In Part 1 of the test, you will be shown 6 photographs and hear four statements about each pho-tograph. The statements will be marked (A), (B), (C), and (D). All statements will be spoken only once and not written down. For each photograph, choose the statement that accurately describes what can be seen in the photograph.

Look at a sample photograph.

Listen to the four statements.

Statement (C), “Palm-trees are planted along the waterfront,” is the most accurate description of what can be seen in the photograph. Therefore, you should mark your answer choice (C).

This part will now start with Question 1. Select your answers as you listen.

Page 38: BASES ET RVISIONS NCESSAIRES POUR LE TOEIC · 2019. 12. 2. · BASES ET RVISIONS NCESSAIRES POUR LE TOEIC 3 CONSIGNES Ā APPLIQUER AVANT LE STAGE Révisez les temps présentés dans

38

1.

2.

3.

4.

5.

6.

Page 39: BASES ET RVISIONS NCESSAIRES POUR LE TOEIC · 2019. 12. 2. · BASES ET RVISIONS NCESSAIRES POUR LE TOEIC 3 CONSIGNES Ā APPLIQUER AVANT LE STAGE Révisez les temps présentés dans

39TEST BLANC 1.

4.

5.

6.

Part 2Directions: In Part 2 of the test, you will listen to 25 questions, each followed by three potential responses. The responses will be marked (A), (B), and (C). All questions and responses will be spoken only once and not written down. For each question, choose the response that accurately answers it.

This part will now start with Question 7. Select your answers as you listen.

7. Mark your answer on the answer sheet.8. Mark your answer on the answer sheet.9. Mark your answer on the answer sheet.10. Mark your answer on the answer sheet.11. Mark your answer on the answer sheet.12. Mark your answer on the answer sheet.13. Mark your answer on the answer sheet.14. Mark your answer on the answer sheet.15. Mark your answer on the answer sheet.16. Mark your answer on the answer sheet.17. Mark your answer on the answer sheet.18. Mark your answer on the answer sheet.19. Mark your answer on the answer sheet.20. Mark your answer on the answer sheet.21. Mark your answer on the answer sheet.22. Mark your answer on the answer sheet.23. Mark your answer on the answer sheet.24. Mark your answer on the answer sheet.25. Mark your answer on the answer sheet.26. Mark your answer on the answer sheet.27. Mark your answer on the answer sheet.28. Mark your answer on the answer sheet.29. Mark your answer on the answer sheet.30. Mark your answer on the answer sheet.31. Mark your answer on the answer sheet.

Page 40: BASES ET RVISIONS NCESSAIRES POUR LE TOEIC · 2019. 12. 2. · BASES ET RVISIONS NCESSAIRES POUR LE TOEIC 3 CONSIGNES Ā APPLIQUER AVANT LE STAGE Révisez les temps présentés dans

40

Part 3Directions: In Part 3 of the test, you will listen to 13 conversations. Every conversation will be followed by three questions about it. Each question will have four answer choices marked (A), (B), (C), and (D). Each conversa-tion will be spoken only once and not written out. For each question, choose the answer choice that accurately answers it.

32. When was the woman planning to do the trades?(A) Today(B) Tomorrow(C) Next week(D) Yesterday

33. Why is today a good day for making trades?(A) The stock market is down.(B) The stock market is up.(C) The mutual funds are down.(D) The mutual funds are up.

34. What should the woman do after completing the trades?(A) Review the transactions with the man(B) Buy more stocks with the leftover money(C) Begin working on the next trades(D) Email the man the new amounts in the accounts

---

35. Where does this conversation likely take place?(A) At a train station(B) At a sailing club(C) At a bus station(D) At an airport

36. What has the man inadvertently done?(A) Petted a dog he shouldn’t have touched(B) Brought too much luggage with him(C) Traveled with prohibited food(D) Confiscated the woman’s passport

37. What can be inferred about the man and woman?(A) The man will stay at the woman’s house.(B) The man will drive the woman somewhere.(C) The woman did not have to wait long for the man.(D) The woman finds the man irritating.

---

Page 41: BASES ET RVISIONS NCESSAIRES POUR LE TOEIC · 2019. 12. 2. · BASES ET RVISIONS NCESSAIRES POUR LE TOEIC 3 CONSIGNES Ā APPLIQUER AVANT LE STAGE Révisez les temps présentés dans

41TEST BLANC 1.

38. What problem does the man have?(A) He can’t read his book.(B) He has lost something he was keeping.(C) He doesn’t understand financial records.(D) He is colorblind.

39. What can be inferred about the company?(A) It is quite large.(B) It is losing money.(C) It was founded by the man.(D) It is run by the woman.

40. What does the man think would be a good idea?(A) To change colors(B) To make some charts(C) To make money(D) To speak with a coworker

---

41. Where does this conversation take place?(A) On the phone(B) In a doctor’s office(C) At the man’s home(D) At the man’s office

42. What paperwork does the man need to complete?(A) An insurance form(B) An emergency contact list(C) A medical record request(D) His schedule of appointments

43. What can be inferred about the man?(A) He is disorganized.(B) He has many health problems.(C) He will visit the doctor soon.(D) He doesn’t have much family.

---

44. What crime has ACNI Bank been accused of?(A) Illegal layoffs(B) Money laundering(C) Insider trading(D) Fraud

Page 42: BASES ET RVISIONS NCESSAIRES POUR LE TOEIC · 2019. 12. 2. · BASES ET RVISIONS NCESSAIRES POUR LE TOEIC 3 CONSIGNES Ā APPLIQUER AVANT LE STAGE Révisez les temps présentés dans

42

45. What has been the shareholders’ response to this accusation?(A) They have been fully supportive of the bank.(B) They have organized a group lawsuit.(C) They are protesting outside the bank.(D) They are demanding government intervention.

46. What prediction does the man make?(A) That many people will be fired(B) That the bank will close(C) That the bank will reopen that afternoon(D) That the government will bail out the bank

---

47. What task will the woman complete today?(A) Fire an employee(B) Hire an employee(C) Promote an employee(D) Demote an employee

48. What is one of Pedro’s characteristics?(A) Intuitiveness(B) Humor(C) Aggressiveness(D) Kindness

49. What can be inferred about Pedro?(A) He has a family to support.(B) He is fairly young.(C) He responds well to professional development.(D) He has successfully completed the training with the man.

---

50. What is the woman’s specialty?(A) Designing websites(B) Developing software(C) Calculating project costs(D) Overhauling computers

51. What can be inferred about the man?(A) He enjoys working with technology.(B) He has a strict limit on the amount of money he spends.(C) He owns a large company.(D) He wants to move his business online.

Page 43: BASES ET RVISIONS NCESSAIRES POUR LE TOEIC · 2019. 12. 2. · BASES ET RVISIONS NCESSAIRES POUR LE TOEIC 3 CONSIGNES Ā APPLIQUER AVANT LE STAGE Révisez les temps présentés dans

43TEST BLANC 1.

52. How many hours of work can the man afford?(A) 14(B) 40(C) 100(D) 4,000

---

53. What is the context of this conversation?(A) A scholarship interview(B) A political debate(C) A question-and-answer session(D) A job interview

54. How many people had the woman spoken to before her conversation with the man?(A) 2(B) 3(C) 4(D) 5

55. When will the two people speak again?(A) Never(B) In the next few days(C) In a week or two(D) Within a couple months

---

56. Why does the man ask for assistance?(A) He can’t read the information.(B) He is late.(C) He is not a local.(D) He is lost.

57. How long will the man’s train ride be?(A) One hour(B) Two hours(C) Three hours(D) Four hours

58. What platform should the man go to?(A) 3(B) 13(C) 23(D) 30

---

Page 44: BASES ET RVISIONS NCESSAIRES POUR LE TOEIC · 2019. 12. 2. · BASES ET RVISIONS NCESSAIRES POUR LE TOEIC 3 CONSIGNES Ā APPLIQUER AVANT LE STAGE Révisez les temps présentés dans

44

59. What meal are the clients about to have?(A) Breakfast(B) Lunch(C) Coffee break(D) Dinner

60. Which of the following is probable regarding the soup?(A) It is unpalatable.(B) It is medium-spicy.(C) It is quite perfumed.(D) It might contain substances the man is allergic to.

61. What main course will the woman have?(A) Steak(B) Duck(C) Salad(D) Octopus

---

62. What is the setting of this conversation?(A) A private meeting(B) A computer class(C) A university lecture(D) A presentation

63. Where are the graphs the woman is looking for?(A) In a folder(B) In a drawer(C) On the desktop(D) On her chair

64. Look at the graphic. What is the proposed share of expenses on projects?(A) Under 10%(B) About 25%(C) Roughly one-third(D) Slightly over a half

Page 45: BASES ET RVISIONS NCESSAIRES POUR LE TOEIC · 2019. 12. 2. · BASES ET RVISIONS NCESSAIRES POUR LE TOEIC 3 CONSIGNES Ā APPLIQUER AVANT LE STAGE Révisez les temps présentés dans

45TEST BLANC 1.

---

Welcome to FDBG ltd.1st floor Sales2nd floor Advertising, Marketing, Customer Service 3rd floor Engineering, R&D4th floor Accounting, Finance

65. What are the man’s and the woman’s jobs?(A) Deliveryman and receptionist(B) Packager and secretary(C) Repairman and receptionist(D) Repairman and secretary

66. Which of the following is true about the package?(A) It is so big that the man needs to take the elevator.(B) The woman doesn’t want to take it.(C) It has to be given to Mr. Yannatos personally.(D) It contains important documents.

67. Look at the graphic. To which floor does the man need to go?(A) 1st(B) 2nd(C) 3rd(D) 4th

---

Salvatore Carducci and Friendsat La Aquila TheaterJanuary 25, 7 p.m.Row 8, Seat 25Price: 100 €, paid by credit cardNo parking on-site. Non-smoking facility.

68. What can be assumed about the show?(A) It is inexpensive.(B) Tickets for it are hard to get.(C) The man will perform in it.(D) It takes place next week.

69. Look at the graphic. How much will the woman pay the man?(A) 7 euros(B) 25 euros(C) 80 euros(D) 100 euros

Page 46: BASES ET RVISIONS NCESSAIRES POUR LE TOEIC · 2019. 12. 2. · BASES ET RVISIONS NCESSAIRES POUR LE TOEIC 3 CONSIGNES Ā APPLIQUER AVANT LE STAGE Révisez les temps présentés dans

46

70. When will the woman pay the man?(A) Now(B) Tomorrow(C) When she orders more checks(D) She hasn’t decided yet.

Part 4Directions: In Part 4 of the test, you will listen to 10 talks. Every talk will be followed by three questions about it. Each question will have four answer choices marked (A), (B), (C), and (D). Each talk will be spoken only once and not written out. For each question, choose the answer choice that accurately answers it.

71. According to the talk, what is a common concern today?(A) Making enough money(B) Ensuring children grow up safely(C) Being able to retire early(D) Investing money well

72. What is the stated problem with the savings accounts?(A) They are not highly valued by investors.(B) Banks are not responsible for them.(C) People don’t earn much because of low interest rates.(D) They are less secure than the housing market investments.

73. What recommendation does the speaker give to the audience?(A) Get a financial planner(B) Buy a new mattress(C) Invest in the housing market(D) Create a detailed plan for retirement

---

74. Where is this announcement given?(A) At a hotel(B) On an airplane(C) On a ship(D) On a train

75. According to the announcement, what is happening by the main pool?(A) People can go swimming.(B) Drinks will be served.(C) Snacks can be purchased.(D) A performance will take place.

76. Where can people find food at 11:05 p.m.?(A) In the forward dining room(B) On the main deck

Page 47: BASES ET RVISIONS NCESSAIRES POUR LE TOEIC · 2019. 12. 2. · BASES ET RVISIONS NCESSAIRES POUR LE TOEIC 3 CONSIGNES Ā APPLIQUER AVANT LE STAGE Révisez les temps présentés dans

47TEST BLANC 1.

(C) In the minibars(D) At the casino

---

77. What product is being discussed?(A) Lotion(B) Cream(C) Soap(D) Shampoo

78. What problem does this product address?(A) Dry skin(B) Oily skin(C) Painful blisters(D) Uncomfortable sores

79. What department does the speaker most likely work in?(A) Research & Development(B) Accounting(C) Human Resources(D) Advertising

---

80. What does the employee handbook look like?(A) Thick with a gray cover(B) Thick with a green cover(C) Thin with a gray cover(D) Thin with a green cover

81. Which of the following topics does the handbook most likely cover?(A) Negotiating salaries(B) Dealing with problematic clients(C) Setting up email on a smartphone(D) Transportation to work

82. What is a stated benefit of the handbook?(A) It is attractive to have on a desk.(B) It saves time.(C) It forces employees to make meetings.(D) It has taken years to create.

---

83. How many metro lines stop not far from the building?(A) 1(B) 2

Page 48: BASES ET RVISIONS NCESSAIRES POUR LE TOEIC · 2019. 12. 2. · BASES ET RVISIONS NCESSAIRES POUR LE TOEIC 3 CONSIGNES Ā APPLIQUER AVANT LE STAGE Révisez les temps présentés dans

48

(C) 3(D) 4

84. What is an added benefit of the bike trail?(A) People use it to exercise during breaks.(B) Employees enjoy having lunch on benches along the trail.(C) It connects the building to the eastern part of the city.(D) It is unpaved and so is friendlier to the environment.

85. What question is the speaker most probably responding to?(A) “What is the company’s bike-to-work policy?”(B) “Does the company promote green living?”(C) “Is the office easily accessible without a car?”(D) “What resources does the company provide for its employees?”

---

86. What can be inferred about Daniela Fernandez?(A) She specializes in international business.(B) She is Mexican.(C) She is retired.(D) She owns her own company.

87. What should a person do to find a professor’s extension?(A) Dial any extension at any time(B) Press 1(C) Press 2(D) Press 3

88. What can be inferred about the school?(A) It offers night classes.(B) Many students speak languages other than English.(C) It offers full-tuition scholarships.(D) It has a number of locations.

---

89. Where does this talk most probably take place?(A) At an employee mediation office(B) At a management seminar(C) At a new employee orientation session(D) At a professional development course

90. What concern has Anita raised?(A) Her chair is uncomfortable.(B) She dislikes her office location.(C) She doesn’t receive emails from Jamie.(D) She doesn’t know when projects are due.

Page 49: BASES ET RVISIONS NCESSAIRES POUR LE TOEIC · 2019. 12. 2. · BASES ET RVISIONS NCESSAIRES POUR LE TOEIC 3 CONSIGNES Ā APPLIQUER AVANT LE STAGE Révisez les temps présentés dans

49TEST BLANC 1.

91. What suggestion does the speaker make?(A) Anita and Jamie should go to dinner together.(B) Anita and Jamie should use technology to solve their problem.(C) Anita should be more open-minded.(D) Jamie should be more polite.

---

92. What is the context of this speech?(A) A promotional event(B) An awards ceremony(C) A wedding(D) A retirement party

93. According to the speech, what has contributed to the speaker’s experiences?(A) A strong company culture(B) The other employees at the company(C) The speaker’s personal growth(D) Great fortune and luck

94. Who is Ioana?(A) The speaker’s manager(B) The speaker’s wife(C) The speaker’s assistant(D) The speaker’s best friend

---

Visa Application GuidePhotograph Guidelines

between 3,5 and 4,5 cm Glossy or matteFull-color only (no black and white!)On white backgroundNo spectacles

betw

een

5,5

and

6,5

cm

photograph

95. Look at the graphic. Most likely, what was the size of the photograph that Ms. Heikkinen submitted?(A) Height 5,0 cm, width 4,0 cm(B) Height 5,5 cm, width 3,5 cm(C) Height 6,0 cm, width 4,0 cm(D) Height 6,5 cm, width 3,5 cm

Page 50: BASES ET RVISIONS NCESSAIRES POUR LE TOEIC · 2019. 12. 2. · BASES ET RVISIONS NCESSAIRES POUR LE TOEIC 3 CONSIGNES Ā APPLIQUER AVANT LE STAGE Révisez les temps présentés dans

50

96. When will Ms. Heikkinen most likely travel?(A) The following day(B) Within the week(C) Within a month(D) Within three months

97. What is the purpose of Ms. Heikkinen’s trip?(A) To attend business meetings with clients(B) To permanently relocate for business(C) To visit family and friends(D) To have a vacation

---

Sunglasses – PricesGrasshopper 4N $50Butterfly 3N $60Dragonfly 5B $70Cicada 2G $80

98. What is Sven’s job?(A) Salesman(B) Model(C) Web content manager(D) Glass engineer

99. How many boutiques does the company have?(A) 1(B) 2(C) 3(D) 10

100. Look at the graphic. What will be the new price of the model Butterfly 3N?(A) $50(B) $54(C) $60(D) $66

This is the end of the Listening section.

Page 51: BASES ET RVISIONS NCESSAIRES POUR LE TOEIC · 2019. 12. 2. · BASES ET RVISIONS NCESSAIRES POUR LE TOEIC 3 CONSIGNES Ā APPLIQUER AVANT LE STAGE Révisez les temps présentés dans

51TEST BLANC 1.

Reading SectionIn the Reading section of the test, you are asked to demonstrate how well you understand written English. It consists of three parts, with special directions in each part. Remember, you are not allowed to take notes. This section will last exactly 75 minutes.

Part 5Directions: You will read a sentence that contains a blank as well as four answer choices that suggest words to fill the empty space. Select the answer choice (A), (B), (C), or (D) that best completes the sentence.

101. Would you mind _________ this in the drawer?(A) to put(B) put(C) puts(D) putting

102. Be ready at noon so we _________ leave on time.(A) can(B) could(C) should(D) would

103. Are you not coming to _________ meeting?(A) a(B) an(C) the(D) those

104. Have you scheduled your trip _________?(A) therefore(B) yet(C) after(D) now

105. I’m supposed to pay this time, _________(A) am I?(B) aren’t I?(C) do I?(D) didn’t I?

106. Do you prefer the metro _________ the bus?(A) if(B) nor(C) or(D) but

Page 52: BASES ET RVISIONS NCESSAIRES POUR LE TOEIC · 2019. 12. 2. · BASES ET RVISIONS NCESSAIRES POUR LE TOEIC 3 CONSIGNES Ā APPLIQUER AVANT LE STAGE Révisez les temps présentés dans

52

107. There are many good doctors here, but of them Dr. Matthews is the _________.(A) good(B) better(C) best(D) worst

108. It is confirmed that I _________ pick you up at 8 p.m. tonight.(A) shall(B) would(C) could(D) ought

109. _________ anyone checked the copyright rules about this?(A) Does(B) Do(C) Have(D) Has

110. _________ will the new construction project be located?(A) Who(B) What(C) Where(D) When

111. Should we call an engineer in to _________ on the project?(A) refer(B) consult(C) conduct(D) determine

112. The researchers _________ tirelessly for years to find a cure.(A) work(B) are working(C) are worked(D) have been working

113. How _________ does the software developer charge per hour?(A) much(B) many(C) often(D) late

114. I’d like to ship this package the _________ way possible.(A) quick(B) quicker(C) most quick(D) quickest

Page 53: BASES ET RVISIONS NCESSAIRES POUR LE TOEIC · 2019. 12. 2. · BASES ET RVISIONS NCESSAIRES POUR LE TOEIC 3 CONSIGNES Ā APPLIQUER AVANT LE STAGE Révisez les temps présentés dans

53TEST BLANC 1.

115. The client threatened to sue, _________ I have my doubts he will go through with it.(A) but(B) so(C) if(D) and

116. Would I need an _________ bank or a commercial bank?(A) irregular(B) investment(C) equality(D) associated

117. If you _________ your taxes on time, you wouldn’t owe late fees.(A) will pay(B) have paid(C) had paid(D) have been paying

118. No one _________ remembers to factor in travel time.(A) ever(B) never(C) neither(D) either

119. You haven’t put away the new inventory, _________(A) didn’t you?(B) did you?(C) haven’t you?(D) have you?

120. If I _________ you, I would upgrade to a touchscreen phone.(A) was(B) were(C) would be(D) will be

121. The heater _________ by the power surge five days ago.(A) is broken(B) broke(C) was broken(D) had been broken

122. _________ bonuses be announced soon?(A) Will(B) Has to(C) Had better(D) Do

Page 54: BASES ET RVISIONS NCESSAIRES POUR LE TOEIC · 2019. 12. 2. · BASES ET RVISIONS NCESSAIRES POUR LE TOEIC 3 CONSIGNES Ā APPLIQUER AVANT LE STAGE Révisez les temps présentés dans

54

123. José submitted his letter of recommendation for _________ yesterday.(A) I(B) me(C) my(D) mine

124. The company _________ for bankruptcy last year.(A) to file(B) files(C) filed(D) has filed

125. Have you not been paying attention to _________ I have been saying?(A) that(B) which(C) who(D) what

126. The merger _________ any time now.(A) announce(B) will be announced(C) announcing(D) to announce

127. All employees are required to sign a non-disclosure _________.(A) agreement(B) pledge(C) promise(D) vow

128. You ought to know better _________ to cancel a meeting with the boss.(A) then(B) than(C) that(D) which

129. Are you satisfied with your return _________ investment?(A) in(B) at(C) on(D) of

130. You gave _________ a pep talk before the meeting because you were feeling nervous.(A) you(B) your(C) yours(D) yourself

Page 55: BASES ET RVISIONS NCESSAIRES POUR LE TOEIC · 2019. 12. 2. · BASES ET RVISIONS NCESSAIRES POUR LE TOEIC 3 CONSIGNES Ā APPLIQUER AVANT LE STAGE Révisez les temps présentés dans

55TEST BLANC 1.

Part 6Directions: You will be presented with four short texts, each containing four blank spaces. Use one of the four answer choices (A), (B), (C), or (D) to fill in the blank. Select the answer choice that best completes the text.

Questions 131 to 134 refer to the following text:Dear Employees,

Our company will be hosting its annual team-building weekend, stationed at the Rose Park Center downtown, on the 28th, 29th, and 30th of May. Employees will receive food and drink tickets provided by the Gray Bull restaurant adjacent to the building. Our days will start at 10 am and finish at 4 pm; outside of these hours, you are encouraged to take the time to relax or to get to know your __131.__ better.

__132.__ Each group __133.__ the task of completing various activities while applying the appropriate team working skills, strengthening the bond within the company to promote quality work. Some of the activities will include corporate training, trivia, innovation labs, and plenty of games to keep everyone excited and occupied.

We look forward to each of your reservations. Please respond __134.__ the next five days to ensure a spot is held for you.

Thank you,Fatima de OliveiraEvent Planning

131.(A) coworkers(B) clients(C) supervisors(D) partners

132.(A) Dinner will not be provided.(B) Relaxation is an important part of our corporate culture.(C) Once you arrive in the morning, you will be assigned to a group.(D) Previous team-building sessions were highly appreciated by all employees.

133.(A) gives(B) is given(C) will give(D) will be given

134.(A) on(B) upon(C) after(D) within

Page 56: BASES ET RVISIONS NCESSAIRES POUR LE TOEIC · 2019. 12. 2. · BASES ET RVISIONS NCESSAIRES POUR LE TOEIC 3 CONSIGNES Ā APPLIQUER AVANT LE STAGE Révisez les temps présentés dans

56

Questions 135 through 138 refer to the following text:Green tea is one of the most versatile substances I __135.__ across in years of being a nutritionist. Whether in the form of oral supplement, topical medication, __136.__ simply in food and drink, its positive effects are boundless and can be easily incorporated into any schedule.

__137.__ However, it is also popular in candies and various foods, making it a flexible source of antioxidants and a great alternative to other caffeinated drinks! It has been known to do as much as prevent cancers, cure other diseases, and assuage different types of pain. As for your overall health, green tea has also been widely used for weight loss, immune system support, and mental alertness! Taking an oral supplement may increase the __138.__ of the substance, but drinking green tea daily or eating something made with green tea extract will also make your lifestyle healthier.

135.(A) come(B) came(C) have come(D) am coming

136.(A) and(B) or(C) so(D) but

137.(A) Green tea’s properties have been known for centuries.(B) Green tea is most often ingested as a drink.(C) The worldwide production of green tea is measured in millions of tons.(D) Green tea contains less caffeine than coffee does.

138.(A) benefits(B) profits(C) detriments(D) subsidies

Questions 139 through 142 refer to the following text:This week, numerous building safety checks were made in the wake of the building collapse on 8th Street. While no one inside the building in question was hurt, it brought on much __139.__ for the surrounding areas, as the same building company had erected over half of the town.

Upon investigation, it was found that the supporting beams were not built to code. __140.__

The company in question, Apex Builders, took full responsibility for the mistake and offered its services to the building’s owners to __141.__ the property.

The remaining buildings in the area were inspected thoroughly and __142.__ to be in perfect condition, proving the collapsed building to be an exception. Apex Builders’ building teams are to be investigated next, as it was the only variable to have caused the building to be assembled incorrectly.

Page 57: BASES ET RVISIONS NCESSAIRES POUR LE TOEIC · 2019. 12. 2. · BASES ET RVISIONS NCESSAIRES POUR LE TOEIC 3 CONSIGNES Ā APPLIQUER AVANT LE STAGE Révisez les temps présentés dans

57TEST BLANC 1.

139.(A) concern(B) problem(C) argument(D) drawback

140.(A) They were, in fact, obsolete, destroying any chance of the building lasting more than two years past its ope-ning.(B) It revealed quite difficult to prove, though, as the code has not changed over the past 50 years.(C) He also concluded that the materials used during the construction were of substandard quality.(D) The inquiry exposed that the supporting beams did not correspond to the industry’s standards.

141.(A) return(B) retire(C) restitute(D) restore

142.(A) finding(B) found(C) founded(D) founding

Questions 143 to 146 refer to the following text:If you are ready to make money and produce honest, rewarding work, look into a position with our in-home care facility: Project Atlas. We provide paid training to dedicated individuals ready to change the lives of people and their families through hard work and kind company. Though we specialize in elder care, we __143.__ offer a number of solutions for those with long-term disability, worldwide.

As a member of our team, you can help make the world a better place and get paid to learn more about __144.__ . Your job will be to visit the homes of our clients and ensure their quality of life is the highest achievable, ranging from minor medical care to housework and errands they are __145.__ to perform.

We are looking only for those with a minimum of 6 months of health service experience, paid or voluntary, who possess a valid driver’s license and are trained in basic first aid. __146.__

143.(A) in addition(B) moreover(C) also(D) too

144.(A) him(B) her(C) it(D) them

Page 58: BASES ET RVISIONS NCESSAIRES POUR LE TOEIC · 2019. 12. 2. · BASES ET RVISIONS NCESSAIRES POUR LE TOEIC 3 CONSIGNES Ā APPLIQUER AVANT LE STAGE Révisez les temps présentés dans

58

145.(A) cannot(B) unable(C) may not(D) haven’t

146.(A) Help everyone around you!(B) Discover the world through this wonderful experience!(C) Change your own life!(D) Apply today through our website!

Part 7Directions: You will be presented with several short texts or combinations of texts as well as questions about the content of these texts. Select the best answer – (A), (B), (C), or (D) – to the question based on the information obtained from these sources.

Questions 147 and 148 refer to the following email:From: Baltasar IbarrolaTo: Agnes LavalDate/Time: November 20, 11:09 a.m.Subject: installation – error

Hello Agnes,

I am having troubles installing the upgrade for our database management software and thought you might be able to help me. As per your instructions, I launch the .exe file and choose the software language. Then the installer offers to select the folder in which the software should be installed; however, whatever folder I choose, I get the message “An error has occurred,” and the installation stops.

Have you run into this situation before? If yes, how did you solve it? If no, would you have any wild guess as for what I should do? Your help would be greatly appreciated.

Thank you,Baltasar

147. Why is Mr. Ibarrola sending this email?(A) To complain about a problem(B) To ask for advice(C) To request a software upgrade(D) To provide instructions

148. What can be assumed about Ms. Laval?(A) Mr. Ibarrola believes that she knows how to solve the problem.(B) She had delivered the installation guide.(C) Mr. Ibarrola does not really appreciate her.(D) She works in the IT department.

Page 59: BASES ET RVISIONS NCESSAIRES POUR LE TOEIC · 2019. 12. 2. · BASES ET RVISIONS NCESSAIRES POUR LE TOEIC 3 CONSIGNES Ā APPLIQUER AVANT LE STAGE Révisez les temps présentés dans

59TEST BLANC 1.

Questions 149 and 150 refer to the following text message exchange:

Missed call from Abdel, 8:13 a.m.RadekHi Abdel. Sorry, can’t talk now. I am at the doctor’s waiting room. Is it urgent?8:14 a.m.

AbdelActually it is. We are expecting you for security checks, remember?8:15 a.m.

RadekI am not sure I understand. Cyril knew I would be absent this morning. He authorized it himself.8:16 a.m.

AbdelAh… My apologies. I have not been informed. But we are missing a security agent here at the building entrance.8:20 a.m.

RadekI think Cyril mentioned that Bobby could replace me. Isn’t he there?8:22 a.m.

AbdelHe actually walked in 5 minutes ago and explained the situation. I am very sorry to have troubled you.8:28 a.m.

149. In his 8:15 message, what does Abdel mean by saying, “Actually it is”?(A) He confirms that his call was urgent.(B) He knows that Radek is at the doctor’s.(C) He cannot talk either and prefers to text.(D) He shows his irritation with Radek’s absence.

150. Among the people mentioned in the messages, who is most likely the supervisor?(A) Abdel(B) Radek(C) Cyril(D) Bobby

Questions 151 and 152 refer to the following notice:Due to inclement weather, bus and metro services have been cancelled for Monday morning through to Wednesday morning. The weather service has indicated that a monumental winter storm will be hitting the city and will drop a record amount of snow, sleet, and hail on the metro area. Residents are encouraged to stay indoors and only go out in absolute emergency situations. Updates about city services and road closures will be available on the city’s website as well as on the city’s emergency radio station.

151. For how long will the storm last?(A) One morning(B) One day(C) Two days(D) Three days

Page 60: BASES ET RVISIONS NCESSAIRES POUR LE TOEIC · 2019. 12. 2. · BASES ET RVISIONS NCESSAIRES POUR LE TOEIC 3 CONSIGNES Ā APPLIQUER AVANT LE STAGE Révisez les temps présentés dans

60

152. What would be an acceptable situation for residents to go outside during the inclement weather?(A) To clean the roads(B) To visit friends(C) To go to work(D) To go to the hospital

Questions 153 and 154 refer to the following online form:Thank you for using LPV ISP services! Please fill in this questionnaire to help us better serve our customers’ needs.

Your name: HSIU LANGYou have been a customer since: APRIL 25, 2016

For how many hours per day do you use the LPV Internet connection, on all devices?Less than 1 hour 1-3 hours 4-6 hours 7-10 hours Over 10 hours

How satisfied are you with your Internet connection speed?Not satisfied Fairly satisfied Very satisfied

How satisfied are you with your mobile Internet coverage?Not satisfied Fairly satisfied Very satisfied

What can we do to improve our services?MAKE CUSTOMER SERVICE REPRESENTATIVES MORE AVAILABLE ON THE PHONE DURING SHUTDOWNS AND OTHER INCONVENIENCES.

[Submit]

153. What is the purpose of this form?(A) To gather feedback from customers(B) To make customers subscribe to new services(C) To create a database of customers(D) To promote the use of Internet among customers

154. What problem does Hsiu Lang point at?(A) Lack of Internet services(B) Low connection speed(C) Very restricted coverage area(D) Difficulty reaching representatives by phone

Questions 155 through 157 refer to the following email:Hi Guillermo,

It’s nice to hear from you. Sorry it’s been so long since our last email exchange. Things have been a bit crazy at the office. We were awarded a major contract last month that we had been bidding on for quite a while. It was exciting to land the big fish, but now it means that we’ve really had to buckle down and get to work to meet all the deadlines. We were the smallest company bidding for the job, but I think they liked that we

Page 61: BASES ET RVISIONS NCESSAIRES POUR LE TOEIC · 2019. 12. 2. · BASES ET RVISIONS NCESSAIRES POUR LE TOEIC 3 CONSIGNES Ā APPLIQUER AVANT LE STAGE Révisez les temps présentés dans

61TEST BLANC 1.

are family-owned and -run. The first stage of the project is due in three weeks so we’ve been researching and doing an analysis of what was missing from previous marketing campaigns and what we can add to make this new campaign more successful. We’ve really been missing you around the office. We could use your expertise right about now. I’m sure you’re enjoying your new job as stay-at-home dad, but if you ever decide to come back to work, you always have a spot here.

Until next time, my friend!Louis

155. What has recently happened at Louis’s company?(A) They have written new contracts for all the employees.(B) They have gone through a major renovation.(C) They have begun work on a new project.(D) They have just lost a bid on an important account.

156. What sort of work does the company do?(A) Marketing(B) Accounting(C) Construction(D) Event planning

157. What can be inferred about Guillermo?(A) He recently got a job at a new company.(B) He and Louis have been friends for years.(C) He has very little experience.(D) He used to work at the same company as Louis.

Questions 158 through 160 refer to the following memo:Dear Team,

The last few years have been hard. The housing market stagnated, and sales were at an all-time low. Finally, sales have begun to pick up and houses are once again moving on and off the market. Our customers are getting prices that are much closer to what they deserve for their homes, and we are able to grow our list of satisfied clients. It is definitely still a buyer’s market and will stay that way until prices come back up to where they should be. That being said, people are finally feeling more financially stable and in a better position to buy a new home. The banks are starting to give loans once more, and the dream of owning a home is becoming a reality for many people. This company has seen some great success over the past year, and this could not have been done without you, the talented realtors who work days, nights, and weekends to get homes sold. We thank you for all that you do and look forward to an even better year next year.

Management

158. What can be inferred about home prices?(A) They are higher than what the home is worth.(B) They are lower than what the home is worth.(C) They are decreasing slowly over time.(D) They are remaining stagnant and unchanging.

Page 62: BASES ET RVISIONS NCESSAIRES POUR LE TOEIC · 2019. 12. 2. · BASES ET RVISIONS NCESSAIRES POUR LE TOEIC 3 CONSIGNES Ā APPLIQUER AVANT LE STAGE Révisez les temps présentés dans

62

159. What is one reason people are able to buy homes again?(A) The banks have restarted lending money.(B) People are earning more money in their jobs.(C) People have been able to save a lot of money recently.(D) There are fewer homes available on the market.

160. Who is the audience for this memo?(A) Management(B) Homebuyers(C) Realtors(D) Bank owners

Questions 161 through 163 refer to the following announcement:Research Labs is happy to announce that the Summer Internship Program is now accepting applications. This is one of the most highly regarded programs for up-and-coming scientists and researchers in the country. __[1]__ Each year, we receive thousands of applications for the program and select the top 15 candidates to bring in for interviews. With only 5 internship slots available, it is always a challenge to select from the extre-mely qualified candidates. __[2]__ We select interns based on their previous experience, research interests, expected fit into our company culture, and uniqueness of their applications. When applying, be sure to note any academic achievements you have earned as well as any stand-out research you have conducted. __[3]__ Application status can be checked online and will be updated as your application moves through the process. You will receive an email if you have been selected to come in for an interview. Thank you for your interest in our program. We hope to work with you soon. __[4]__

161. For what sort of work will the intern be responsible?(A) Market research(B) Academic research(C) Scientific research(D) Legal research

162. How many interns will the company accept?(A) 5(B) 15(C) 100(D) 1000

163. There are four positions in the text, marked [1], [2], [3], and [4]. In which position does the following phrase best fit?We encourage applications including recommendations from previous research positions as well as from current or previous professors.(A) [1](B) [2](C) [3](D) [4]

Page 63: BASES ET RVISIONS NCESSAIRES POUR LE TOEIC · 2019. 12. 2. · BASES ET RVISIONS NCESSAIRES POUR LE TOEIC 3 CONSIGNES Ā APPLIQUER AVANT LE STAGE Révisez les temps présentés dans

63TEST BLANC 1.

Questions 164 through 167 refer to the following notice:The full list of benefits and the explanations of each benefit can be found in the employee handbook that was given out at your orientation. If you have misplaced your handbook or would like another copy, please request one from HR. As stated on page 70 of your handbook, only full-time employees are entitled to full benefits. A full-time employee is someone who works at least 40 hours a week. All other employees – those who work less than 40 hours a week or “part-time” as well as contractors – are not entitled to full benefits. Employees have the option of negotiating some sort of perks package during the hiring process. In the past, part-time employees have been awarded certain healthcare benefits provided that those benefits were in direct accordance to health risks posed by the job. These benefits exceptions must be cleared through mana-gement and cannot be authorized by Human Resources. Any questions about benefits should be directed to your Human Resources Representative. Please be aware that most concerns are addressed in the employee handbook in the Frequently Asked Questions section. It is recommended you consult this before talking to an HR representative.

164. What is assumed in this notice?(A) All the readers of the notice have attended an orientation session.(B) All HR managers are full-time employees.(C) All part-time employees are entitled to full healthcare benefits.(D) All employees work at least 20 hours a week.

165. Where can a replacement employee handbook be obtained?(A) At employee orientation(B) In the HR department(C) From the management(D) On the company’s website

166. What benefits have been negotiated in the past by part-time employees?(A) Sick leave(B) Vacation days(C) Healthcare(D) Retirement savings

167. Where should employees go first to get questions answered?(A) To the management team(B) To Human Resources(C) To the company’s website(D) To the handbook’s FAQ section

Questions 168 through 171 refer to the following email:Dear Customer,

Your trip is around the corner! You are scheduled to fly from Lisbon to Munich on Thursday, February 19th. Your flight will depart Lisbon at 09:15 and will arrive in Munich at 13:15. You are currently assigned seat 17B, which is a middle seat. __[1]__ If you would like to change your seat, you may do so by visiting our website. You also have the option of upgrading your seat to a window or aisle for as little as 5€. __[2]__ You may also select a premium seat, which allows for twice the legroom as a regular seat, for as little as 20€. If you

Page 64: BASES ET RVISIONS NCESSAIRES POUR LE TOEIC · 2019. 12. 2. · BASES ET RVISIONS NCESSAIRES POUR LE TOEIC 3 CONSIGNES Ā APPLIQUER AVANT LE STAGE Révisez les temps présentés dans

64

are checking luggage, please note that the first bag is complimentary. __[3]__ A second checked bag will cost 25€ and a third an additional 50€. __[4]__ Any items deemed too large to take into the main cabin will be checked at the gate and available at baggage claim in Munich. If you have any questions before your flight, don’t hesitate to email or call one of our reservations experts.

Sincerely,Himmel Airlines

168. What time is the take-off?(A) 9:15(B) 13:15(C) 17:00(D) 20:00

169. How can a passenger change his or her seat?(A) By calling customer service(B) By visiting the company’s website(C) By emailing a reservation’s expert(D) By visiting the ticket counter at the airport

170. How much does it cost to check 3 bags?(A) 75€(B) 50€(C) 25€(D) 20€

171. There are four positions in the text, marked [1], [2], [3], and [4]. In which position does the following phrase best fit?Hand luggage is permitted on board the flight providing that it fits either under the seat in front of you or in the overhead compartment.(A) [1](B) [2](C) [3](D) [4]

Questions 172 through 175 refer to the following online chat:

Melissa Kwambo15:01

Hi everyone. As you know, we need to decide on the dates of moving the office. Note that we’ll need 2 consecutive working days for that. Any constraints?

Yves Kerlouann15:02

Sure. In terms of company-wide events, we have the annual holidays obviously, from December 20th to January 2nd, but that’s the only thing I can think of.

Melissa Kwambo15:02

Any department-specific information?

Ying Kong15:04

We will need two days, as usual, to do the inventory right before the holidays. It’s absolutely impossible for us to pack and unpack at the same time.

Günter Zimmermann15:05

And we’ll have to finalize the shipments to overseas clients before the long absence as well. Customers inside the country will not be too affected though.

Melissa Kwambo15:07

Alright, Günter, how long do you think it will take you?

Page 65: BASES ET RVISIONS NCESSAIRES POUR LE TOEIC · 2019. 12. 2. · BASES ET RVISIONS NCESSAIRES POUR LE TOEIC 3 CONSIGNES Ā APPLIQUER AVANT LE STAGE Révisez les temps présentés dans

65TEST BLANC 1.

Günter Zimmermann15:08

Well, it depends on the orders received by December 15th, of course. Any pro-jected or unconfirmed sale orders as of now?

Sean Fitzpatrick15:09

That’s an on-going process, as usual, but let me check with the team and tell you more precisely in 5 minutes.

Sean Fitzpatrick15:14

Looks like we have a couple of hot prospects: one in Mexico and another in Australia. They can be signed any time now, but of course I can’t guarantee that.

Melissa Kwambo15:16

OK, I see. My conclusion, basically, is that December is not a good time for the moving. Let’s try to schedule it for early January instead. I hope everyone agrees with me.

172. In the first message, what does Ms. Kwambo mean by asking, “Any constraints?”?(A) She welcomes suggestions from the participants for the dates of the moving.(B) She would like to know when the moving cannot take place.(C) She needs to know which events are coming up.(D) She feels already that January is a better month for the moving.

173. When will the inventory take place?(A) December 15th(B) December 18th-19th(C) December 20th-21st(D) December 20th - January 2nd

174. Who is responsible for sending merchandise to clients?(A) Melissa Kwambo(B) Yves Kerlouann(C) Ying Kong(D) Günter Zimmermann

175. What is Sean Fitzpatrick’s role in the company?(A) CEO(B) Logistics Coordinator(C) Sales Director(D) HR Manager

Questions 176 through 180 refer to the following letter and chart:Dear Anwar,

Thank you very much for expressing interest in our company.

As you know, we are one of the leading consulting groups in the world and have offices around the globe. We specialize in a variety of industries including Government and NGOs, Trade, Technology, Manufacturing, and Agriculture. I have attached a chart showing the distribution of our activities.

We hire people who are experts in their field and have them lead teams of financial planners, instructional designers, and analysts. We believe that creativity and quality of product know no experience level, so we create teams that have recent graduates working alongside experienced professionals.

Page 66: BASES ET RVISIONS NCESSAIRES POUR LE TOEIC · 2019. 12. 2. · BASES ET RVISIONS NCESSAIRES POUR LE TOEIC 3 CONSIGNES Ā APPLIQUER AVANT LE STAGE Révisez les temps présentés dans

66

We also believe in the importance of creating a work-life balance so we provide many benefits to our employees to encourage them to work hard and play hard. We have a gym and recreational room in each of our buildings and order in lunch for the entire company on Fridays. This promotes an atmosphere of cooperation instead of competition.

We are always looking for new talent to join our teams in sectors we are in the process of increasing our presence in. If you feel like you would be a good fit here then I highly recommend you send in your CV and cover letter as well as a description of a project you would enjoy working on. I hope to hear from you soon.

Sincerely,

Udo SteinbeckHiring Manager

Number of Projects per Sector

176. What is one occupational field that the company offers?(A) Analyst(B) Accountant(C) Farmer(D) Software Developer

177. How could the consulting teams be described?(A) Large(B) International(C) Diverse(D) Erudite

178. What amenity is not mentioned?(A) A gym(B) A rec room(C) Weekly lunches(D) Health insurance

Page 67: BASES ET RVISIONS NCESSAIRES POUR LE TOEIC · 2019. 12. 2. · BASES ET RVISIONS NCESSAIRES POUR LE TOEIC 3 CONSIGNES Ā APPLIQUER AVANT LE STAGE Révisez les temps présentés dans

67TEST BLANC 1.

179. In which industry is the company likely hiring at the moment?(A) Trade(B) Government/NGOs(C) Technology(D) Manufacturing

180. In which industry does the company have the most projects?(A) Manufacturing(B) Agriculture(C) Technology(D) Agriculture

Questions 181 through 185 refer to the following letters:Dear Sir,

I am writing to inform you that your requested merger with MNDY Technologies has been put on hold pending an investigation into the nature of the merger and the potential effects of that merger on the tech-nologies market. The companies are the two largest producers of technology and a concern has been raised that a merger will create a monopoly of the market, specifically the mobile device application sector. As per government regulations, a monopoly may not exist in any industry where it could have a negative impact for consumers. In this case, the concern would be that prices would be set higher than is fair and that advance-ments in technology would be slowed due to lack of pressure to create new applications. Our team of inves-tigators will visit both your company and MNDY Technologies to determine if, indeed, a monopoly would exist as a result of a merger. This government investigation is expected to take six months but could take less time with full cooperation from both companies.

Sincerely,

Mark JacobsDear Mr. Jacobs,

Thank you for contacting my office with regards to your investigation. We will obviously be more than happy to assist. Our own lawyers have worked closely with the legal team at MNDY to be sure that a merger will not result in a monopoly. We are very concerned with following the law to a tee and do not wish to do anything that might harm our customers. Through their examination, it was determined that given the fact that the companies do research, development, and sales of a wide variety of software for desktop computers, the risk of a monopoly is low. Neither of the two companies has a particularly large mobile device application depart-ment. The fusion of these two departments will, in fact, merely make us a player equal to other competitors in the market – not market leaders. I am, of course, glad to share our research with you and will cooperate fully with your investigation.

Sincerely,

Eugenia AlvarezCEO

Page 68: BASES ET RVISIONS NCESSAIRES POUR LE TOEIC · 2019. 12. 2. · BASES ET RVISIONS NCESSAIRES POUR LE TOEIC 3 CONSIGNES Ā APPLIQUER AVANT LE STAGE Révisez les temps présentés dans

68

181. Why is the merger a potential concern?(A) It would be very expensive.(B) It could create market dominance.(C) It could take a long time.(D) It will be extremely unpopular with consumers.

182. What sort of organization does Mark Jacobs likely work for?(A) Government office(B) Technology firm(C) Consulting organization(D) Mobile application developer

183. Who has already helped Eugenia Alvarez to look into issues with the merger?(A) The companies’ legal teams(B) The companies’ boards of directors(C) The companies’ management teams(D) The companies’ marketing divisions

184. What product does Eugenia Alvarez’s company make?(A) Desktop computers(B) Laptop computers(C) Mobile phones(D) Software

185. What is the most probable reason for Ms. Alvarez to stress that she will fully cooperate with the inves-tigation?(A) She wants to seem helpful in order to avoid a fine.(B) Mr. Jacobs said it would make the investigation go faster.(C) She and Mr. Jacobs used to work together.(D) She is trying to show that Mr. Jacobs is biased.

Questions 186 through 190 refer to the following emails and inventory list:From: Nuno GomesTo: Debra HuntingDate/Time: February 12, 9:15 a.m.Subject: additional equipment

Dear Ms. Hunting,

Thank you again for choosing our convention center for your upcoming conference.

I am writing to inform you that, unfortunately, we have to ask you to bring more equipment than initially specified: namely, extension cords and headsets. While we do have some here, I am not sure they will be suf-ficient for all your teams. I suggest you bring double of the originally planned quantities.

We appreciate you as a customer and count on your cooperation.

Sincerely yours,

Page 69: BASES ET RVISIONS NCESSAIRES POUR LE TOEIC · 2019. 12. 2. · BASES ET RVISIONS NCESSAIRES POUR LE TOEIC 3 CONSIGNES Ā APPLIQUER AVANT LE STAGE Révisez les temps présentés dans

69TEST BLANC 1.

Nuno GomesCenter ManagerGomes Convention Center

From: Debra HuntingTo: Amanda O’ConnellDate/Time: February 12, 10:05 a.m.Subject: need a favor!

Hi Amanda,

Would you do me a favor sometime today or tomorrow? I’ve already left for the conference but got an email this morning that the convention center needs us to bring a few more items. Apparently they don’t have enough of the basic equipment... I am forwarding their email to you.

I know we have packed already, but I need you to go and check the supply closet to see if we have all the new items that have been requested. If we don’t have them, I need you to get them – either from a different department or from a store. If you end up having to buy things, use your company card. Also, remember to bring along the company’s proof of non-profit status. We are tax-exempt so the stores need to remove the tax from the grand total of the purchase. I’ve enclosed the inventory list that I made up before leaving. Please make adjustments to it once you’ve secured the additional items.

Thanks,Debra

Item Quantity Item QuantityPens 150 Extension cords 4Pads of paper 75 Walkie-talkies 6USB drives 75 Flashlights 6Extra registration sheets 10 Headsets 3Giveaway bags 75 Collar microphones 8Promotional flyers 150 Packs of batteries 6

186. Why does Debra need Amanda’s help?(A) She doesn’t understand what needs to be done.(B) She has already left for the conference.(C) She needs permission to change the inventory list.(D) She needs Amanda’s company credit card.

187. Where is the first place Amanda should look for the additional items?(A) The supply closet(B) Another department(C) A store(D) Debra’s desk

188. Why must Amanda bring proof of the company’s non-profit status?(A) To be given some items for free(B) To raise some money for the conference(C) To be able to cut to the front of the line when paying(D) To avoid having to pay sales tax on the purchase

Page 70: BASES ET RVISIONS NCESSAIRES POUR LE TOEIC · 2019. 12. 2. · BASES ET RVISIONS NCESSAIRES POUR LE TOEIC 3 CONSIGNES Ā APPLIQUER AVANT LE STAGE Révisez les temps présentés dans

70

189. How many extension cords will be listed on the updated inventory?(A) 4(B) 6(C) 8(D) 12

190. What is the likely number of the conference participants?(A) 8(B) 10(C) 75(D) 150

Questions 191 through 195 refer to the following email, schedule, and text message exchange:

May 20, 3:15 p.m.

Dear Marianne,

I’m so excited that you decided to accept the position here in the London office. There is a great team out here, and I think you’ll fit in well. It’s been a little hard adjusting to the weather after living in Los Angeles for my entire life, but I’m sure you won’t have a problem with that having grown up in Seattle and Vancouver. The weather is pretty similar, really – overcast and drizzly with periods of sun a few times a day. Public trans-portation here is top-notch, and I recommend taking the train from the airport when you arrive. I’ve attached a train schedule for you so you can see which one works given your 7:45 a.m. arrival. Remember you’ll need to clear immigration and customs, so I recommend expecting to get to the train platform about 2 hours after you land. Our offices are a few blocks away from the train station, so I can meet you there, and we can walk back to the office together. Have a safe flight!

Sanjay

Departure – Airport Arrival – Central London8:30 a.m. 8:50 a.m.9:00 a.m. 9:20 a.m.9:30 a.m. 9:50 a.m.10:00 a.m. 10:20 a.m.10:30 a.m. 10:50 a.m.

MarianneHi Sanjay,So sorry, there is a huge line at the passport control…May 23, 9:20 a.m.

SanjayWelcome to London! :) Does it look very bad? Do you think you’ll make it to the first train, as planned?May 23, 9:22 a.m.

Page 71: BASES ET RVISIONS NCESSAIRES POUR LE TOEIC · 2019. 12. 2. · BASES ET RVISIONS NCESSAIRES POUR LE TOEIC 3 CONSIGNES Ā APPLIQUER AVANT LE STAGE Révisez les temps présentés dans

71TEST BLANC 1.

MarianneI honestly doubt that. It doesn’t seem to be moving at all.May 23, 9:30 a.m.

SanjayNo problem. Just text me again when you are on the train, and I’ll step out of the office then.May 23, 9:31 a.m.

MarianneOk, I am through with the passport. Customs seems fluid. So, it looks like I will be only half an hour late.May 23, 9:45 a.m.

SanjayGreat. See you soon!May 23, 9:50 a.m.

191. Where did Sanjay grow up?(A) London(B) Los Angeles(C) Seattle(D) Vancouver

192. What is the weather typically like in Seattle?(A) Sunny and dry(B) Overcast and dry(C) Sunny and rainy(D) Overcast and rainy

193. Which train to London was Marianne planning to take?(A) The 8:30 a.m. train(B) The 9:00 a.m. train(C) The 9:30 a.m. train(D) The 10:00 a.m. train194. Where will Sanjay meet Marianne?(A) At the airport(B) At the train station(C) In front of the office(D) At the office

195. At what time will Marianne arrive in Central London?(A) 10:00 a.m.(B) 10:20 a.m.(C) 10:30 a.m.(D) 10:50 a.m.

Page 72: BASES ET RVISIONS NCESSAIRES POUR LE TOEIC · 2019. 12. 2. · BASES ET RVISIONS NCESSAIRES POUR LE TOEIC 3 CONSIGNES Ā APPLIQUER AVANT LE STAGE Révisez les temps présentés dans

72

Questions 196 through 200 refer to the following newsletter, press release, and email:YBG Newsletter

Dear Media Partners,

Yesterday afternoon, YBG Bank announced that it has hired a new Chief Risk Officer. The former CRO left early last month after announcing that he would be stepping down for health reasons. The position has been occupied temporarily by Natasha Marinkova, a managing director in the credit risk department, while the hunt for a new CRO took place. In the end, the bank decided to promote internally and officially gave Marinkova the job. The bank has released the official statement, which you fill find attached.

You are welcome to share this announcement through your media channels.

Public Relations DepartmentYBG Bank

YBG BankOfficial informationFor internal and external distribution

We are pleased to announce that Natasha Marinkova has accepted our offer to become the new Chief Risk Officer. Natasha has worked for YBG for six years and has risen through the ranks by demonstrating an innate understanding of how to manage risk in today’s volatile markets. She has been an instrumental part of the credit risk team and has proven herself to be a natural leader as well as an excellent team player. We are excited to have Natasha play such a vital role at YBG and look forward to all that she has to contribute to our future growth and success.

The Management

From: Giulia AuricchioTo: [email protected]/Time: October 8, 16:20Subject: request for an interview

Dear Sirs,

As the manager of the Women in Management section at Sapphire Business Magazine, (Sapphire Media Holdings), I have been thrilled to learn that Ms. Marinkova has been appointed the CRO of YBG Bank. Please extend my congratulations to her and to the bank’s management for this excellent choice. Moreover, this promotion is a valuable example of women taking leadership positions when you consider that at only 40 years old, Ms. Marinkova will be the youngest CRO ever employed by a Swiss bank.

In this regard, I would be honored if Ms. Marinkova could devote some of her time to an interview that we will gladly publish in Women in Management. I am ready to meet her at her earliest convenience.

Best regards,

Page 73: BASES ET RVISIONS NCESSAIRES POUR LE TOEIC · 2019. 12. 2. · BASES ET RVISIONS NCESSAIRES POUR LE TOEIC 3 CONSIGNES Ā APPLIQUER AVANT LE STAGE Révisez les temps présentés dans

73TEST BLANC 1.

Giulia AuricchioEditorSapphire Business Magazine

196. What was Natasha Marinkova’s previous job?(A) CRO(B) Public Relations Director(C) Managing Director(D) Editor

197. What can be inferred about Ms. Marinkova?(A) She has been promoted several times before.(B) She doesn’t work well in group settings.(C) She dislikes the media.(D) She is a subscriber of Sapphire Business Magazine.

198. What is one of the qualities the management attributes to Ms. Marinkova?(A) A deep understanding of management(B) An unwavering respect for business ethics(C) A strong sense of morality(D) A natural talent for understanding risk

199. How old was Ms. Marinkova when she began working for YBG bank?(A) 34(B) 36(C) 40(D) 46

200. What is the main purpose of Ms. Auricchio’s email?(A) To introduce herself and her magazine(B) To promote gender equality in management(C) To congratulate Natasha Marinkova on her promotion(D) To obtain an interview with Natasha Marinkova

This is the end of the test. If there is time remaining, you may go back and review your answers for Parts 5, 6, and 7.

Page 74: BASES ET RVISIONS NCESSAIRES POUR LE TOEIC · 2019. 12. 2. · BASES ET RVISIONS NCESSAIRES POUR LE TOEIC 3 CONSIGNES Ā APPLIQUER AVANT LE STAGE Révisez les temps présentés dans

74

TOEIC L&R PRACTICE TEST 1

file TOEICL&R.PracticeTest1.Listening.mp3

Listening SectionIn the Listening section of the test, you are asked to demonstrate how well you understand spoken English. It consists of four parts, with special directions in each part. Remember, you are not allowed to take notes. This section will last approximately 45 minutes.

Part 1Directions: In Part 1 of the test, you will be shown 6 photographs and hear four statements about each pho-tograph. The statements will be marked (A), (B), (C), and (D). All statements will be spoken only once and not written down. For each photograph, choose the statement that accurately describes what can be seen in the photograph.

Look at a sample photograph.

Listen to the four statements.

(A) The ship is very modern.(B) Passengers are boarding the ship.(C) Palm-trees are planted along the waterfront.(D) People are having a party on a boat.

Statement (C), “Palm-trees are planted along the waterfront,” is the most accurate description of what can be seen in the photograph. Therefore, you should mark your answer choice (C).

This part will now start with Question 1. Select your answers as you listen.

Number 1.(A) The business lunch has just finished.(B) They are discussing a graph.(C) The dessert has been served.(D) They are enjoying a drink.

Page 75: BASES ET RVISIONS NCESSAIRES POUR LE TOEIC · 2019. 12. 2. · BASES ET RVISIONS NCESSAIRES POUR LE TOEIC 3 CONSIGNES Ā APPLIQUER AVANT LE STAGE Révisez les temps présentés dans

75TEST BLANC 1.

Number 2.(A) All parking spaces are taken.(B) A red car is driving through the parking lot.(C) The city is on the seaside.(D) Tourists are walking towards the building.

Number 3.(A) The man is typing on the keyboard.(B) The papers are under the computer.(C) The screen has been turned off.(D) The pen is being used.

Number 4.(A) They look concerned.(B) They are arguing about a project.(C) They are using the computer.(D) They are taking a break.

Number 5.(A) Cruise ships are floating on the water.(B) People are watching the marina.(C) The weather is cloudy.(D) A skyscraper is being built.

Number 6.(A) They are talking on the phone.(B) A man is pointing at the computer.(C) Women are taking notes.(D) The meeting is adjourned.

Part 2Directions: In Part 2 of the test, you will listen to 25 questions, each followed by three potential responses. The responses will be marked (A), (B), and (C). All questions and responses will be spoken only once and not written down. For each question, choose the response that accurately answers it.

This part will now start with Question 7. Select your answers as you listen.

Number 7. How much does parking cost?(A) Costs keep rising, and quality keeps falling.(B) Parking is available in the lot next door.(C) It depends on how long you leave your car.

Page 76: BASES ET RVISIONS NCESSAIRES POUR LE TOEIC · 2019. 12. 2. · BASES ET RVISIONS NCESSAIRES POUR LE TOEIC 3 CONSIGNES Ā APPLIQUER AVANT LE STAGE Révisez les temps présentés dans

76

Number 8. Do you know where the blue folder is?(A) I use the green folder regularly.(B) I left the folder with your secretary.(C) I saw her at the welcome desk 5 minutes ago.

Number 9. Where am I supposed to sign?(A) You’re supposed to get a lawyer to read it.(B) I usually sign papers in my office.(C) On the last page on the signature line.

Number 10. Who is doing the printing?(A) The same company we usually use.(B) The documents have already been sent to the printer.(C) Isn’t Cecilia in charge of ordering items?

Number 11. When will the reports arrive?(A) The documents are due on Monday.(B) They should get here on Tuesday.(C) The flight arrived an hour ago.

Number 12. Do you have the file handy?(A) It’s right here in my bag.(B) My hands are tied, I’m afraid.(C) The file was about the Yuzuki case.

Number 13. Is the website up and running yet?(A) Your website is quite beautifully designed.(B) I went running yesterday after work.(C) It should be up in the next hour.

Number 14. Was your flight on time?(A) It was early, actually.(B) It was supposed to land at 9 p.m.(C) It was quite comfortable.

Number 15. What’s the weather supposed to be like?(A) I’m not sure whether he is supposed to come or not.(B) It’s going to drizzle most of the day.(C) I like it when it’s warm and breezy.

Number 16. When do we get paid?(A) Usually on the 1st and 15th of the month.(B) I pay my bills on time each month.(C) By wire transfer to our account.

Number 17. How are you feeling this morning?(A) Mornings I prefer to work from home.(B) I had a feeling that this was a bad idea.(C) I’m on the mend but still quite sore.

Page 77: BASES ET RVISIONS NCESSAIRES POUR LE TOEIC · 2019. 12. 2. · BASES ET RVISIONS NCESSAIRES POUR LE TOEIC 3 CONSIGNES Ā APPLIQUER AVANT LE STAGE Révisez les temps présentés dans

77TEST BLANC 1.

Number 18. Who has applied for the position?(A) The position is available to management-level only.(B) I know that Carlos Martinez has submitted his resume.(C) I applied for a new passport last week.

Number 19. What was the name of the street?(A) The same as my last name: Davis!(B) It’s the next street over.(C) I can never remember people’s names.

Number 20. Do you hear that?(A) That is my new book.(B) I heard it on the news yesterday.(C) It is just the copier.

Number 21. Where did you say you were from?(A) My parents were from Ohio.(B) I grew up in New Zealand.(C) I came directly from the office.

Number 22. When is the conference?(A) Somewhere in Europe, I think.(B) In the fall sometime.(C) To discuss the new industry trends.

Number 23. Why are you late?(A) My car wouldn’t start this morning.(B) I really can’t stand being late.(C) Why not arrive early?

Number 24. How did this end up on my desk?(A) I ended up making a reservation.(B) Your office is very convenient.(C) Aren’t you in charge of the project?

Number 25. Are you responsible for arranging transportation?(A) You can get there using the bus or train.(B) I was, but then Salvatore took over.(C) No, he was just arranging the furniture.

Number 26. When are taxes due this year?(A) This year will be an expensive one.(B) Taxes are such a burden.(C) As usual: in April and September.

Number 27. What do you think of this color for my new office chair?(A) It’s a bit flashy, don’t you think?(B) Your chair looks good there.(C) I love red, purple, and blue.

Page 78: BASES ET RVISIONS NCESSAIRES POUR LE TOEIC · 2019. 12. 2. · BASES ET RVISIONS NCESSAIRES POUR LE TOEIC 3 CONSIGNES Ā APPLIQUER AVANT LE STAGE Révisez les temps présentés dans

78

Number 28. Have you already settled the bill?(A) This was an expensive visit.(B) I’ve just given him my card.(C) I’m settled in to my new place.

Number 29. When does your new job start?(A) Hiring started last week.(B) The job is in a factory.(C) My first day is next Monday.

Number 30. Whom did you send the email to?(A) The email went out this morning.(B) All the team leaders and their assistants.(C) Boris emailed Jane a few minutes ago.

Number 31. What department are you in charge of?(A) I run the Development office.(B) I found it in the Menswear department.(C) They charged me extra for rush shipping.

Part 3Directions: In Part 3 of the test, you will listen to 13 conversations. Every conversation will be followed by three questions about it. Each question will have four answer choices marked (A), (B), (C), and (D). Each conversa-tion will be spoken only once and not written out. For each question, choose the answer choice that accurately answers it.

Questions 32 through 34 refer to the following conversation:MAN: Have you made those trades yet? If not, today’s a good day to do so.WOMAN: It’s on my calendar for tomorrow, but I notice the market is down today.MAN: Yes. Only some of the mutual funds are trading up, but in general the individual stocks are down so it’s a good time to buy. When you’ve completed the transactions, please email me the new figures for the accounts.

Number 32. When was the woman planning to do the trades?Number 33. Why is today a good day for making trades?Number 34. What should the woman do after completing the trades?

Questions 35 through 37 refer to the following conversation:WOMAN: It’s so good to see you! Welcome to Canada! I’m glad you finally made it. What took you so long?MAN: I got held up at customs. The security dog smelled something in my luggage, and they wanted to check it out. Unfortunately, they subsequently confiscated it.WOMAN: Oh no! What was it?MAN: Some delicious salami that I had brought you from Denmark as a hostess gift.

Number 35. Where does this conversation likely take place?

Page 79: BASES ET RVISIONS NCESSAIRES POUR LE TOEIC · 2019. 12. 2. · BASES ET RVISIONS NCESSAIRES POUR LE TOEIC 3 CONSIGNES Ā APPLIQUER AVANT LE STAGE Révisez les temps présentés dans

79TEST BLANC 1.

Number 36. What has the man inadvertently done?Number 37. What can be inferred about the man and woman?

Questions 38 through 40 refer to the following conversation:MAN: I can’t understand this bookkeeping at all. I mean, I get that we are in the red, but I can’t see how we got there.WOMAN: Do you want me to have Linda come and explain?MAN: I think that would be wise. I really need a clear breakdown of our income and expenses so that I can see where we can cut back, to start making a profit again.

Number 38. What problem does the man have?Number 39. What can be inferred about the company?Number 40. What does the man think would be a good idea?

Questions 41 through 43 refer to the following conversation:WOMAN: Good morning, Mr. Larsson. This is Sylvia calling from Dr. Monroe’s office. How are you?MAN: I’m fine, thanks. What can I do for you?WOMAN: I was just reviewing your files, and it seems we are missing some paperwork for you – specifically your emergency contact information. I’d like to fax the form to you and have you fill it out to complete your records.MAN: Please do. I can then bring it in at my upcoming appointment.

Number 41. Where does this conversation take place?Number 42. What paperwork does the man need to complete?Number 43. What can be inferred about the man?

Questions 44 through 46 refer to the following conversation:WOMAN: Have you seen the headlines this morning?MAN: No. What’s going on?WOMAN: The government has accused ACNI Bank of fraud. Apparently the fund managers have been siphoning off profits for years. A number of shareholders are getting together to sue the bank.MAN: Wow. I bet there will be a lot of people out of the job by this afternoon.

Number 44. What crime has ACNI Bank been accused of?Number 45. What has been the shareholders’ response to this accusation?Number 46. What prediction does the man make?

Questions 47 through 49 refer to the following conversation:WOMAN: I’m not looking forward to today. I have to sit down with Pedro and let him go.MAN: Oh no. I thought he was doing better?WOMAN: No. He’s such a nice guy, but he is not cut out for sales. He’s just not aggressive enough. I’ve tried doing extra training with him, but it hasn’t worked.MAN: That’s a shame. He’s just starting out though. He’ll be ok.

Number 47. What task will the woman complete today?

Page 80: BASES ET RVISIONS NCESSAIRES POUR LE TOEIC · 2019. 12. 2. · BASES ET RVISIONS NCESSAIRES POUR LE TOEIC 3 CONSIGNES Ā APPLIQUER AVANT LE STAGE Révisez les temps présentés dans

80

Number 48. What is one of Pedro’s characteristics?Number 49. What can be inferred about Pedro?

Questions 50 through 52 refer to the following conversation:MAN: I was hoping to get a quote from you about designing my website.WOMAN: Sure. We usually charge $40 per hour, but the total cost depends on the amount of work needed. Do you have a website you’d like overhauled, or do you need a brand new one?MAN: The website I have is very outdated so I don’t think it’s worth overhauling. If an entirely new site is within my budget, then I’d like that. I surely can’t spend more than $4,000 on the project though.

Number 50. What is the woman’s specialty?Number 51. What can be inferred about the man?Number 52. How many hours of work can the man afford?

Questions 53 through 55 refer to the following conversation:WOMAN: That’s about it on my end. Do you have any questions?MAN: I think you have covered everything. When should I expect to hear back from you?WOMAN: You were our last candidate to interview so now we just need to make our decision which of the five of you is the best fit. To be honest, I’m fully expecting to recommend you for the position. Anticipate a call from me in the next day or two.

Number 53. What is the context of this conversation?Number 54. How many people had the woman spoken to before her conversation with the man?Number 55. When will the two people speak again?

Questions 56 through 58 refer to the following conversation:MAN: Excuse me. Could you tell me which platform the train to Amsterdam is leaving from? I’m having trouble reading the Departures board.WOMAN: Good afternoon. Are you on the local or the express? They’re both in the station at the moment.MAN: The local I think. It’s leaving at 1:15 p.m. and arriving at 4:15 p.m.WOMAN: No. That’s the express. It is leaving from platform 30.

Number 56. Why does the man ask for assistance?Number 57. How long will the man’s train ride be?Number 58. What platform should the man go to?

Questions 59 through 61 refer to the following conversation:WOMAN 1: Good evening, my name is Emily. I will be your server tonight. Have your choices been made?WOMAN 2: I believe so, yes. What about you, Mathew?MAN: I only wanted to ask… How spicy is your fish soup of the day?WOMAN 1: Actually, it isn’t spicy at all. It definitely features several aromatic herbs, but nothing that would burn your palate.MAN: Perfect. I will have the soup then, and a medium-rare steak with salad as the main course.WOMAN 1: Definitely. And for you, Ma’am?WOMAN 2: I will go for the octopus sashimi as a starter and then the roast duck in orange sauce. Thank you.

Number 59. What meal are the clients about to have?

Page 81: BASES ET RVISIONS NCESSAIRES POUR LE TOEIC · 2019. 12. 2. · BASES ET RVISIONS NCESSAIRES POUR LE TOEIC 3 CONSIGNES Ā APPLIQUER AVANT LE STAGE Révisez les temps présentés dans

81TEST BLANC 1.

Number 60. Which of the following is probable regarding the soup?Number 61. What main course will the woman have?

Questions 62 through 64 refer to the following conversation:MAN: Now, I want to talk about where the money has been going. Kathy, could you please pull those graphs up on the screen?WOMAN: Sure. Are they in the Donaldson Foundation folder or on the desktop?MAN: The desktop. Ah, there we go. The majority of the funds is being spent on advertising. This is fine except that it means that not enough of the donations are actually going towards projects. This needs to change.WOMAN: Should I switch to the proposed expense structure?MAN: Yes, please, that’s the next slide. As you can see, we would like to keep advertising under 10% of our total expenses.

Number 62. What is the setting of this conversation?Number 63. Where are the graphs the woman is looking for?Number 64. Look at the graphic. What is the proposed share of expenses on projects?

Questions 65 through 67 refer to the following conversation:MAN: Hi there. Is this FDBG limited? I have a package for Mr. Andreas Yannatos.WOMAN: Good morning and welcome. Certainly; where should I sign?MAN: I am afraid this is hand delivery, so I have to see him myself.WOMAN: No problem. He works in the Marketing Department, and you are welcome to take the elevator, on your left.

Number 65. What are the man’s and the woman’s jobs?Number 66. Which of the following is true about the package?Number 67. Look at the graphic. To which floor does the man need to go?

Questions 68 through 70 refer to the following conversation:MAN: I finally have our tickets for tomorrow’s show. I am so excited!WOMAN: Perfect! Me too, we’re so lucky. How much do I owe you?MAN: Here is your ticket – but remember that as a patron I get 20% off face value.WOMAN: Even better! I will write you a check right away. Thank you so much.

Number 68. What can be assumed about the show?Number 69. Look at the graphic. How much will the woman pay the man?Number 70. When will the woman pay the man?

Page 82: BASES ET RVISIONS NCESSAIRES POUR LE TOEIC · 2019. 12. 2. · BASES ET RVISIONS NCESSAIRES POUR LE TOEIC 3 CONSIGNES Ā APPLIQUER AVANT LE STAGE Révisez les temps présentés dans

82

Part 4Directions: In Part 4 of the test, you will listen to 10 talks. Every talk will be followed by three questions about it. Each question will have four answer choices marked (A), (B), (C), and (D). Each talk will be spoken only once and not written out. For each question, choose the answer choice that accurately answers it.

Questions 71 through 73 refer to the following talk:WOMAN: I get asked all the time about saving for retirement. People want to know the best way to safely grow their money so that they don’t have to worry once they’ve stopped working. I think this is a common concern today. Interest rates on savings accounts are at an all-time low. The housing market is still recovering from the crash of 2008. People consider sticking their cash under their mattresses and hoping for the best. This clearly isn’t a very good option. What I always recommend is that people sit down together and map out a detailed plan of what they want to do during their retirement and exactly how much this is going to cost. Once they have determined that, they can look at the best options to grow their wealth.

Number 71. According to the talk, what is a common concern today?Number 72. What is the stated problem with the savings accounts?Number 73. What recommendation does the speaker give to the audience?

Questions 74 through 76 refer to the following announcement:MAN: Welcome aboard! This is your captain speaking. We’re excited to have everyone with us for our 5-day and 4-night cruise around the Greek Islands. As we pull out of port, we invite everyone up to the main deck to wave goodbye to family and friends. We are serving complimentary cocktails and snacks to get you in the spirit. We’ll also have some performers by the main pool and some dance instructors teaching traditional Greek dances. The buffet dinner will be served in the forward dining room beginning at 5 p.m. and ending at 10 p.m. Late-night dining is available in any of the bars, as well as in the casino and lounge area.

Number 74. Where is this announcement given?Number 75. According to the announcement, what is happening by the main pool?Number 76. Where can people find food at 11:05 p.m.?

Questions 77 through 79 refer to the following talk:WOMAN: Bobby, I’m glad I found you. I’ve been working on the new ad campaign but need to run it by someone. Let me know what you think. Here it is: “Winter can be a rough time for hands. The dry air leaves skin painfully cracked and split. Skincare experts will suggest all kinds of lotions and creams to try to fix the problem, but none of these works because they don’t address one of the main reasons why this is a problem: repeated hand washing. Every time you wash your hands, you dry them out more. Since you can’t (and shouldn’t) stop washing your hands, you should start using moisturizing hand soap. This kind of soap condi-tions while it cleans, leaving hands moisturized and protecting them from further cracking and splitting.” That’s it. What do you think?

Number 77. What product is being discussed?Number 78. What problem does this product address?Number 79. What department does the speaker most likely work in?

Page 83: BASES ET RVISIONS NCESSAIRES POUR LE TOEIC · 2019. 12. 2. · BASES ET RVISIONS NCESSAIRES POUR LE TOEIC 3 CONSIGNES Ā APPLIQUER AVANT LE STAGE Révisez les temps présentés dans

83TEST BLANC 1.

Questions 80 through 82 refer to the following orientation speech:MAN: Please take your employee handbook out of the orientation packet you picked up when you came in. This is the thick document with a gray cover. This is your resource for everything office-related. If you need to know whom to call to get your computer repaired, the number is in there. If you need to know how to get a new ID card, the information is in there. If you need to know how to setup your voicemail, the steps are in there. We have spent years refining this book to make it as comprehensive as it possibly can be. This saves everyone time since you are able to troubleshoot your own problems and don’t have to set up a meeting with someone every time you have a question.

Number 80. What does the employee handbook look like?Number 81. Which of the following topics does the handbook most likely cover?Number 82. What is a stated benefit of the handbook?

Questions 83 through 85 refer to the following talk:WOMAN: Great question! One of the best things about our location is that it is accessible by many different modes of transportation. There are three buses that drop off right at the front door. In addition, the Blue and the Green metro lines stop just four blocks way. There is also a fantastic bike trail that passes in front of the building. It runs from downtown all the way out to the western edge of the city. Lots of employees bike to work using the trail. I’ll also mention that many others use their lunch breaks to go for a walk or run along it. It’s a great resource to have actually. I veered off topic a bit, but in answer to your question – yes, the office is easy to get to without a car.

Number 83. How many metro lines stop not far from the building?Number 84. What is an added benefit of the bike trail?Number 85. What question is the speaker most probably responding to?

Questions 86 through 88 refer to the following message:MAN: Thank you for calling McAllister School of Management. Where managers learn to become leaders. We are pleased to have the world-famous Daniela Fernandez teaching our Global Business Strategy course this semester. We are currently accepting students for the fall semester. Course listings and prices can be found on our website under the “Registration” tab. To continue in English, please press 7. To continue in Spanish, please press 9. If you know your party’s extension, you may dial it at any time. To hear a staff and professor directory, please press 1. For school locations, please press 2. For the scholarship office, please press 3. To request information about our weekend Accounting Seminar, please press 4. To be connected with an operator, please press 0. Thank you, and have a great day.

Number 86. What can be inferred about Daniela Fernandez?Number 87. What should a person do to find a professor’s extension?Number 88. What can be inferred about the school?

Questions 89 through 91 refer to the following talk:WOMAN: Thank you for coming in today. I’ve spoken to both of you about the situation and feel like I understand what is most troubling for you as a team. I have some suggestions that I’d like to offer to see if perhaps they help create a more comfortable working environment. Anita, one of the concerns you’ve raised is that you are not sure when projects are due or what the scope of the project is since Jamie often gives you only small parts to work on. I think that this can be improved if Jamie sets up a shared online calendar and

Page 84: BASES ET RVISIONS NCESSAIRES POUR LE TOEIC · 2019. 12. 2. · BASES ET RVISIONS NCESSAIRES POUR LE TOEIC 3 CONSIGNES Ā APPLIQUER AVANT LE STAGE Révisez les temps présentés dans

84

database to keep track of the different parts of the project. This way both of you can see what elements have been completed, what is left to do, and when deadlines are.

Number 89. Where does this talk most probably take place?Number 90. What concern has Anita raised?Number 91. What suggestion does the speaker make?

Questions 92 through 94 refer to the following speech:MAN: Thank you all for coming tonight. I’m just so touched by the outpouring of support and well-wishes I’ve received since announcing my retirement. I feel so fortunate to have found an organization so early on in my career that allowed me to develop and grow and become the person I am today. I have had so many wonderful experiences over the years, and I know that it’s because of the incredible people I’ve had the honor to work with. I’d especially like to thank my assistant, Ioana, who has been by my side for the past decade making sure that I showed up where I needed to be when I needed to be. You are a treasure, and I will miss working with you every day.

Number 92. What is the context of this speech?Number 93. According to the speech, what has contributed to the speaker’s experiences?Number 94. Who is Ioana?

Questions 95 through 97 refer to the following voice message:WOMAN: Hello Ms. Heikkinen. I’m calling from the embassy. We received your visa application but are unable to process it. While you included a photograph, unfortunately, it is the wrong size. Please review the visa application guide we have provided for the correct size. In addition, though you sent in the application fee, you neglected to include the rush-processing fee that is required for visas requested less than one week before travel. You indicated that the purpose of your trip is business, which means that we will require a notice from your company stating the nature of your business as well as a list of the names and addresses of the clients you will meet while in the country. Please collect these items and send them to our offices as quickly as possible.

Number 95. Look at the graphic. Most likely, what was the size of the photograph that Ms. Heikkinen sub-mitted?Number 96. When will Ms. Heikkinen most likely travel?Number 97. What is the purpose of Ms. Heikkinen’s trip?

Questions 98 through 100 refer to the following voice message:WOMAN: Hello Sven, I need you to update the prices on our website as soon as possible. We have just had a meeting with the Sales Team and decided to lower the prices on all sunglasses by 10% for the next 3 weeks. Note that this is neither a special offer nor a discount – we simply follow the competition. I will ask Olga to print the new price tags for both of our boutiques as well… Oh, I almost forgot. The reduction does not apply to the Butterfly models of glasses – their price remains as is. Call me if you have any questions. Oh, and did I mention it was urgent?

Number 98. What is Sven’s job?Number 99. How many boutiques does the company have?Number 100. Look at the graphic. What will be the new price of the model Butterfly 3N?

This is the end of the Listening section.

Page 85: BASES ET RVISIONS NCESSAIRES POUR LE TOEIC · 2019. 12. 2. · BASES ET RVISIONS NCESSAIRES POUR LE TOEIC 3 CONSIGNES Ā APPLIQUER AVANT LE STAGE Révisez les temps présentés dans

85TEST BLANC 2

Ce livre est spécialement dédié au nouveau TOEIC.

Vous retrouverez une partie d’entraînement au TOEIC dans le livre.

Pour la partie listening du test d’entrainement, vous pourrez retrouver la piste audio avec les différents liens proposes en bas.

QR CODE de téléchargement du fichier audio :

Vidéos intéractives des Toeic d’entraînement :

https://www.youtube.com/watch?v=Pn2i7U3RaKQ&t=1184s

Retrouvez toutes les informations et les tests sur Toeicator.com

Page 86: BASES ET RVISIONS NCESSAIRES POUR LE TOEIC · 2019. 12. 2. · BASES ET RVISIONS NCESSAIRES POUR LE TOEIC 3 CONSIGNES Ā APPLIQUER AVANT LE STAGE Révisez les temps présentés dans

86

Page 87: BASES ET RVISIONS NCESSAIRES POUR LE TOEIC · 2019. 12. 2. · BASES ET RVISIONS NCESSAIRES POUR LE TOEIC 3 CONSIGNES Ā APPLIQUER AVANT LE STAGE Révisez les temps présentés dans

87TEST BLANC 2

TOEIC L&R PRACTICE TEST 2

Play the file TOEICL&R.PracticeTest2.Listening.mp3

Listening SectionIn the Listening section of the test, you are asked to demonstrate how well you understand spoken English. It consists of four parts, with special directions in each part. Remember, you are not allowed to take notes. This section will last approximately 45 minutes.

Part 1Directions: In Part 1 of the test, you will be shown 6 photographs and hear four statements about each pho-tograph. The statements will be marked (A), (B), (C), and (D). All statements will be spoken only once and not written down. For each photograph, choose the statement that accurately describes what can be seen in the photograph.

Look at a sample photograph.

Listen to the four statements.

Statement (C), “Palm-trees are planted along the waterfront,” is the most accurate description of what can be seen in the photograph. Therefore, you should mark your answer choice (C).

This part will now start with Question 1. Select your answers as you listen.

Page 88: BASES ET RVISIONS NCESSAIRES POUR LE TOEIC · 2019. 12. 2. · BASES ET RVISIONS NCESSAIRES POUR LE TOEIC 3 CONSIGNES Ā APPLIQUER AVANT LE STAGE Révisez les temps présentés dans

88

1.

2.

3.

4.

5.

6.

Page 89: BASES ET RVISIONS NCESSAIRES POUR LE TOEIC · 2019. 12. 2. · BASES ET RVISIONS NCESSAIRES POUR LE TOEIC 3 CONSIGNES Ā APPLIQUER AVANT LE STAGE Révisez les temps présentés dans

89TEST BLANC 2

Part 2Directions: In Part 2 of the test, you will listen to 25 questions, each followed by three potential responses. The responses will be marked (A), (B), and (C). All questions and responses will be spoken only once and not written down. For each question, choose the response that accurately answers it.

This part will now start with Question 7. Select your answers as you listen.

7. Mark your answer on the answer sheet.8. Mark your answer on the answer sheet.9. Mark your answer on the answer sheet.10. Mark your answer on the answer sheet.11. Mark your answer on the answer sheet.12. Mark your answer on the answer sheet.13. Mark your answer on the answer sheet.14. Mark your answer on the answer sheet.15. Mark your answer on the answer sheet.16. Mark your answer on the answer sheet.17. Mark your answer on the answer sheet.18. Mark your answer on the answer sheet.19. Mark your answer on the answer sheet.20. Mark your answer on the answer sheet.21. Mark your answer on the answer sheet.22. Mark your answer on the answer sheet.23. Mark your answer on the answer sheet.24. Mark your answer on the answer sheet.25. Mark your answer on the answer sheet.26. Mark your answer on the answer sheet.27. Mark your answer on the answer sheet.28. Mark your answer on the answer sheet.29. Mark your answer on the answer sheet.30. Mark your answer on the answer sheet.31. Mark your answer on the answer sheet.32. Mark your answer on the answer sheet.33. Mark your answer on the answer sheet.34. Mark your answer on the answer sheet.35. Mark your answer on the answer sheet.36. Mark your answer on the answer sheet.37. Mark your answer on the answer sheet.38. Mark your answer on the answer sheet.39. Mark your answer on the answer sheet.40. Mark your answer on the answer sheet.

Page 90: BASES ET RVISIONS NCESSAIRES POUR LE TOEIC · 2019. 12. 2. · BASES ET RVISIONS NCESSAIRES POUR LE TOEIC 3 CONSIGNES Ā APPLIQUER AVANT LE STAGE Révisez les temps présentés dans

90

Part 3Directions: In Part 3 of the test, you will listen to 13 conversations. Every conversation will be followed by three questions about it. Each question will have four answer choices marked (A), (B), (C), and (D). Each conversa-tion will be spoken only once and not written out. For each question, choose the answer choice that accurately answers it.

32. What are the woman and the man discussing?(A) The sales team’s vacation(B) The paperwork(C) The organization of the next convention(D) The choice of representatives

33. Who will go to the convention?(A) Taylor, John, Adam, and Paul(B) Taylor, Adam, and Paul(C) John, Adam, and Paul(D) Paul, John, and Taylor

34. Why does the woman recommend Paul?(A) He is an excellent sales representative.(B) He has just returned from vacation.(C) He has made significant progress lately.(D) He knows all the paperwork already.

---

35. Where will the man advertise the restaurant?(A) Online(B) In a newspaper(C) Online and in a newspaper(D) Neither online nor in a newspaper

36. What other ad does the man need to place?(A) Job ads for cooks(B) Want ads for maintenance workers(C) Job ads for cleaners(D) Want ads for builders

37. What can be assumed about the woman?(A) She is the man’s assistant.(B) She doesn’t like advertising.(C) She makes the final decisions about hiring.(D) She is alarmed by the lack of personnel.

---38. What does the woman think about Courtney?

Page 91: BASES ET RVISIONS NCESSAIRES POUR LE TOEIC · 2019. 12. 2. · BASES ET RVISIONS NCESSAIRES POUR LE TOEIC 3 CONSIGNES Ā APPLIQUER AVANT LE STAGE Révisez les temps présentés dans

91TEST BLANC 2

(A) She is looking forward to working with Courtney.(B) She believes Courtney is an expert.(C) She considers Courtney too slow.(D) She is not that fond of Courtney.

39. What can be assumed about the department?(A) It is exceptionally understaffed.(B) It deals with insurance.(C) It’s a Training department.(D) The work in it is badly organized.

40. What does the woman warn the man about?(A) That he might not have his workload reduced right away(B) That he will not benefit from Courtney’s contribution(C) That Courtney learns very slowly(D) That the work will be poorly distributed in the future

---

41. What does the man think about the woman?(A) That she has more money than he does(B) That she can hire him(C) That she is a good manager(D) That she can give him good advice

42. According to the woman, what is the first step in portfolio analysis?(A) Timing the wealth management correctly(B) Examining the checking accounts of the portfolio holder(C) Verifying the stock-bond balance in the portfolio(D) Analyzing swaps and options present in the portfolio

43. What does the man think about high-risk investments?(A) He is completely averse to risk.(B) He might invest some of his money into high-risk securities.(C) He can invest up to a half of his portfolio into high-risk securities.(D) He is eager to invest a serious share of his fortune into high-risk securities.

---

44. When will the meeting take place?(A) At 12 a.m.(B) At noon(C) At 2 p.m.(D) Tomorrow morning

Page 92: BASES ET RVISIONS NCESSAIRES POUR LE TOEIC · 2019. 12. 2. · BASES ET RVISIONS NCESSAIRES POUR LE TOEIC 3 CONSIGNES Ā APPLIQUER AVANT LE STAGE Révisez les temps présentés dans

92

45. What reason does the man give for not being able to attend the meeting?(A) He will be at another important meeting at the same time.(B) He considers the meeting unimportant.(C) He will be receiving new equipment for his office at the same time.(D) He has been away from the office all morning.

46. What does the woman think about the meeting?(A) That it is of utmost importance(B) That the man shouldn’t miss it(C) That not attending it wouldn’t be a problem(D) That their boss will be angry if they both are not present

---

47. What is the woman looking for?(A) Her boarding pass(B) Her boarding gate(C) The right terminal(D) The airport

48. In what direction does the woman need to go?(A) She needs to walk straight ahead.(B) She needs to turn left.(C) She needs to turn right.(D) She needs to turn around.

49. What does the man suggest the woman do before her flight?(A) Accompany him(B) Do some shopping(C) Browse the Internet(D) Hurry up

---

50. What is Sophia’s last name?(A) Whitly(B) Whitley(C) Whitely(D) Whiteley

51. What does Giovanni say about the man Sophia is trying to reach?(A) He is absent.(B) He isn’t taking calls today.(C) He doesn’t take calls from applicants at all.(D) The best way to reach him is to send a message by email.

52. What position did Sophia apply for?

Page 93: BASES ET RVISIONS NCESSAIRES POUR LE TOEIC · 2019. 12. 2. · BASES ET RVISIONS NCESSAIRES POUR LE TOEIC 3 CONSIGNES Ā APPLIQUER AVANT LE STAGE Révisez les temps présentés dans

93TEST BLANC 2

(A) Caller(B) Messenger(C) Hiring manager(D) Assistant manager

---

53. Why do the man and the woman need to reschedule the meeting?(A) Their client can’t make it tomorrow.(B) Their client prefers to meet today.(C) Their colleagues are unavailable.(D) The meeting room is occupied.

54. When does the man prefer to meet?(A) Today at 4 p.m.(B) Today at 5 p.m.(C) Tomorrow at 4 p.m.(D) Tomorrow at 5 p.m.

55. What do the man and the woman need to do before confirming the new meeting time to the client?(A) Clean the meeting room(B) Verify it with their team(C) Ask for client’s opinion(D) Return the client’s call

---

56. What is the man’s ID number?(A) 9384(B) 9843(C) 9834(D) 9348

57. What is the man’s problem?(A) He has been fired.(B) He has not received his salary.(C) He has not used his vacation.(D) He has not been paid for unused vacation.

58. What does the woman say about the problem?(A) That the man should go on vacation(B) That she doesn’t have that name on the file(C) That she can help the man right away(D) That there is an issue with the system

---

59. What can be assumed about the speakers?

Page 94: BASES ET RVISIONS NCESSAIRES POUR LE TOEIC · 2019. 12. 2. · BASES ET RVISIONS NCESSAIRES POUR LE TOEIC 3 CONSIGNES Ā APPLIQUER AVANT LE STAGE Révisez les temps présentés dans

94

(A) All of them have met before.(B) Ms. Tanaka works in the same office as Mr. Dubois.(C) Mrs. Dubois and Ms. Tanaka are friends.(D) All of them have children.

60. Which of the following is true about the Overfield Drive apartment?(A) It is closer to Mr. Dubois’s office than the other apartment.(B) It is larger than the other one.(C) It has a park nearby.(D) Mrs. Dubois considers that it will be better for her children.

61. What will Mr. and Mrs. Dubois do next?(A) Complete the paperwork(B) Go to Mr. Dubois’s office(C) Contact the owner of the Russell Gardens apartment(D) Make their decision about the apartment

---

Our newest laptops – starting from only $399!*including taxes1030X Green Light $799Process 700X $599Steel 300 $499Adequ8 $399

62. What can be assumed about the man?(A) He travels for work.(B) He dislikes fancy applications.(C) His computer’s processor is slow.(D) He doesn’t need a touchscreen for work.

63. Which of the following does the woman find important in a laptop?(A) Numerous applications(B) Good memory(C) Touchscreen(D) Presentation software

64. Look at the graphic. How much will the woman probably pay for her new laptop?(A) $399(B) $799(C) $599(D) $499

---

Page 95: BASES ET RVISIONS NCESSAIRES POUR LE TOEIC · 2019. 12. 2. · BASES ET RVISIONS NCESSAIRES POUR LE TOEIC 3 CONSIGNES Ā APPLIQUER AVANT LE STAGE Révisez les temps présentés dans

95TEST BLANC 2

Midday SandwichesOrder formSandwich Ingredients Desired

QuantityRemarks

Fast and Fresh Swedish bread, salmon, sour cream 1 2 3 4Hot and Healthy Plain roll, chorizo, green salad 1 2 3 4Big and Beautiful Baguette, bacon, eggs, tomato 1 2 3 4Green and Gorgeous Sesame bread, green salad, nuts, olive oil 1 2 3 4

65. What is the woman’s job?(A) Cook(B) Waitress(C) Restaurant manager(D) Cashier

66. Where can the order forms be picked up?(A) From the woman(B) At the entrance(C) At the counter on the right(D) Left of the woman

67. Look at the graphic. Which of the sandwiches would the man need to modify if he were to order it?(A) Fast and Fresh(B) Hot and Healthy(C) Big and Beautiful(D) Green and Gorgeous

---

FitForm GymTuesday Lunchtime Group ClassesTime Class Room12:00 Stretching Room D12:00 Pool Bike Small Pool12:15 Yoga Room A12:30 Body Pump Room C12:45 Boot Camp Outdoor Lawn

68. What is the woman’s problem?(A) She missed her Stretching class.(B) She doesn’t want to go to Body Pump class.(C) She neglected to warm up before her class.(D) She was unaware of a change in the class schedule.

69. Which of the following is true about the Stretching class?(A) It takes place on Tuesdays.(B) It is harder than the Yoga class.

Page 96: BASES ET RVISIONS NCESSAIRES POUR LE TOEIC · 2019. 12. 2. · BASES ET RVISIONS NCESSAIRES POUR LE TOEIC 3 CONSIGNES Ā APPLIQUER AVANT LE STAGE Révisez les temps présentés dans

96

(C) It is always well attended.(D) It is for beginners only.

70. Look at the schedule. How much time does the woman have to warm up?(A) 5 minutes(B) 15 minutes(C) 30 minutes(D) 45 minutes

Part 4Directions: In Part 4 of the test, you will listen to 10 talks. Every talk will be followed by three questions about it. Each question will have four answer choices marked (A), (B), (C), and (D). Each talk will be spoken only once and not written out. For each question, choose the answer choice that accurately answers it.

71. What does the man say about his work hours?(A) They are regular.(B) They are gratifying.(C) They are exciting.(D) They are unusual.

72. Which of the following aspects did the man help tourists with?(A) Accommodation(B) Event tickets(C) Meeting locals(D) Entertainment

73. According to the man, what is an essential quality for becoming a tour guide?(A) Speaking foreign languages(B) Enjoying working with people(C) Being adventurous(D) Being prepared to start over

---

74. At which of the following moments will the offices of Steel Manufacturing be open?(A) Sunday 9:30 a.m.(B) Monday 7:30 a.m.(C) Friday 6:30 p.m.(D) Saturday 7 p.m.

75. Which option would a caller dial when wishing to track a delivery of metalwork?(A) 1(B) 3(C) 4(D) 7

Page 97: BASES ET RVISIONS NCESSAIRES POUR LE TOEIC · 2019. 12. 2. · BASES ET RVISIONS NCESSAIRES POUR LE TOEIC 3 CONSIGNES Ā APPLIQUER AVANT LE STAGE Révisez les temps présentés dans

97TEST BLANC 2

76. Which option would a caller dial when trying to reach the Accounting Department?(A) 1(B) 3(C) 5(D) 6

---

77. What is the man selling?(A) Pastime activities(B) Safety equipment(C) Insurance against accidents(D) Courses

78. According to the talk, which of the following describes deep sea diving?(A) It is often accompanied by mistakes and accidents.(B) It is increasingly popular.(C) It helps stay safe while in water.(D) It is useful for both beginners and veterans.

79. Why does the man mention the Association of Diving Safety Education?(A) To emphasize the importance of diving safety(B) To indicate that a regulatory body exists(C) Because he is a member of this Association(D) Because his product is validated by this Association

---

80. What is the main purpose of this talk?(A) To sell a product(B) To promote green energies(C) To inform about energy storage options(D) To warn against atmospheric phenomena

81. Which of the following is mentioned as an advantage of the solar panel roofing?(A) It is inexpensive.(B) The installation is fast and easy.(C) It guarantees outage protection.(D) The existing roof doesn’t need to be modified.

82. What does the woman say about the robustness of the solar panels?(A) They are prone to wind, rain, and hail.(B) They are as sturdy as roof tiles.(C) They are actually more robust than standard tiles.(D) Various debris can damage them.

---

Page 98: BASES ET RVISIONS NCESSAIRES POUR LE TOEIC · 2019. 12. 2. · BASES ET RVISIONS NCESSAIRES POUR LE TOEIC 3 CONSIGNES Ā APPLIQUER AVANT LE STAGE Révisez les temps présentés dans

98

83. What is the purpose of this speech?(A) To welcome visitors to the job fair(B) To provide a job description(C) To sell services(D) To underline the importance of education

84. Which of the following is not listed as part of the speaker’s responsibilities?(A) Communicating with clients(B) Supervising lighting work(C) Selecting the necessary equipment(D) Teaching cameramen

85. What was the woman’s previous job?(A) Job fair organizer(B) Director of photography(C) Photography teacher(D) Camera operator

---

86. What is the most probable context of this talk?(A) A job interview(B) A thank-you speech(C) An appeal to save the neighborhood(D) An information session

87. Which of the following can be considered a source of inspiration for the woman?(A) Mechanical engineering education(B) Giving back to the community(C) Landscaping techniques(D) Way of thinking different from hers

88. What does the woman promise to do?(A) To change the team’s philosophy completely(B) To take a volunteering position(C) To deliver more than is expected of her(D) To gain more experience in mechanical engineering

---

89. Who is the most direct target audience for this advertisement?(A) Individuals(B) Businesses(C) Government bodies(D) Educational institutions

Page 99: BASES ET RVISIONS NCESSAIRES POUR LE TOEIC · 2019. 12. 2. · BASES ET RVISIONS NCESSAIRES POUR LE TOEIC 3 CONSIGNES Ā APPLIQUER AVANT LE STAGE Révisez les temps présentés dans

99TEST BLANC 2

90. According to the talk, what is the basis of the firm’s reputation?(A) Multiple successful cases(B) Clients’ satisfaction(C) Endorsement by respectable schools(D) A full range of legal services

91. Why does the speaker mention “family” in the advertisement?(A) To stipulate that the firm’s services are primarily for families(B) To point out the proximity between the firm and its clients, as that of a family(C) To highlight that Northbrook Associates is a family-run business(D) To introduce a new specialization of the company: family law

---

92. How many people typically attend the speaker’s seminars at one time?(A) 1-50(B) 50-100(C) 100-150(D) Over a million

93. What characteristic does participation in the seminar produce?(A) Flexibility(B) Creativity(C) Dedication(D) Fortitude

94. What can we infer about the speaker?(A) He is relatively shy.(B) He gives motivational talks.(C) He is a leadership trainer.(D) He is new to this job.

---

Sweetwater Resorts and HotelsResort/Hotel Country OpenedSweetwater Antalya Turkey 2018Sweetwater Rhodes Greece 2017Sweetwater Volterra Italy 2017Sweetwater Dubrovnik Croatia 2016

95. Look at the graphic. Approximately how many years has it taken to build the advertised resort?(A) 1(B) 2(C) 3(D) 5

Page 100: BASES ET RVISIONS NCESSAIRES POUR LE TOEIC · 2019. 12. 2. · BASES ET RVISIONS NCESSAIRES POUR LE TOEIC 3 CONSIGNES Ā APPLIQUER AVANT LE STAGE Révisez les temps présentés dans

100

96. Which of the following features is not advertised?(A) Services for families(B) Conference centers(C) Computer lounges(D) Visits to surrounding towns

97. Which of the following can be assumed about Sweetwater Resorts and Hotels?(A) It has a bonus system for customers.(B) It has hotels all over the world.(C) All its resorts are on the seaside.(D) It focuses on business travelers.

---

98. In what industry does Stephen work?(A) Commercial property rent(B) Event management(C) Construction(D) Food and beverages

99. What is a special feature of Einav’s hall?(A) It has a very popular bar.(B) It is extremely spacious.(C) It is equipped with cutting-edge technology.(D) It boasts a spectacular view.

100. Look at the graphic. What mistake did Stephen make in his plan?(A) The location of the cocktail tables is wrong.(B) The podium should be closer to the entrance.(C) The number of tables is incorrect.(D) The locations of the bar and the podium should be switched.

This is the end of the Listening section.

Page 101: BASES ET RVISIONS NCESSAIRES POUR LE TOEIC · 2019. 12. 2. · BASES ET RVISIONS NCESSAIRES POUR LE TOEIC 3 CONSIGNES Ā APPLIQUER AVANT LE STAGE Révisez les temps présentés dans

101TEST BLANC 2

Reading SectionIn the Reading section of the test, you are asked to demonstrate how well you understand written English. It consists of three parts, with special directions in each part. Remember, you are not allowed to take notes. This section will last exactly 75 minutes.

Part 5Directions: You will read a sentence that contains a blank as well as four answer choices that suggest words to fill the empty space. Select the answer choice (A), (B), (C), or (D) that best completes the sentence.

101. I am convinced that Jake is _________ honest man and not a fraudster.(A) a(B) an(C) the(D) that

102. When exactly did Mr. McDonovan _________ his own company?(A) start(B) grow(C) generate(D) make

103. Tomorrow I _________ leave the office before 7 p.m., sorry.(A) won’t can(B) will cannot(C) will be unable(D) won’t be able to

104. Aren’t you looking forward _________ to Dubai?(A) move(B) to move(C) moving(D) to moving

105. _________ when have you been Ms. O’Connor’s secretary?(A) From(B) After(C) Since(D) Of

106. Gina and Alfredo couldn’t respect the deadline anyway, _________?(A) did they(B) could they(C) isn’t it(D) could it

Page 102: BASES ET RVISIONS NCESSAIRES POUR LE TOEIC · 2019. 12. 2. · BASES ET RVISIONS NCESSAIRES POUR LE TOEIC 3 CONSIGNES Ā APPLIQUER AVANT LE STAGE Révisez les temps présentés dans

102

107. I was supposed to spend Monday with my family, _________ I chose to come to work instead.(A) but(B) if(C) so(D) or

108. Please, never use company _________ when sending personal letters, will you?(A) stationery(B) headletter(C) watermark(D) paperknife

109. Who of our 10 employees lives _________ from the office?(A) farther(B) the farthest(C) further(D) the furthest

110. The Sales Department _________ just signed a sensational new contract with our biggest client!(A) is(B) has(C) was(D) had

111. We _________ to order some extra chairs, just in case.(A) shall(B) should(C) ought(D) must

112. After taking a plane from Vancouver to Frankfurt, I always feel _________.(A) jatleg(B) jatlegged(C) jetlag(D) jetlagged

113. Isn’t Ms. Alexandropoulos a full-time employee? – No, she is an external _________.(A) consultant(B) deliverer(C) professional(D) subordinate

114. I suggest that Yuki _________ the new project with her direct supervisor.(A) discuss(B) discusses(C) discussing(D) to discuss

Page 103: BASES ET RVISIONS NCESSAIRES POUR LE TOEIC · 2019. 12. 2. · BASES ET RVISIONS NCESSAIRES POUR LE TOEIC 3 CONSIGNES Ā APPLIQUER AVANT LE STAGE Révisez les temps présentés dans

103TEST BLANC 2

115. Ms. Bolshova has just given me two great _________ about the bookkeeping!(A) advice(B) advices(C) advises(D) pieces of advice

116. No right-minded person _________ talk rudely to the big boss.(A) would ever(B) would never(C) wouldn’t ever(D) wouldn’t never

117. In our store, you can pay by check, _________ you are a local resident.(A) granted(B) provided(C) delivered(D) supplied

118. Igor felt quite ill yesterday, but today he has called and said that he feels even _________.(A) iller(B) illest(C) worse(D) worst

119. Philippa _________ the mail when the phone rang.(A) sorted(B) was sorting(C) will sort(D) sorts

120. _________ distribute dividends this year?(A) Shouldn’t we(B) Should not we(C) Not should we(D) We not should

121. Our department will surely stay within the _________ for this quarter, even with high travel expenses.(A) funds(B) budget(C) account(D) balance

122. If Pekka _________ more careful yesterday, he wouldn’t have broken his phone.(A) is(B) was(C) were(D) had been

Page 104: BASES ET RVISIONS NCESSAIRES POUR LE TOEIC · 2019. 12. 2. · BASES ET RVISIONS NCESSAIRES POUR LE TOEIC 3 CONSIGNES Ā APPLIQUER AVANT LE STAGE Révisez les temps présentés dans

104

123. I am used _________ with people from different countries.(A) work(B) to work(C) working(D) to working

124. Do you think the company will_________ charges against Mr. Hildebrandt for disclosing the informa-tion to the media?(A) press(B) force(C) urge(D) push

125. The party that took place at the office _________ cheered up the entire team.(A) now(B) tomorrow(C) yesterday(D) next week

126. _________ hire temporary workers to fill in the gaps in the production.(C) Let(A) Let we(C) Shall we(D) Let us

127. Celine _________ submit her hour report last Friday.(A) must(B) were to(C) had to(D) will

128. Can you please ask the _________ to bring the drinks?(A) teller(B) surgeon(C) waiter(D) realtor

129. Our new premises _________ in the most beautiful neighborhood of the city.(A) locate(B) locating(C) are located(D) are locating

130. Will you please send the draft to me first and only _________ to Harry?(A) than(B) that(C) their(D) then

Page 105: BASES ET RVISIONS NCESSAIRES POUR LE TOEIC · 2019. 12. 2. · BASES ET RVISIONS NCESSAIRES POUR LE TOEIC 3 CONSIGNES Ā APPLIQUER AVANT LE STAGE Révisez les temps présentés dans

105TEST BLANC 2

Part 6Directions: You will be presented with four short texts, each containing four blank spaces. Use one of the four answer choices (A), (B), (C), or (D) to fill in the blank. Select the answer choice that best completes the text.

Questions 131 to 134 refer to the following text:__131.__ We have dozens of styles to fit your décor, as well as indoor or weather-resistant outdoor fabrics. Trims, fillings, fabrics, colors, and sizes all come customizable when you order from our website. Our patented mattress is specially crafted to ease the joint pain often caused by sleeping on unsupportive surfaces. __132.__ other pet beds will sag and tear over time, the Faun-PM dog bed has a lifetime warranty: if the product ever fails, we will take it back and give you a full refund.

Surprise your furry friend with a new bed and new toys! Each of our customized items __133.__ with a mystery gift for your pet that may include: a small plush toy, a bag of treats, or a bath set to sample our latest organic dog shampoos.

Order today and open a __134.__ to get an additional 30% off your entire purchase!

131.(A) We are proud to introduce a new line of dog foods: Faun-PM!(B) All our products now come with a 10% discount for new customers!(C) The new Faun-PM dog bed is the best new addition to your home!(D) Look no further: we have everything your dog might need!

132.(A) Afterwards(B) Because(C) While(D) Instead

133.(A) come(B) comes(C) has come(D) have come

134.(A) pet club(B) credit card(C) shipped package(D) membership account

Questions 135 through 138 refer to the following text:Dear Mr. Evan Wallace,

We write to you __135.__ your account held with South Canal Supply.

Page 106: BASES ET RVISIONS NCESSAIRES POUR LE TOEIC · 2019. 12. 2. · BASES ET RVISIONS NCESSAIRES POUR LE TOEIC 3 CONSIGNES Ā APPLIQUER AVANT LE STAGE Révisez les temps présentés dans

106

As you have been made aware, the balance in your account has reached a great debt. The total cost of our service is $10,400, and we request you begin tapering off your expenses and pay your debts before making further investments. We cannot condone any more loans made in your party’s favor __136.__ the balance is settled, including any equipment rentals, purchases, or shipments.

We suppose you have not had return business with your clients, indicating that your business practices are not paying off as you had proposed. As we can afford to keep only __137.__ business partners, legal action will be taken if no progress is made within the next 5 business days.

Sincerely,

Agustin Arellano__138.__South Canal Supply

135.(A) to regard(B) regarding with(C) in regards to(D) regardless of

136.(A) when(B) since(C) after(D) until

137.(A) reliable(B) variable(C) understanding(D) forsaking

138.(A) Legal Services(B) Marketing Team(C) Accounting(D) Direct Investment Department

Questions 139 through 142 refer to the following text:Thank you for __139.__ to Next Step Care!

With your donation, you will receive a care package as to further show our gratitude. Your gift includes can-dies, a storybook, and a festive tea light to light up your end-of-the-year holidays. We will also be sending email __140.__ of the work completed thanks to your donations!

Next Step Care holds volunteer efforts around the world to promote the involvement of charity work and

Page 107: BASES ET RVISIONS NCESSAIRES POUR LE TOEIC · 2019. 12. 2. · BASES ET RVISIONS NCESSAIRES POUR LE TOEIC 3 CONSIGNES Ā APPLIQUER AVANT LE STAGE Révisez les temps présentés dans

107TEST BLANC 2

provide a better quality of life for those who need a little assistance. With your help donating and spreading the message, we hope to reach even the __141.__ corners of the earth.

__142.__ . It will help get our ideas through to your friends and acquaintances.

139.(A) donate(B) to donate(C) donated(D) donating

140.(A) updates(B) upbringings(C) upgrades(D) uploads

141.(A) closest(B) remotest(C) nearest(D) warmest

142.(A) Please accept, once again, the expression of our gratitude.(B) If you have a minute, we ask you to share our website on social media.(C) The money you provide is invaluable for people in need.(D) Your gift package will reach you very soon, we hope.

Questions 143 through 146 refer to the following text:Dear Employees,

Due to the current status of our building’s parking lot, we are forced to implement changes to ensure the safety of our staff and proper organization within the property. We have recently come into a volume of employment that our company-owned parking lot __143.__ contain, forcing some to park outside property lines.

__144.__ However, accidents and similar __145.__ outside the company’s property are not something we will be able to assist with.

For the next couple of weeks, we will be providing parking passes for the adjacent parking garage while we expand and update our current lot. Please report __146.__ the Employee Resources office before leaving to receive your temporary parking benefits.

Once our new parking lot is ready for use, you will receive your permanent parking pass to hang on your mirror, as well as a parking number that corresponds to your employee ID and office location.

Sincerely,Alana GunterHead of HR Department

Page 108: BASES ET RVISIONS NCESSAIRES POUR LE TOEIC · 2019. 12. 2. · BASES ET RVISIONS NCESSAIRES POUR LE TOEIC 3 CONSIGNES Ā APPLIQUER AVANT LE STAGE Révisez les temps présentés dans

108

143.(A) cannot(B) may not(C) mustn’t(D) doesn’t have to

144.(A) It has been brought to our attention that vandalism is frequent in our parking lot.(B) Note that acts of vandalism will no longer be tolerated anywhere within our parking lot.(C) Within our own parking lot, acts of vandalism have never taken place so far.(D) We are able and eager to protect and insure against vandalism within our own parking lot.

145.(A) phenomena(B) causes(C) incidents(D) circumstances

146.(A) for(B) to(C) on(D) in

Part 7Directions: You will be presented with several short texts or combinations of texts as well as questions about the content of these texts. Select the best answer – (A), (B), (C), or (D) – to the question based on the information obtained from these sources.

Questions 147 and 148 refer to the following ticket:NeueHansa FerriesFrom: OSLO To: STOCKHOLMDeparture: May 4, 14:00 Arrival: May 5, 10:00Adults: 1 Children: 1 Vehicles: noneCabin Class: B, double occupancy Price: 2495 NOK

Valid on this route, date, and time only. Not subject to exchange. Not subject to refund. Pets not allowed.

147. What kind of transport is this ticket for?(A) Plane(B) Train(C) Bus(D) Ship

Page 109: BASES ET RVISIONS NCESSAIRES POUR LE TOEIC · 2019. 12. 2. · BASES ET RVISIONS NCESSAIRES POUR LE TOEIC 3 CONSIGNES Ā APPLIQUER AVANT LE STAGE Révisez les temps présentés dans

109TEST BLANC 2

148. What is the duration of the journey?(A) 10 hours(B) 14 hours(C) 20 hours(D) 24 hours

Questions 149 and 150 refer to the following email:From: Audra Sauliunaite, Lightning Online Magazines ltd.To: John McCarranDate/Time: June 20, 9:15 a.m.Subject: Out of Office Reply: URGENT: article modification request

Dear Sender,

Thank you for your message. I am currently on vacation and will return to the office on Monday July 2nd. For important matters, please contact my colleague Mr. Clark Field at [email protected] or by calling +44 5558 8282. Otherwise, I will gladly reply to your email upon my return.

Best regards,

Audra SauliunaiteEditorLightning Online Magazines ltd.

149. Why is Mr. McCarran receiving this email?(A) Ms. Sauliunaite’s is on holiday.(B) Ms. Sauliunaite’s is on a business trip.(C) Ms. Sauliunaite’s doesn’t work for Lightning Online Magazines anymore.(D) Ms. Sauliunaite’s has been replaced by Mr. Clark Field.

150. What will Mr. McCarran most probably do upon the receipt of this email?(A) Wait for Ms. Sauliunaite’s reply on July 2nd(B) Call Ms. Sauliunaite on the phone(C) Contact Mr. Field(D) Send one more email to Lightning Online Magazines ltd.

Questions 151 and 152 refer to the following instructions:Thank you for purchasing our Instant Pressure Cooker Plus!

Pressure cooking surpasses many other means of cooking by shortening the wait time while the food still maintains the vitamins and minerals other methods would have eliminated. Before using your Instant Pres-sure Cooker, make sure that there are no flaws in the composition of the pot or packaging. If any are found, contact us for a free replacement. Use a minimum of one cup of water in the pot to cook properly, but most recipes will provide the proper amount for optimal cooking. After adding the water and food, center the pot on top of a larger burner set to high and monitor the pressure gauge on the lid. Adjust the heat accordingly

Page 110: BASES ET RVISIONS NCESSAIRES POUR LE TOEIC · 2019. 12. 2. · BASES ET RVISIONS NCESSAIRES POUR LE TOEIC 3 CONSIGNES Ā APPLIQUER AVANT LE STAGE Révisez les temps présentés dans

110

until the food is simmering and set your timer. Remember to release the pressure correctly before taking off the lid, and enjoy your meal!

151. What is the first thing to do when cooking food in the device described in the instructions?(A) Raise the temperature of the food(B) Examine the device for flaws(C) Add at least one cup of water(D) Monitor the pressure gauge

152. It can be assumed that Instant Pressure Cooker Plus has which of the following forms?(A) A cup(B) A lid(C) A pot(D) A burner

Questions 153 and 154 refer to the following text message exchange:

Ryu TakahashiHi Danuta. Are you far from the office already?4:37 p.m.

Danuta KowalskaYes, I would say so. What’s up?4:45 p.m.

Ryu TakahashiI can’t connect to the office wi-fi… What could be the matter?4:46 p.m.

Danuta KowalskaOh no! I forgot to tell you. The password has changed.4:50 p.m.Danuta KowalskaThe new one is Ty67GA1.4:51 p.m.

Ryu TakahashiLet me try that and see if it works.4:52 p.m.Ryu TakahashiYes, it has worked. Thank you very much!4:57 p.m.

Danuta KowalskaSorry about that.5:03 p.m.

Ryu TakahashiNo worries, and have a safe trip!5:10 p.m.

153. What can be assumed about Danuta?(A) She knows a lot about new technologies.(B) She is an office manager.(C) She left the office not long ago.(D) She is Ryu’s secretary.

Page 111: BASES ET RVISIONS NCESSAIRES POUR LE TOEIC · 2019. 12. 2. · BASES ET RVISIONS NCESSAIRES POUR LE TOEIC 3 CONSIGNES Ā APPLIQUER AVANT LE STAGE Révisez les temps présentés dans

111TEST BLANC 2

154. In her 4:50 p.m. message, what does Danuta mean by saying “Oh no!”?(A) She doesn’t believe Ryu.(B) She considers that there is no problem with the wi-fi connection.(C) She is disappointed with herself.(D) She is surprised by Ryu’s previous message.

Questions 155 through 157 refer to the following notice:To: All GuestsDate: May 28th

In the wake of the recent changes to building specifications within amusement parks, our location will be going through heavy construction this coming fall. We will temporarily close down all affected rides and buildings until the new specifications have been met, which will include five of our roller coasters, two child-ren’s rides, and three restaurants. Guards will be posted at the entrances to each, providing information and preventing anyone from wandering into unsafe areas. Ticket prices for the season will come with 50% off all food purchases within the park as a consolation for the reduced variety in rides as construction continues; we will also be offering two-day passes at the same price as a single day pass. In the event that you have booked tickets after the 28th of March and before the publication of this notice, you will be reimbursed with the option to trade in your tickets for those you can use at one of our many sister locations. We apologize for any inconvenience and can assure all rides will be open to the public by the 9th of October. For further informa-tion please refer to our company website to find your location’s representative.

155. What is the main topic of this notice?(A) Changes in price policy(B) Upcoming renovation of the park(C) Immediate changes in safety procedures(D) Apologies for previous inaccessibility of the park

156. How many installations will be closed in total?(A) 5(B) 7(C) 10(D) 50

157. Which of the following is true regarding tickets purchased in September?(A) A two-day ticket will cost the same as a one-day ticket.(B) They can be exchanged for tickets at another park.(C) They can be reimbursed.(D) A one-day ticket will be 50% cheaper than regular tickets.

Questions 158 through 160 refer to the following email:Dear Mr. Anderson,

In regards to your online resume, we are contacting you to request a preliminary interview with our team at Mount Green Developers. We find your independent studies in the field of mathematics and engineering to be the exact fit for someone to join our organization. In this position, we will require you to work a combined

Page 112: BASES ET RVISIONS NCESSAIRES POUR LE TOEIC · 2019. 12. 2. · BASES ET RVISIONS NCESSAIRES POUR LE TOEIC 3 CONSIGNES Ā APPLIQUER AVANT LE STAGE Révisez les temps présentés dans

112

shift of on-site work with the nation’s top computer scientists, as well as to lecture visiting students and per-form basic training for interns.

__[1]__ The starting salary for our current position averages $125,000 a year, but we are more than willing to compensate for the educational work and further studies needed to fulfill the role by starting you at $130,000. __[2]__ We offer the highest private insurance and excellent benefits for those just out of school, like your-self. __[3]__ Our organization also requires yearly performance enrichment services, in which we pay for two weeks’ worth of refreshment courses to keep our top-rated staff educated and to encourage the learning process throughout your career. __[4]__

I will be glad to hear from you with your availabilities for the interview.

Thank you,Kim ChoiTalent Acquisition Manager

158. What does Kim Choi offer to Mr. Anderson?(A) A job(B) Salary increase(C) Education courses(D) A meeting

159. Which of the following does the position’s description not include?(A) Teaching students(B) Recruiting interns(C) Working in computer science(D) Continuing education

160. There are four positions in the text, marked [1], [2], [3], and [4]. In which position does the following phrase best fit?There is also the chance to excel to $150,000 by the end of your first two years.(A) [1](B) [2](C) [3](D) [4]

Questions 161 through 163 refer to the following advertisement:Welcome and thank you for choosing Express Tax Services Online, where we fill individual and business tax returns and eliminate the stress that accompanies each tax season. We have tax specialists online 24/7 all year round to handle any questions, comments, or concerns. With your online account, you can upload files, receipts, and employment information to keep track of any small transactions or changes and avoid the heavy workload when filing for the year. You can also choose to file through mail with our express shipping options to ensure your taxes are filed on time, where we are more than happy to reimburse the extra money needed for first class shipping.

ETSO also provides professional personal assistance for owners of small businesses and offers a discount for individuals in their first two years of owning a business. Across the board we have been voted the most

Page 113: BASES ET RVISIONS NCESSAIRES POUR LE TOEIC · 2019. 12. 2. · BASES ET RVISIONS NCESSAIRES POUR LE TOEIC 3 CONSIGNES Ā APPLIQUER AVANT LE STAGE Révisez les temps présentés dans

113TEST BLANC 2

personable and helpful service, being more customized to fit the needs of the individual than any other tax service nationwide.Call the hotline number, located on our front page to be put through to a representative who can answer any questions involving your account.

161. How can the services provided by Express Tax Services Online be described?(A) Stressful(B) Customizable(C) Seasonal(D) Free

162. What do clients, who do not wish to file online, receive from Express Tax Services Online?(A) A discount(B) Reimbursement of postal charges(C) Additional workload(D) Tax refund163. What are the conditions to qualify for a discount for business owners?(A) Being a small business owner and filing online(B) Having owned any business for over 2 years(C) Having been incorporated less than 2 years ago(D) Having owned a small business for less than 2 years

Questions 164 through 167 refer to the following online chat:

Zoltan Petofi17:00

Benjy, Lola, thanks for joining in. So, as you know, I am preparing a proposal for a new and very promising client. I need the project to go as smoothly as possible.

Benjamin Judge17:01

You are welcome, Zoltan. So, how can we help?

Zoltan Petofi17:02

The financial part is more or less clear to me – I mean the costs and the margins. But I need more information regarding the delays.

Lola Svobodova17:03

What’s the scale of the order exactly?

Zoltan Petofi17:05

They are still hesitating, as the final order will depend on both the price and the delays, but for now we are going for 2,000 porcelain plates, 500 saucers, and 20 large porcelain trays.

Lola Svobodova17:06

Just plain or with any pattern?

Zoltan Petofi17:07

Plain white porcelain.

Lola Svobodova17:09

That doesn’t sound like a huge order to me. I am sure that at the current production capacity we can manufacture those in 5 working days.Benjamin, how long will it take your team to pack them afterwards?

Benjamin Judge17:11

As the packing machine is still under repair, we’ll have to do it manually. 3 days, I would say. But if it’s urgent, we can hire temporary personnel. You tell me.

Zoltan Petofi17:12

That actually sounds wonderful to me. “Under two weeks” is better than what the client is expecting. Thank you both so much!

Page 114: BASES ET RVISIONS NCESSAIRES POUR LE TOEIC · 2019. 12. 2. · BASES ET RVISIONS NCESSAIRES POUR LE TOEIC 3 CONSIGNES Ā APPLIQUER AVANT LE STAGE Révisez les temps présentés dans

114

164. What is the company’s business?(A) Food and beverage production(B) Catering(C) China manufacturing(D) Packing and shipping

165. Why has Zoltan Petofi organized this chat session?(A) He is curious about the machines involved in the production.(B) He wants to inform his colleagues about the new client.(C) He needs more financial information from Benjamin and Lola.(D) He would like to know how long the production will take.

166. In Lola Svobodova’s message at 17:03, the word “scale” is closest in meaning to which of the following?(A) Weight(B) Size(C) Degree(D) Measurements

167. In his 17:11 message, what does Benjamin Judge mean by saying “You tell me”?(A) He wants Zoltan to confirm that temporary personnel should be hired.(B) He is disappointed that the packing machine is still under repair.(C) He confirms that he understands that the client’s order is urgent.(D) He allows his colleagues to choose between packing in 3 days and hiring extra personnel.

Questions 168 through 171 refer to the following letter:Dear Sirs,

I have been a consumer of both your beauty product line and your spa services at Spa Z for many years now. My entire skincare regimen is stocked with only your items, I visit the Spa Z resort twice a month, and I have been receiving excellent service since 2003.

__[1]__ Apparently, the staff members you have employed have not been trained to accommodate the changes, and the products you are now selling have inflated prices for less material. __[2]__

For example, last month I walked in two minutes late to my scheduled time and was told the computer system in place would not allow me to check in late, causing a hassle before I could attend my appointment. __[3]__ This week, I was talked into purchasing a new line of hair products and realized the new hairspray was half the size, but twice the cost of the old one. When I inquired about the product and the other sizes, the advisor at the desk wasn’t able to answer any of my questions.

__[4]__ I would deeply regret revoking my membership from the spa, but I am afraid I will have to do it if nothing is done about the undertrained staff.

Sincerely,

Rose Lee

Page 115: BASES ET RVISIONS NCESSAIRES POUR LE TOEIC · 2019. 12. 2. · BASES ET RVISIONS NCESSAIRES POUR LE TOEIC 3 CONSIGNES Ā APPLIQUER AVANT LE STAGE Révisez les temps présentés dans

115TEST BLANC 2

168. How can Rose Lee’s letter be defined?(A) A complaint(B) A reimbursement claim(C) A visit report(D) A performance evaluation

169. What happened last month?(A) Ms. Lee was refused the service.(B) Ms. Lee couldn’t schedule an appointment.(C) Ms. Lee was late for her appointment.(D) Ms. Lee was sold a product she was disappointed with.

170. What is the problem with the new hairspray?(A) It is of low quality.(B) It is way more expensive.(C) It is twice as big as the old one.(D) It doesn’t exist in other sizes.

171. There are four positions in the text, marked [1], [2], [3], and [4]. In which position does the following phrase best fit?However, in the past few months, after the company’s hiatus to adjust to the merger with ResortCo, I have to admit that the quality of your services has deteriorated immensely.(A) [1](B) [2](C) [3](D) [4]

Questions 172 through 175 refer to the following memo:Dear Staff,

As discussed in last week’s meeting, the Optometry Expo is coming up quickly and open spots for last-minute attendees are running low. If you are wishing to attend, you may either respond to the Expo newsletter with your name and workplace information or call them directly to sign up and receive your attendant ID.Those who go will receive vouchers for each panel deemed beneficial to the studies focused on those within our offices, such as ophthalmology and disease. Any panels or classes held outside our current schedule will be at the expense of the individual, though you are encouraged to explore your options to further your education in the field. While accommodation will be included for those attending, food and drink will be at the expense of the employee.We have temporarily entered an exchange program to train new employees in order to prepare them for work in larger, busier offices, rather than the small university clinics they start in. Employees chosen to train multiple individuals will be compensated with a bonus at the end of the month, based on the productivity throughout the training program.If there are any further questions, contact me to clear up any discrepancies as soon as possible.

Thank you,Einar Nilssen

Page 116: BASES ET RVISIONS NCESSAIRES POUR LE TOEIC · 2019. 12. 2. · BASES ET RVISIONS NCESSAIRES POUR LE TOEIC 3 CONSIGNES Ā APPLIQUER AVANT LE STAGE Révisez les temps présentés dans

116

172. What can be assumed about Optometry Expo?(A) It is not very popular, so Mr. Nilssen is trying to promote it.(B) It is rather difficult to register for, so Mr. Nilssen is ready to provide assistance with it.(C) Participation in Optometry Expo is inexpensive.(D) It is held in another city.

173. Which of the following is necessary for registering for Optometry Expo?(A) Attendant ID(B) A panel voucher(C) Optometry Expo newsletter(D) Employment information

174. Which field is Mr. Nilssen’s organization in?(A) Event Management(B) Education(C) Medicine(D) Finance

175. Who will receive a bonus at the end of the month?(A) All those who will have attended the Optometry Expo(B) All new employees(C) Only the new employees who previously worked in university clinics(D) Employees who will have trained other employees

Questions 176 through 180 refer to the following email and chart:Dear Jessica,

To address your questions at the last meeting, we have compiled the survey results of the two previous years in order to compare our 2015 and 2016 marketing models.

Our experts have taken into account our customer satisfaction polls, carried out among 1,000 randomly selec-ted clients. From our findings, we conclude that the marketing event in late 2015 was extremely successful and resulted in higher customer satisfaction ratings the following year. We, therefore, recommend holding it annually in order to boost both brand recognition and customer retention rate.

Thank you for the information supplied at the meeting; it has brought much insight to our online ratings. For instance, your intuition regarding the proportion of unhappy customers has proved to be correct. We will organize an internal meeting as soon as possible in order to address the situation.

Please feel free to call for any clarifications.

Thank you,Donald

Page 117: BASES ET RVISIONS NCESSAIRES POUR LE TOEIC · 2019. 12. 2. · BASES ET RVISIONS NCESSAIRES POUR LE TOEIC 3 CONSIGNES Ā APPLIQUER AVANT LE STAGE Révisez les temps présentés dans

117TEST BLANC 2

Customer Satisfaction Polls

176. Which department does Donald most probably work for?(A) Customer Care(B) Marketing(C) Event Management(D) Sales

177. Why does Donald say that the event was extremely successful?(A) The number of customers happy with the company increased the following year.(B) The brand recognition and retention rate increased the following year.(C) 1,000 clients were extremely happy about the event.(D) The number of clients unsatisfied with the company decreased the following year.

178. What might have been Jessica’s guess during the meeting?(A) The number of happy customers has increased.(B) The number of happy customers has decreased.(C) The number of unhappy customers has increased.(D) The number of unhappy customers has decreased.

179. Which of the following client categories has been stable between 2015 and 2016?(A) “Satisfied”(B) “Not sure”(C) “Unsatisfied”(D) “Very unsatisfied”

180. What will most probably be Donald’s next action?(A) Hold a meeting(B) Organize an event(C) Order more polls(D) Improve customer retention rate

Page 118: BASES ET RVISIONS NCESSAIRES POUR LE TOEIC · 2019. 12. 2. · BASES ET RVISIONS NCESSAIRES POUR LE TOEIC 3 CONSIGNES Ā APPLIQUER AVANT LE STAGE Révisez les temps présentés dans

118

Questions 181 through 185 refer to the following emails:Dear Mr. Aguillar,

We regret to inform you that we have come across a few discrepancies in our last shipment. It has been two months since the changes in your warehouse location were made, and we expected such mistakes to have been addressed and solved by this point in time. First, our previous shipment was made over three trips, not the usual single trip, which has caused a delay in stocking our location. As for supplies such as cups, lids, and straws: a larger amount than was expected has been delivered, resulting in an imbalanced invoice, whereas we ordered only 30 units of each the previous week. Finally, we have also received the wrong cleaning products. Please respond if you may shed some light on the situation.

Sincerely,Justin GradyManager

Dear Mr. Grady,

Thank you for your message sent yesterday. I am happy to provide the explanations regarding the situation.

From the supervising manager’s signature on the previous few orders, we have observed that your management has changed, indicating another person placing the orders. While this person has ordered the correct amount of the usual supplies, he or she may have failed to mention to you the modified schedule of the deliveries. Two months ago we indeed changed warehouse locations. We do apologize for the multiple trucks arriving with different parts of your inventory, but it was the only way to get them to your facility without delay when coming from our multiple warehouses.

Regarding the excessive plasticware, we will gladly allow you to keep the surplus and the new invoice will be addressed to you, reflecting only the 30 units of each item, as per your original order. Finally, your order of the cleaning products will be reviewed by an experienced member of our shipping team, and I will inform you of the results within one business day.

We value you as a customer and thank you for your patience.

Sincerely,

Armando Aguillar,Head of Shipping

181. What does Mr. Grady request in his email?(A) New supplies(B) Reimbursement(C) A delivery change(D) Clarification

182. What justification does Mr. Aguillar provide for three deliveries instead of one?(A) The products delivered were coming from different warehouses.(B) Warehouse location was changed two months ago.(C) The manager at Mr. Grady’s company has changed.(D) The order was a lot larger than usual.

Page 119: BASES ET RVISIONS NCESSAIRES POUR LE TOEIC · 2019. 12. 2. · BASES ET RVISIONS NCESSAIRES POUR LE TOEIC 3 CONSIGNES Ā APPLIQUER AVANT LE STAGE Révisez les temps présentés dans

119TEST BLANC 2

183. What kind of changes have recently taken place at Mr. Grady’s and Mr. Aguillar’s companies?(A) Change of office location at Mr. Grady’s company and change of a manager at Mr. Aguillar’s company(B) Change of a manager at Mr. Grady’s company and change of warehouse locations at Mr. Aguillar’s company(C) Change of a manager at Mr. Grady’s company and change of office location at Mr. Aguillar’s company(D) Change of warehouse locations at Mr. Grady’s company and change of office location at Mr. Aguillar’s company

184. What will Mr. Grady’s company not be paying for?(A) The previous delivery(B) The next delivery(C) Cups, lids, and straws(D) Cleaning products

185. What does Mr. Aguillar say about the situation with the cleaning products?(A) He confirms that Mr. Grady is right.(B) He offers Mr. Grady to keep them.(C) He believes Mr. Grady’s manager has made a mistake.(D) He doesn’t know what happened.

Questions 186 through 190 refer to the following email and its attachments:

Dear Candice,

Congratulations on being selected as a Camp Counselor!

It is important to us that the children attending this summer camp have fun and stay safe, and we are well prepared to show you how. You will find enclosed your schedule for these exciting days. As you can see, Orientation will start at 8 am on Sunday the 25th of May and continue through the following Saturday. All counselors will be shown through the grounds and learn how to guide our campers through each event and activity. Once initiated and settled in, you will be placed into 4 groups of 5 to fit our 4 cabins, which will be adjacent to the children’s 4.The list of items provided and not provided on the premises is also attached.

The payment for your service will be given at the end of the 20-day camp session, before which you can choose to receive a paper copy or direct deposit of your check.

We are looking forward to seeing you among our team members!

Gordon O’BrianPine Camp Director

PROVIDED BRING YOUR OWN- lodging (shared accommodation)- meals (breakfast, lunch, dinner)- camping equipment

- clothes, incl. hiking clothes and shoes- toiletries, incl. sunscreen- personal entertainment items, such as books, music players, etc.

Page 120: BASES ET RVISIONS NCESSAIRES POUR LE TOEIC · 2019. 12. 2. · BASES ET RVISIONS NCESSAIRES POUR LE TOEIC 3 CONSIGNES Ā APPLIQUER AVANT LE STAGE Révisez les temps présentés dans

120

ACTIVITY DATE TIMEArrival Saturday, May 24th 3 pmOrientation Introduction Sunday, May 25th 9 am Camping Safely Monday, May 26th 10 amFire Building Tuesday, May 27th 8 pmHiking and Day Trip Preparation Wednesday, May 28th 8 amKeeping Campers Happy and Busy Thursday, May 29th 10 amAddressing Problem Children Friday, May 30th 2 pmFinal Meeting/Training Completion Saturday, May 31st 9 amCamp Start Monday, June 2nd 7 amMid-Season Review Thursday, June 12th 12 pmCamp Completion/Departure Sunday, June 22nd 5 pm

186. How many counselors will there be at the camp?(A) 4(B) 5(C) 20(D) 24

187. What is an example of an item that Candice will not need to bring to the camp?(A) A tent(B) Shower gel(C) A hat(D) A magazine

188. Which of the following activities is most probably nocturnal?(A) Camping Safely(B) Fire Building(C) Hiking and Day Trip Preparation(D) Keeping Campers Happy and Busy

189. What will happen on June 2nd?(A) Counselors will arrive at the camp.(B) Children will arrive at the camp.(C) Camp policies will be reviewed.(D) The training will start.

190. On which date will the camp counselors be paid?(A) May 24th(B) June 13th(C) June 20th(D) June 22nd

Page 121: BASES ET RVISIONS NCESSAIRES POUR LE TOEIC · 2019. 12. 2. · BASES ET RVISIONS NCESSAIRES POUR LE TOEIC 3 CONSIGNES Ā APPLIQUER AVANT LE STAGE Révisez les temps présentés dans

121TEST BLANC 2

Questions 191 through 195 refer to the following emails and list:From: Ingo Schlossberg, Financial DepartmentTo: Sales Team (click here for the recipients list)Date/Time: February 8, 18:15Subject: IMPORTANT: Fraud Alert – request for information

Dear Travelers,

We have at least one confirmed case of fraudulent use of the company’s credit card. The transaction originated in Singapore; for this reason, I am contacting all those of you currently in the country. Can you please do the following?- Exercise extra care when paying by credit card or withdrawing cash.- Never leave your card unattended.- Within 24 hours, send me a list of your card payments in Singapore, for verification of the company’s accounts.- In case you suspect any fraud with your card, contact me immediately by phone.

If you have any questions, please address them to me as well.

Sincerely,Ingo SchlossbergFraud Prevention ManagerFinancial Department

From: Teresa Van der Merwe, Sales TeamTo: Ingo Schlossberg, Financial DepartmentDate/Time: February 9, 10:05 a.m.Subject: Re: IMPORTANT: Fraud Alert – request for information

Dear Mr. Schlossberg,

In response to your yesterday’s email, I have compiled a list of my payments while working here in Singapore. Keep in mind that some expenses are pending, and I will be able to provide a finished list upon my return to the office on February 10th. I also remind you that there are six of us from my department working at the convention, each one using his or her credit card linked to the same bank account. According to the proce-dures in place, I have kept all receipts, bills, tickets, and any paper statements acquired throughout the trip and have advised my colleagues to do the same to keep our records straight when we return. All financial exchanges made within our department will be directed toward our Head of Sales. There will be additional charges tomorrow for my taxi to the airport, probably around 50 Singapore dollars, and I will save that receipt as well.

Sincerely,Teresa Van der MerweSales Manager

Page 122: BASES ET RVISIONS NCESSAIRES POUR LE TOEIC · 2019. 12. 2. · BASES ET RVISIONS NCESSAIRES POUR LE TOEIC 3 CONSIGNES Ā APPLIQUER AVANT LE STAGE Révisez les temps présentés dans

122

Expense List – Teresa Van der Merwe

Date Expense AmountFebruary 6th Taxi from the airport 48 SGDFebruary 6th Hotel 480 SGDFebruary 7th Convention Pass 50 SGDFebruary 7th Business Dinner 120 SGDFebruary 8th Dry Cleaning 35 SGDFebruary 8th Hotel Room Service 38 SGDFebruary 9th Metro Ticket 10 SGD

Total: 781 SGD

191. What has triggered Mr. Schlossberg’s email?(A) He received a message from Ms. Van der Merwe.(B) An incident related to a credit card has been detected in Singapore.(C) His colleagues needed a reminder of the procedures in place.(D) He needs to compile a list of all expenses made by his colleagues in Singapore.

192. What is the primary purpose of Ms. Van der Merwe’s trip to Singapore?(A) Attending a convention(B) Meeting with clients(C) Investigating fraud(D) Advising her colleagues

193. For how many nights does Ms. Van der Merwe stay in Singapore?(A) 3(B) 4(C) 5(D) 6

194. Which of the following is true about Ms. Van der Merwe’s expense list?(A) It is most probably incomplete.(B) It reflects the expenses of other members of her department.(C) She doesn’t necessarily have proof of payment for all items.(D) It should be reviewed by Head of Sales first.

195. Which of the following items is Ms. Van der Merwe’s second largest expense?(A) Taxi from the airport(B) Hotel(C) Convention Pass(D) Business Dinner

Page 123: BASES ET RVISIONS NCESSAIRES POUR LE TOEIC · 2019. 12. 2. · BASES ET RVISIONS NCESSAIRES POUR LE TOEIC 3 CONSIGNES Ā APPLIQUER AVANT LE STAGE Révisez les temps présentés dans

123TEST BLANC 2

Questions 196 through 200 refer to the following online advertisement, review, and comment:

Offaly County Farmers Market

What would a trip to Ireland be without visiting at least one of its famous farmers markets!

Offaly County Farmers Market, boasting over 70 stalls and open every Saturday, 7 a.m. to 4 p.m., is com-mitted to selling products only from within Offaly County: a policy that has been strictly enforced for the past 25 years.

Vendors offer an unbelievably wide choice of fresh and processed fruit and vegetables, bread and pastries, milk, butter, cheeses, meats, trout, and traditional beverages – all grown, reared, caught, brewed, pickled, baked, smoked, or cooked by the stall holder.

Located only 100 kilometers west of Dublin, the market is easy to reach by car or train. Remember to bring along your largest basket!

You are leaving a review for: Offaly County Farmers MarketDate visited: December 9thYour name *: Mildred Bernstein

Your review:My husband and I gave Offaly Market a try last week and weren’t sorry about it. Less than one hour by train from Dublin, and we were in the middle of a vibrant, welcoming, and generous place. The only regret is that we didn’t find any trout, which is, however, specifically mentioned in the market description. We’ve been compensated though, with exquisite dry bacon – apparently, some variety not found anywhere else in Ireland. Highly recommended!PS. I actually did forget my basket, but that turned out to be an advantage: I had a chance to get a nicely decorated local one, for a very reasonable price.

Date posted: December 15th

* Only your first name will be published

Commenting on: Offaly County Farmers MarketIn response to: Mildred B., posted December 15th

Your name *: Ciaran O’Healy

Your comment:Dear Mildred,We are honoured with your visit to Offaly County Farmers Market and are pleased to learn that it has been an enjoyable experience.As for the trout: I totally agree that it can be a disappointment, as I am a big fan myself, but please keep in mind that trout fishing is authorised in Ireland from February through October only. Next time you visit, I suggest you try our cured products – some of them are even more delicious than the fresh fish!Looking forward to welcoming you again in Offaly!

Page 124: BASES ET RVISIONS NCESSAIRES POUR LE TOEIC · 2019. 12. 2. · BASES ET RVISIONS NCESSAIRES POUR LE TOEIC 3 CONSIGNES Ā APPLIQUER AVANT LE STAGE Révisez les temps présentés dans

124

Ciaran O’HealyPresident, Offaly County Farmers Market Cooperative

Date posted: December 19th

* Only your first name will be published

196. The website where the advertisement appeared targets primarily which of the following groups?(A) Offaly County farmers(B) Offaly County residents(C) Residents of Dublin(D) Foreign tourists

197. On which day of the week did Mildred Bernstein publish her review?(A) Monday(B) Friday(C) Saturday(D) Sunday

198. Why couldn’t Mildred Bernstein find trout on the market?(A) The advertisement was misleading: trout is never sold in Offaly.(B) She prefers cured fish, but only fresh trout were available.(C) She visited the market outside of the trout fishing season.(D) She was offered dry bacon instead and preferred it.

199. The word “reasonable” in the last sentence of Mildred Bernstein’s review is closest in meaning to:(A) low(B) intelligent(C) exaggerated(D) rational

200. Which of the following is probably true about Ciaran O’Healy?(A) He is involved in Offaly Market’s organization.(B) He dislikes fish products.(C) He likes posting comments regarding Offaly Market.(D) He is a vendor at Offaly Market.

This is the end of the test. If there is time remaining, you may go back and review your answers for Parts 5, 6, and 7.

Page 125: BASES ET RVISIONS NCESSAIRES POUR LE TOEIC · 2019. 12. 2. · BASES ET RVISIONS NCESSAIRES POUR LE TOEIC 3 CONSIGNES Ā APPLIQUER AVANT LE STAGE Révisez les temps présentés dans

125TEST BLANC 2

TOEIC L&RPRACTICE TEST 2

file TOEICL&R.PracticeTest2.Listening.mp3

Listening SectionIn the Listening section of the test, you are asked to demonstrate how well you understand spoken English. It consists of four parts, with special directions in each part. Remember, you are not allowed to take notes. This section will last approximately 45 minutes.

Part 1Directions: In Part 1 of the test, you will be shown 6 photographs and hear four statements about each pho-tograph. The statements will be marked (A), (B), (C), and (D). All statements will be spoken only once and not written down. For each photograph, choose the statement that accurately describes what can be seen in the photograph.

Look at a sample photograph.

Listen to the four statements.

(A) The ship is very modern.(B) Passengers are boarding the ship.(C) Palm-trees are planted along the waterfront.(D) People are having a party on a boat.

Statement (C), “Palm-trees are planted along the waterfront,” is the most accurate description of what can be seen in the photograph. Therefore, you should mark your answer choice (C).

This part will now start with Question 1. Select your answers as you listen.

Number 1.(A) There are four portions of dessert on the table.(B) The forks are next to the cups.(C) Each cup is placed on a saucer.(D) Two people are enjoying coffee.

Page 126: BASES ET RVISIONS NCESSAIRES POUR LE TOEIC · 2019. 12. 2. · BASES ET RVISIONS NCESSAIRES POUR LE TOEIC 3 CONSIGNES Ā APPLIQUER AVANT LE STAGE Révisez les temps présentés dans

126

Number 2.(A) They are working in an office.(B) The man is driving a car.(C) The woman is looking at the screen.(D) She is wearing sunglasses.

Number 3.(A) A woman is crossing the street.(B) Two towers are on the same side of the road.(C) People are watching a parade.(D) The clock tower is decorated with banners.

Number 4.(A) A doctor is checking the woman’s ears.(B) She is having her eyesight examined.(C) A man is operating medical equipment.(D) A woman is looking attentively into a microscope.

Number 5.(A) People are taking advantage of the sunny weather.(B) There are bathers in both swimming pools.(C) Spectators are cheering the swimmers.(D) The pool area is closed.

Number 6.(A) A man is waiting for the bus.(B) A man is buying a ticket.(C) A man is validating his pass.(D) A man is walking to the end of the bus.

Part 2Directions: In Part 2 of the test, you will listen to 25 questions, each followed by three potential responses. The responses will be marked (A), (B), and (C). All questions and responses will be spoken only once and not written down. For each question, choose the response that accurately answers it.

This part will now start with Question 7. Select your answers as you listen.

Number 7. How can I change the password for my email account?(A) Try sending an email to me instead.(B) Only the IT team can. Talk to them.(C) I have lost my password too.

Number 8. How many questions should there be in the Customer Satisfaction Questionnaire?(A) A maximum of ten.(B) I want every customer to fill in the questionnaire.(C) Print several hundreds of them.

Page 127: BASES ET RVISIONS NCESSAIRES POUR LE TOEIC · 2019. 12. 2. · BASES ET RVISIONS NCESSAIRES POUR LE TOEIC 3 CONSIGNES Ā APPLIQUER AVANT LE STAGE Révisez les temps présentés dans

127TEST BLANC 2

Number 9. We have extra staplers in the supply closet, don’t we?(A) Staples come in different shapes and sizes.(B) The supply closet is on the second floor.(C) There were some, last time I checked.

Number 10. What do you think about the scandal around the new advertising campaign?(A) Don’t you believe the product advertised is revolutionary?(B) The campaign is horribly structured, in my opinion.(C) Well, it has added even more publicity, hasn’t it?

Number 11. Which color do you prefer for the carpet: dark red or light gray?(A) Dark gray sounds good to me.(B) Neither, to be honest.(C) OK, what are the options?

Number 12. I can hardly hear you. Can you call my landline instead?(A) Sure. I will in a second.(B) Of course. Call my mobile.(C) I am calling to discuss your email.

Number 13. Who is responsible for the logistics of the Sydney event?(A) Mr. Ricardo goes to Sydney every year.(B) Leonidas and his team, I believe.(C) The guest speaker is Dr. Everton.

Number 14. Where will the new branch be opened?(A) Within a couple of years, I think.(B) Mr. Iwamoto will be responsible for the opening.(C) Somewhere in South-East Asia.

Number 15. What’s that strange noise?(A) I don’t see anything strange around me.(B) Mrs. Zhang has brought her little dog to the office again.(C) That must be the responses to our want ad posting.

Number 16. When will NGYT Incorporated finally pay the bill we sent them?(A) I agree – we should wait a little more.(B) Their incorporation took place last month.(C) They promised to by the end of this week.

Number 17. Would you care for some dessert?(A) Yes, I’ll have a shrimp cocktail, please.(B) Not today, thank you.(C) No, I don’t care.

Number 18. Can you describe the individual you saw in the hallway?(A) Yes, every floor of the building has individual offices.(B) I was so scared, I am afraid I don’t remember any details.(C) I think it was wide and well illuminated.

Page 128: BASES ET RVISIONS NCESSAIRES POUR LE TOEIC · 2019. 12. 2. · BASES ET RVISIONS NCESSAIRES POUR LE TOEIC 3 CONSIGNES Ā APPLIQUER AVANT LE STAGE Révisez les temps présentés dans

128

Number 19. Do we really have to drive all the way there?(A) No, we can take the train and then rent a car.(B) Yes, driver’s license is obligatory there.(C) No, this is not what we have.

Number 20. When will we finally create the Risk Management service?(A) The services we provide are not risky.(B) After another couple of fraud cases, I guess.(C) The management risks everything in this affair.

Number 21. Wasn’t Mr. Severski a judge before?(A) Yes, he was judged severely.(B) I think he doesn’t remember.(C) No, just a notary.

Number 22. Which department has hired the most people this month?(A) Department stores constantly hire new employees.(B) After what happened, clearly Cybersecurity.(C) A lot of people were fired from this department just a month ago.

Number 23. Do you take the tube every morning to come to work?(A) Actually, I ride my scooter.(B) Yes, I will have two tubes of medicine.(C) I am never late for work in the morning.

Number 24. Why wasn’t the CEO present at the meeting?(A) The detailed minutes of the meeting.(B) A very interesting coincidence.(C) An early business lunch.

Number 25. What time is the checkout, please?(A) You can check out now if you wish.(B) Noon, at the latest.(C) Come and pick up your check at 10 o’clock.

Number 26. How many CVs have we received?(A) Hundreds – that means a lot of work for us.(B) I have sent my CV too.(C) About two and a half, as of today.

Number 27. Etienne is a great salesman, isn’t he?(A) Yes, he quit the company a week ago.(B) No, no one knows him that well.(C) Yes, and at such a young age!

Number 28. In which currencies can I hold an account at your bank?(A) If it’s a checking account, then only euros and Danish Krones.(B) You can always exchange currencies at our bank.(C) Currently, you do not hold any account at all with us.

Page 129: BASES ET RVISIONS NCESSAIRES POUR LE TOEIC · 2019. 12. 2. · BASES ET RVISIONS NCESSAIRES POUR LE TOEIC 3 CONSIGNES Ā APPLIQUER AVANT LE STAGE Révisez les temps présentés dans

129TEST BLANC 2

Number 29. Stu, did you pay by credit card or cash in the restaurant last night?(A) I didn’t. It was Rebecca.(B) The restaurant last night was really expensive.(C) I have a new credit card. Have a look!

Number 30. So, is it profit or loss for the second quarter?(A) It’s a lot better than the first quarter.(B) Yes, I have finished my calculations.(C) The fixed costs were too high, so you can guess...

Number 31. Who is the new head of the Public Relations Department?(A) All the relations have been employed.(B) Yes, the old head got promoted last week.(C) Some lady I don’t know.

Part 3Directions: In Part 3 of the test, you will listen to 13 conversations. Every conversation will be followed by three questions about it. Each question will have four answer choices marked (A), (B), (C), and (D). Each conversation will be spoken only once and not written out. For each question, choose the answer choice that accurately answers it.

Questions 32 through 34 refer to the following conversation:WOMAN: Today we need to talk about our next convention. We must decide who will represent us.MAN: John, Adam, and Taylor have all been exceptional representatives of our sales team. I think we should ask them to take on the task.WOMAN: Adam has already declined, as he is going to be on vacation. But Paul has been greatly improving, so he would be a good substitute.MAN: I agree; I’ll send the three of them an email with the appropriate paperwork.

Number 32. What are the woman and the man discussing?Number 33. Who will go to the convention?Number 34. Why does the woman recommend Paul?

Questions 35 through 37 refer to the following conversation:MAN: I’m going to put out an ad for our new restaurant opening. Should I place it online or in the news-paper?WOMAN: Both would be better. We still need to find line-cooks and a cleaning service for after hours.MAN: I’ve already placed an ad for them, but we will need to get someone for the upkeep of the grounds around the building.WOMAN: We’ve received a couple of applications already. I’ll let you look at them before I make any decisions.

Number 35. Where will the man advertise the restaurant?Number 36. What other ad does the man need to place?Number 37. What can be assumed about the woman?

Page 130: BASES ET RVISIONS NCESSAIRES POUR LE TOEIC · 2019. 12. 2. · BASES ET RVISIONS NCESSAIRES POUR LE TOEIC 3 CONSIGNES Ā APPLIQUER AVANT LE STAGE Révisez les temps présentés dans

130

Questions 38 through 40 refer to the following conversation:WOMAN: I can’t wait for Courtney to be transferred to our department. She is going to be so much help!MAN: I had to evaluate five different insurance cases yesterday; I’m excited for the work to be distributed better.WOMAN: We will have to train her to start with, so it may be more work than it’s worth at first.MAN: Then I hope she learns quickly!

Number 38. What does the woman think about Courtney?Number 39. What can be assumed about the department?Number 40. What does the woman warn the man about?

Questions 41 through 43 refer to the following conversation:MAN: Good morning, I was wondering if you could take a look at my portfolio and tell me what you think of it. I know you’ve had more experience in investing.WOMAN: No problem. Wealth management becomes easier to understand with time. I will start by checking if your portfolio is well-balanced in terms of stock and bonds and provide some suggestions within a couple of days.MAN: Yes, please, I would really appreciate that. Also, keep in mind that I would be willing to invest a small percentage into higher-risk securities, such as swaps and options.

Number 41. What does the man think about the woman?Number 42. According to the woman, what is the first step in portfolio analysis?Number 43. What does the man think about high-risk investments?

Questions 44 through 46 refer to the following conversation:WOMAN: Have you received the memo for the meeting this afternoon? It is scheduled for 12 p.m.; the boss said he had important news.MAN: I haven’t seen anything; I’ve been away from my desk all morning. I hope it’s not too important – I have to meet with a client then, and I can’t miss that.WOMAN: Just between us, it’s really only to tell us about the equipment upgrades we’re supposed to receive. New desktops, printers, and scanners are coming in for the whole office building.

Number 44. When will the meeting take place?Number 45. What reason does the man give for not being able to attend the meeting?Number 46. What does the woman think about the meeting?

Questions 47 through 49 refer to the following conversation:WOMAN: Excuse me, could you point me in the right direction? I need to find the terminal for my next flight, but this airport is enormous! Here’s my boarding pass.MAN: Of course I can. You’re going to be looking for terminal 3A, which is just at the end of the southeast corridor behind you. I can take you there if you like, but you do have 2 hours until your flight leaves, and we have many shops and restaurants for you to browse until then.WOMAN: I should be able to find it now, thank you for your help!

Page 131: BASES ET RVISIONS NCESSAIRES POUR LE TOEIC · 2019. 12. 2. · BASES ET RVISIONS NCESSAIRES POUR LE TOEIC 3 CONSIGNES Ā APPLIQUER AVANT LE STAGE Révisez les temps présentés dans

131TEST BLANC 2

Number 47. What is the woman looking for?Number 48. In what direction does the woman need to go?Number 49. What does the man suggest the woman do before her flight?

Questions 50 through 52 refer to the following conversation:MAN: Good afternoon and thank you for calling Safe Co., this is Giovanni, how can I help you?WOMAN: Hello, my name is Sophia Whitely, W-H-I-T-E-L-Y. Is there a hiring manager I could speak with?MAN: Our hiring manager isn’t working today, but I would be able to take a message for him.WOMAN: I would just like to request an update on the status of my application for the assistant manager position. Thank you.

Number 50. What is Sophia’s last name?Number 51. What does Giovanni say about the man Sophia is trying to reach?Number 52. What position did Sophia apply for?

Questions 53 through 55 refer to the following conversation:MAN: I am afraid we may have to reschedule the 5-o’clock meeting with our new client tomorrow. Half of our team will not be able to make it.WOMAN: They have just called back and said that they have an opening to meet today if it fits our schedule better.MAN: I’ll ask the others if they would like to meet here at 4 p.m. Our meeting room should be clear then.WOMAN: Tell me when you’ve decided, and I’ll return their call.

Number 53. Why do the man and the woman need to reschedule the meeting?Number 54. When does the man prefer to meet?Number 55. What do the man and the woman need to do before confirming the new meeting time to the client?

Questions 56 through 58 refer to the following conversation:MAN: I came with questions about my last paycheck, would you be able to help me?WOMAN: Yes sir; what was the name of the file?MAN: Heinz Hoffman. Employee ID is 9384. I was terminated two weeks ago but never got a check for my unused vacation days. Would I be able to pick that up here?WOMAN: I have it right here in the system, I just need to print it out.

Number 56. What is the man’s ID number?Number 57. What is the man’s problem?Number 58. What does the woman say about the problem?

Questions 59 through 61 refer to the following conversation:MAN: Good morning, Ms. Tanaka. We have good news for you: It’s been difficult, but we have made our decision.WOMAN 1: Mr. Dubois, Mrs. Dubois, great to see you. So, is it Overfield Drive or Russell Gardens?WOMAN 2: Russell Gardens. We know the apartment is smaller, but having a park nearby is a real advantage for the kids.MAN: And it’s two metro stations closer to my office, too!WOMAN 2: So, what’s the next step? Some paperwork, for sure?

Page 132: BASES ET RVISIONS NCESSAIRES POUR LE TOEIC · 2019. 12. 2. · BASES ET RVISIONS NCESSAIRES POUR LE TOEIC 3 CONSIGNES Ā APPLIQUER AVANT LE STAGE Révisez les temps présentés dans

132

WOMAN 1: Yes, I’m gonna give you some forms to fill in first, and then I’ll contact the owner in order to start the sale procedure.MAN: Can we fill them in now?WOMAN 1: Of course. Please have a seat.

Number 59. What can be assumed about the speakers?Number 60. Which of the following is true about the Overfield Drive apartment?Number 61. What will Mr. and Mrs. Dubois do next?

Questions 62 through 64 refer to the following conversation:WOMAN: Would you be able to recommend a laptop for work?MAN: I use the 1030X Green Light. It has a touchscreen and works well for presentations and travel.WOMAN: I don’t need a lot of fancy applications; I just need a fast processor and adequate memory.MAN: Then I would suggest the Steel 300. It’s the newest model and has the best design for what you want to do.

Number 62. What can be assumed about the man?Number 63. Which of the following does the woman find important in a laptop?Number 64. Look at the graphic. How much will the woman probably pay for her new laptop?

Questions 65 through 67 refer to the following conversation:WOMAN: Welcome to Midday Sandwiches! Can I have your order form, please?MAN: Actually, this is my first time here. Can you please explain how it works?WOMAN: With pleasure. You take one of the order forms at the entrance, check the desired quantity cor-responding to your choice, and bring the form to me. You pay here and pick up your sandwich at the large counter on your right.MAN: Ok, sounds like a great concept. What if I want to modify something in my sandwich?WOMAN: You write it on the form. Remember: you can remove ingredients, but not add them.MAN: That should be fine. I am only allergic to dairy products.

Number 65. What is the woman’s job?Number 66. Where can the order forms be picked up?Number 67. Look at the graphic. Which of the sandwiches would the man need to modify if he were to order it?

Questions 68 through 70 refer to the following conversation:WOMAN: Hello. I was wondering why there is no one in the Stretching class. It is supposed to start in 5 minutes, and usually there is a sizeable group of people in by now.MAN: Hi. Actually, the Stretching class has been moved to Thursday noon. The announcement was posted at the reception, as well as on our Internet site. I am sorry for the inconvenience.WOMAN: Oh no… Well, since I am here anyway, is there anything else I can do at this hour?MAN: You sure can. Why don’t you try the Yoga class? It’s a little harder, for sure, but even beginners are welcome. Another option is Body Pump.WOMAN: I definitely wanna try Yoga. Thank you!MAN: It’s in Room A. I suggest you warm up a little bit first.

Number 68. What is the woman’s problem?Number 69. Which of the following is true about the Stretching class?Number 70. Look at the schedule. How much time does the woman have to warm up?

Page 133: BASES ET RVISIONS NCESSAIRES POUR LE TOEIC · 2019. 12. 2. · BASES ET RVISIONS NCESSAIRES POUR LE TOEIC 3 CONSIGNES Ā APPLIQUER AVANT LE STAGE Révisez les temps présentés dans

133TEST BLANC 2

Part 4Directions: In Part 4 of the test, you will listen to 10 talks. Every talk will be followed by three questions about it. Each question will have four answer choices marked (A), (B), (C), and (D). Each talk will be spoken only once and not written out. For each question, choose the answer choice that accurately answers it.

Questions 71 through 73 refer to the following talk:MAN: I believe that becoming a tour guide was one of the best career choices I could have taken. I spoke Italian fluently and had the right attitude, so obviously, I jumped at the opportunity to fly to Italy and begin again. The hours were odd and occasionally long, but I lived in a beautiful apartment with a view of the sea. Every day was an adventure, and I was regularly making friends from all over the world. The adrenaline of a constantly changing and moving job was enough to keep me excited for the next day. I helped tourists find transportation, food, lodging, and any other sights along the way to enhance their visit. They were always grateful for my help, and I would recommend this job to anyone who enjoys the company of others.

Number 71. What does the man say about his work hours?Number 72. Which of the following aspects did the man help tourists with?Number 73. According to the man, what is an essential quality for becoming a tour guide?

Questions 74 through 76 refer to the following voice message:WOMAN: Hello, and thank you for calling Steel Manufacturing, proud to be the leading provider of metal works. To continue in Spanish, please press 1.We offer you the highest quality machinery, supplies, and metalwork services. Our office hours are 8 a.m. to 6 p.m. Monday through Saturday, and 9 a.m. to 5 p.m. on Sunday. If you know the extension of the person you are trying to reach, please enter it at any time. To speak to a customer service representative in our office, please press 2. For our employee directory, please press 3. To track your order, please press 4. For a full list of our services, please press 5. To hear all options again, please press 7. For all other questions, please press 6 or wait to be automatically put through to a customer service representative.

Number 74. At which of the following moments will the offices of Steel Manufacturing be open?Number 75. Which option would a caller dial when wishing to track a delivery of metalwork?Number 76. Which option would a caller dial when trying to reach the Accounting Department?

Questions 77 through 79 refer to the following talk:MAN: About 70% of the Earth is covered in water, and among many of the pastimes quickly growing in popularity, deep sea diving takes full advantage of this immense playground. To remedy any amateur mistakes or accidents, we provide a comprehensive course to those seeking a professional approach to their hobby. Diving safety is incredibly valuable and becoming more prevalent with the Association of Diving Safety Edu-cation. Our courses provide you with the expertise needed to help you stay safe with the proper techniques and equipment. It is available for new young divers, as well as veterans who are in need of refreshment.

Number 77. What is the man selling?Number 78. According to the talk, which of the following describes deep sea diving?Number 79. Why does the man mention the Association of Diving Safety Education?

Page 134: BASES ET RVISIONS NCESSAIRES POUR LE TOEIC · 2019. 12. 2. · BASES ET RVISIONS NCESSAIRES POUR LE TOEIC 3 CONSIGNES Ā APPLIQUER AVANT LE STAGE Révisez les temps présentés dans

134

Questions 80 through 82 refer to the following talk:WOMAN: Hello and thank you for hearing us today! My colleagues and I are going to take this opportunity to explain to you the real benefits of solar panel roofing. Our company’s fully customizable roofs are tailored to fit your home’s individual energy needs while maintaining both its form and function. They are not only a more attractive choice for the modern home, but also much sturdier than standard roof tiles, giving additional protection against wind, rain, hail, and various debris. As your home utilizes its power, our on-site battery allows for long-term energy storage, ensuring uninterrupted use in the case of an outage. While amply cove-ring your household’s electricity needs, our solar roof technology promises to last for the next 40 years!

Number 80. What is the main purpose of this talk?Number 81. Which of the following is mentioned as an advantage of the solar panel roofing?Number 82. What does the woman say about the robustness of the solar panels?

Questions 83 through 85 refer to the following speech:WOMAN: Welcome to the job fair! Hopefully, you’ve all found professions that interest you to take part in. I am a director of photography, which means I oversee the lighting and camera work on a given project. I talk with the directors and establish their intentions, and then choose equipment accordingly. While you need only two years of school in order to become certified, a lot of the experience required for the job, like I have, will be gained as a camera operator. It takes a while to work your way up to director of photography, but with the right vision, you’ll be able to achieve it in time.

Number 83. What is the purpose of this speech?Number 84. Which of the following is not listed as part of the speaker’s responsibilities?Number 85. What was the woman’s previous job?

Questions 86 through 88 refer to the following talk:WOMAN: Well, this company first stood out to me when I saw the growth in the neighborhood that was founded on its work. The new parks and cleaner streets were a huge inspiration for me; giving back to the community is a huge part of the motivation in my work. I started out with the volunteer jobs that were offered through your organization and was thrilled to see an opening for a full-time position. I love seeing others with similarly well-intentioned philosophies and would look forward to each day I got to be a part of it. My mechanical engineering background and experience are exactly what this job was created for, and I know I can provide just what the position requires, and more.

Number 86. What is the most probable context of this talk?Number 87. Which of the following can be considered a source of inspiration for the woman?Number 88. What does the woman promise to do?

Questions 89 through 91 refer to the following advertisement:MAN: At Northbrook Associates, we provide complete assistance with your legal needs. We strive for excel-lence and client satisfaction, specializing in labor law, immigration law, civil rights, and consumer law. From your first call at our office, you’ll know you’re in good hands as we guide you through the process from start to finish. Our expert legal representatives have been certified by the highest-ranking law schools across the country and are fiercely motivated to put in the extra work for their clients. Joining our legal team as a patron provides you not only with top-rate legal representation but with a family built on the foundation of always lending a helping hand.

Page 135: BASES ET RVISIONS NCESSAIRES POUR LE TOEIC · 2019. 12. 2. · BASES ET RVISIONS NCESSAIRES POUR LE TOEIC 3 CONSIGNES Ā APPLIQUER AVANT LE STAGE Révisez les temps présentés dans

135TEST BLANC 2

Number 89. Who is the most direct target audience for this advertisement?Number 90. According to the talk, what is the basis of the firm’s reputation?Number 91. Why does the speaker mention “family” in the advertisement?

Questions 92 through 94 refer to the following speech:MAN: I’m happy to be here today. It isn’t often that I get to talk to millions of people at one time. Typically, my seminars are for smaller audiences so that I can truly dedicate time to each participant. I like to leave plenty of time for open discussion during my talks, and I find that once you get over 50 people in a room, this is much more challenging. When I have particularly large audiences, and by that, I mean 100 or more, I often break people up into small groups so that I can circulate and contribute to each discussion. I also feel like this kind of small group work fosters incredible creativity and independence. And truly, that’s what leadership training is all about.

Number 92. How many people typically attend the speaker’s seminars at one time?Number 93. What characteristic does participation in the seminar produce?Number 94. What can we infer about the speaker?

Questions 95 through 97 refer to the following advertisement:MAN: Sweetwater Resorts and Hotels are proud to present you our latest opening in beautiful seaside Tur-key. In our newest location, we offer a family-friendly environment surrounded by the beautiful mountains. The ground was broken in 2015 and finished swiftly, creating hundreds of jobs and bringing in new waves of tourism that boosted the life of the towns in the surrounding area. We offer a range of packages, including single rooms with exclusive spa access and multiple room packages with access to our conference centers and computer lounges. Accumulate points with our rewards system after staying several nights, and soon you’ll be vacationing for free!

Number 95. Look at the graphic. Approximately how many years has it taken to build the advertised resort?Number 96. Which of the following features is not advertised?Number 97. Which of the following can be assumed about Sweetwater Resorts and Hotels?

Questions 98 through 100 refer to the following voice message:MAN: Hello Einav. This is Stephen calling from Outward Events, regarding the setup for our upcoming cocktail with the construction business representatives. It’s rather simple, really, but I will send you the plan anyway. We’re gonna need 8 cocktail tables positioned as close to the windows as possible, so that the partici-pants can enjoy the gorgeous view from your hall. Along the opposite wall, please place the bar, the longest you can make, with several barmen working simultaneously. The podium with two microphones should be facing the entrance door, so I guess the best place for it is the back of the room. Call me if you have any questions!

Number 98. In what industry does Stephen work?Number 99. What is a special feature of Einav’s hall?Number 100. Look at the graphic. What mistake did Stephen make in his plan?

This is the end of the Listening section.

Page 136: BASES ET RVISIONS NCESSAIRES POUR LE TOEIC · 2019. 12. 2. · BASES ET RVISIONS NCESSAIRES POUR LE TOEIC 3 CONSIGNES Ā APPLIQUER AVANT LE STAGE Révisez les temps présentés dans
Page 137: BASES ET RVISIONS NCESSAIRES POUR LE TOEIC · 2019. 12. 2. · BASES ET RVISIONS NCESSAIRES POUR LE TOEIC 3 CONSIGNES Ā APPLIQUER AVANT LE STAGE Révisez les temps présentés dans

TOEIC L&RANSWERS

Page 138: BASES ET RVISIONS NCESSAIRES POUR LE TOEIC · 2019. 12. 2. · BASES ET RVISIONS NCESSAIRES POUR LE TOEIC 3 CONSIGNES Ā APPLIQUER AVANT LE STAGE Révisez les temps présentés dans

138

TOEIC L&RPRACTICE TEST 1

Listening Section1. B2. C3. D4. A5. D6. B7. C8. B9. C10. A11. B12. A13. C14. A15. B16. A17. C18. B19. A20. C21. B22. B23. A24. C25. B26. C27. A28. B29. C30. B31. A32. B33. A34. D

35. D36. C37. A38. C39. B40. D41. A42. B43. C44. D45. B46. A47. A48. D49. B50. A51. B52. C53. D54. C55. B56. A57. C58. D59. D60. C61. B62. D63. C64. D65. A66. C67. B68. B

69. C70. A71. D72. C73. D74. C75. D76. D77. C78. A79. D80. A81. C82. B83. B84. A85. C86. A87. B88. D89. A90. B91. A92. D93. B94. C95. A96. B97. A98. C99. B100. C

Page 139: BASES ET RVISIONS NCESSAIRES POUR LE TOEIC · 2019. 12. 2. · BASES ET RVISIONS NCESSAIRES POUR LE TOEIC 3 CONSIGNES Ā APPLIQUER AVANT LE STAGE Révisez les temps présentés dans

139RÉPONSES TEST 1

Reading Section101. D102. A103. C104. B105. B106. C107. C108. A109. D110. C111. B112. D113. A114. D115. A116. B117. C118. A119. D120. B121. C122. A123. B124. C125. D126. B127. A128. B129. C130. D131. A132. C133. D134. D

135. C136. B137. B138. A139. A140. A141. D142. B143. C144. C145. B146. D147. B148. B149. A150. C151. C152. D153. A154. D155. C156. A157. D158. B159. A160. C161. C162. A163. C164. A165. B166. C167. D168. A

169. B170. A171. D172. B173. B174. D175. C176. A177. C178. D179. B180. C181. B182. A183. A184. D185. B186. B187. A188. D189. C190. C191. B192. D193. D194. B195. D196. C197. A198. D199. A200. D

Page 140: BASES ET RVISIONS NCESSAIRES POUR LE TOEIC · 2019. 12. 2. · BASES ET RVISIONS NCESSAIRES POUR LE TOEIC 3 CONSIGNES Ā APPLIQUER AVANT LE STAGE Révisez les temps présentés dans

140

TOEIC L&RPRACTICE TEST 2

Listening Section1. C2. C3. D4. B5. A6. C7. B8. A9. C10. C11. B12. A13. B14. C15. B16. C17. B18. B19. A20. B21. C22. B23. A24. C25. B26. A27. C28. A29. A30. C31. C32. D33. D34. C35. C

36. B37. C38. A39. B40. A41. D42. C43. B44. B45. A46. C47. C48. D49. B50. C51. A52. D53. C54. A55. B56. A57. D58. C59. A60. B61. A62. A63. B64. D65. D66. B67. A68. D69. C70. B

71. D72. A73. B74. A75. C76. D77. D78. B79. A80. A81. D82. C83. B84. D85. D86. A87. B88. C89. A90. C91. B92. A93. B94. C95. C96. D97. A98. B99. D100. C

Page 141: BASES ET RVISIONS NCESSAIRES POUR LE TOEIC · 2019. 12. 2. · BASES ET RVISIONS NCESSAIRES POUR LE TOEIC 3 CONSIGNES Ā APPLIQUER AVANT LE STAGE Révisez les temps présentés dans

141RÉPONSES TEST 2

Reading Section101. B102. A103. D104. D105. C106. B107. A108. A109. B110. B111. C112. D113. A114. A115. D116. A117. B118. C119. B120. A121. B122. D123. D124. A125. C126. D127. C128. C129. C130. D131. C132. C133. B134. D

135. C136. D137. A138. A139. D140. A141. B142. B143. A144. D145. C146. B147. D148. C149. A150. C151. B152. C153. C154. C155. B156. C157. A158. D159. B160. A161. B162. B163. D164. C165. D166. B167. D168. A

169. C170. B171. A172. D173. D174. C175. D176. B177. A178. C179. C180. A181. D182. A183. B184. C185. D186. C187. A188. B189. B190. D191. B192. A193. B194. A195. D196. D197. B198. C199. A200. A

Page 142: BASES ET RVISIONS NCESSAIRES POUR LE TOEIC · 2019. 12. 2. · BASES ET RVISIONS NCESSAIRES POUR LE TOEIC 3 CONSIGNES Ā APPLIQUER AVANT LE STAGE Révisez les temps présentés dans

142

TOEIC L&RPRACTICE TEST 1

Listening Section

Part 11. Answer (B)In the photograph, one can see people pointing at a document that looks like a graph or a chart, so statement (B) is an accurate assumption. The meal is clearly in progress, so statement (A) is wrong. There are no desserts in sight, so statement (C) is also wrong. Only one person is holding a glass and not drinking from it, so statement (D) is largely inexact.

2. Answer (C)In the photograph, once can observe the sea and a city next to it, so statement (C) is clearly correct. There are free parking spaces, the red car is parked, and no one is walking towards the building.

3. Answer (D)In the photograph, one can see a man’s hand holding a pen – either pointing at something in the document or writing on it. Statement (D) is therefore the best description. The man is not typing, the papers are on the com-puter, and the computer is screen is clearly on.

4. Answer (A)The man and the woman definitely look preoccupied, or concerned, so statement (A) is correct. They are not talking to each other, so statement (B) is not correct. While the computer can be seen in the photograph, the two people are not looking at it and are not touching any computer equipment, so statement (C) cannot be assumed as correct. The man and woman are clearly in the middle of work, and nothing suggests that they are taking a break – statement (D) cannot be right.

5. Answer (D)In the photograph, one can see no cruise ships (which are typically much larger than the yachts seen in the picture), people, or a single cloud. Therefore, statements (A), (B), and (C) are incorrect. In the background, one can see an unfinished skyscraper with a crane over it, which suggests that the construction is in progress and statement (D) is correct.

6. Answer (B)No one in the picture is either speaking on the phone or taking notes, so statements (A) and (C) cannot be cor-rect. One of the men is pointing at the computer screen with his pen and the other with his hand, so statement (B) is correct. Statement (D) is wrong because “adjourned” means “postponed” or “suspended,” while in the picture the meeting is clearly in progress.

Page 143: BASES ET RVISIONS NCESSAIRES POUR LE TOEIC · 2019. 12. 2. · BASES ET RVISIONS NCESSAIRES POUR LE TOEIC 3 CONSIGNES Ā APPLIQUER AVANT LE STAGE Révisez les temps présentés dans

143CORRECTION DÉTAILLÉE TEST 1

Part 27. Answer (C)Response (A) gives a general statement about the relationship between cost and quality but this statement cannot refer to parking since one doesn’t talk about the quality of parking. Response (B) explains where parking is avai-lable but not how much it costs. Response (C) says that the price depends on the length of time the car is parked.

8. Answer (B)Response (A) talks about a green folder, not a blue one. Response (C) describes the location of a person (“her”), not an object. Response (B) gives the folder’s last known location.

9. Answer (C)Response (A) talks about having a lawyer read the document but not where to sign the document. Response (B) gives a physical location of where the person responding to the question signs things but not where the spea-ker who actually asked the question should sign. Response (C) describes where in the document a signature is required.

10. Answer (A)Response (B) says when the documents were sent to the printing company but not who the printing company is. Response (C) gives a name but does so in relation to ordering items, not printing. Response (A) answers the question by indicating the usual company.

11. Answer (B)Response (A) talks about a deadline for submitting certain documents, not about their arrival. Response (C) talks about flights, not reports. Response (B) gives a time when the reports are expected.

12. Answer (A)Response (B) uses an idiomatic expression, “my hands are tied,” to express that someone cannot be of any help. Response (C) describes what the file was about but not whether or not it is readily available. Response (A) answers the question by explaining that the file is easy to get.

13. Answer (C)The question asks if the website is functional. Response (A) describes a website but does not say when the web-site might be available for viewing. Response (B) talks about exercising but not a website. Response (C) gives an answer about when the site will be ready: “in the next hour.”

14. Answer (A)Response (B) tells when the flight was scheduled to land but not whether or not it actually landed at that time. Response (C) describes the conditions of the flight but not its punctuality. Response (A) says that the flight arrived earlier than expected and so answers the question.

15. Answer (B)Response (A) talks about some person and uses a homophone of the word “weather”: “whether.” Response (C) describes a person’s favorite weather conditions but not what the forecast is that day. Response (B) describes the weather saying it will “drizzle,” or lightly rain, all day.

Page 144: BASES ET RVISIONS NCESSAIRES POUR LE TOEIC · 2019. 12. 2. · BASES ET RVISIONS NCESSAIRES POUR LE TOEIC 3 CONSIGNES Ā APPLIQUER AVANT LE STAGE Révisez les temps présentés dans

144

16. Answer (A)Response (B) talks about paying the bills not about receiving a paycheck. Response (C) indicates the method of payment, not the date. Response (A) specifies the days and is therefore the correct answer.

17. Answer (C)Response (A) talks about what a person prefers to do in the mornings, but not how that person is feeling. Res-ponse (B) talks about a past prediction but not about feeling in a physical sense, as the question demands. Res-ponse (C) describes how a person is physically doing.

18. Answer (B)Response (A) describes who is allowed to apply for the job but doesn’t say whether or not these people have actually done so. Response (C) talks about a passport application not a job application. Response (B) answers by giving the name of someone who sent in his resume to apply for the job.

19. Answer (A)Response (B) describes the location of the street but does not give its name. Response (C) refers to people’s names not street names. Response (A) gives the name of the street by saying that it is the same as the speaker’s last name.

20. Answer (C)Response (A) talks about a book, which cannot be heard. Response (B) refers to hearing a news report the pre-vious day. Response (C) says what the noise the people are currently hearing is.

21. Answer (B)Response (A) describes where one’s parents were from but not oneself. Response (C) explains where the person just was, but doesn’t answer the question, which asks where the person was born and raised. Response (B) says where the person was raised.

22. Answer (B)Response (A) says where the conference will be but not when. Response (C) gives a purpose of the conference but not when it is. Response (B) gives a suggested time of when the conference might be held.

23. Answer (A)Response (B) simply says that the person dislikes lateness, but does not provide a reason for being late. Response (C) asks a question about being early but doesn’t refer to the question at all. Response (A) gives a reason: the person had car trouble.

24. Answer (C)The question asks, basically, “Why do I have to deal with it?” Response (A) talks about a reservation but doesn’t answer the question. Response (B) simply says that the person’s office is in a nice location but doesn’t say how something came to be on the person’s desk. Response (C) indicates that whatever it is was placed on the desk because the person is leading the project it relates to.

25. Answer (B)Response (A) describes how to get to a place using public transportation but doesn’t answer the question. Response (C) explains that a person was reorganizing his office. Response (B) says who was in charge of the transportation.

26. Answer (C)Response (A) says that the year will be costly for some reason but doesn’t refer to the taxes at all. Response (B)

Page 145: BASES ET RVISIONS NCESSAIRES POUR LE TOEIC · 2019. 12. 2. · BASES ET RVISIONS NCESSAIRES POUR LE TOEIC 3 CONSIGNES Ā APPLIQUER AVANT LE STAGE Révisez les temps présentés dans

145CORRECTION DÉTAILLÉE TEST 1

offers an opinion about taxes but doesn’t say when they must be paid. Response (C) gives two months when the taxes are due.

27. Answer (A)Response (B) simply says the chair looks nice where it is located but doesn’t talk about the color. Response (C) lists the favorite colors of the speaker but doesn’t answer the question, which asks about one specific color – the color of the chair. Response (A) answers the question by saying that the color is too bright and therefore unsui-table.

28. Answer (B)The question asks whether the bill has been paid. Response (A) offers an opinion about the price but doesn’t answer the question. Response (C) uses the verbal phrase “to settle in,” which means to get comfortable. Response (B) indicates that the speaker is waiting for the credit card to be charged.

29. Answer (C)Response (A) refers to the hiring for the position not the actual position start date. Response (B) describes the location of the job. Response (C) explains when the person will begin work.

30. Answer (B)Response (A) says when the email was sent, not to whom. Response (C) talks about an email sent by someone else, not the speaker in the question. Response (B) lists whom the email was sent to.

31. Answer (A)Response (B) talks about where the person found, presumably, an article of clothing. Response (C) refers to an extra fee the person was charged. Response (A) explains the office the person works in and so is the best choice.

Part 332. Answer (B)The woman says it is on her calendar for tomorrow.

33. Answer (A)The woman says that the stock market is down. Then the man mentions that the stocks themselves are down and concludes that “it’s a good time to buy.”

34. Answer (D)The man requests that the woman email him the “new figures” for the accounts.

35. Answer (D)The man says he got held up a customs. This likely means he was in an international airport. Answer choices (A), (B), and (C) are unlikely to be correct, given the distance between Denmark and Canada.

36. Answer (C)The man says that he had salami (a kind of cured meat) in his luggage that the dog smelled and that was taken away from him (“confiscated”). The fact that the product was confiscated means it is prohibited for imports.

Page 146: BASES ET RVISIONS NCESSAIRES POUR LE TOEIC · 2019. 12. 2. · BASES ET RVISIONS NCESSAIRES POUR LE TOEIC 3 CONSIGNES Ā APPLIQUER AVANT LE STAGE Révisez les temps présentés dans

146

37. Answer (A)The man says he had brought her a “hostess gift,” which implies that he was going to stay at her house.

38. Answer (C)The man says that he can’t understand the bookkeeping. This means that he doesn’t understand the financial records he has been given.

39. Answer (B)The man says that the company is “in the red.” This is an idiomatic expression meaning that the company is losing money or owes a lot of money. It is not clear who the man and woman are in the company so answer choices (C) and (D) cannot be assumed as correct. The conversation only mentions three people, so the company’s size (answer choice (A)) cannot be determined.

40. Answer (D)The woman asks if she should have Linda come and explain the documents the man is looking at. Presumably Linda is a coworker who is somehow more familiar with the documents.

41. Answer (A)The woman begins the conversation by introducing herself and saying that she is calling from a doctor’s office. This is a clear indication that the conversation takes place over the phone.

42. Answer (B)The woman says she does not have the man’s emergency contact information and suggests faxing him the blank form to complete.

43. Answer (C)The man says he can bring the form back at his “upcoming” appointment. This indicates that he will see the doctor soon.

44. Answer (D)The woman says that the bank has been accused of “fraud.”

45. Answer (B)The woman says that shareholders are getting together to sue the bank – this is a paraphrase of “are organizing a group lawsuit.”

46. Answer (A)The man says that he thinks that many people will be “out of the job,” which indicates that he thinks they will be fired.

47. Answer (A)The woman says that she has to let Pedro go. To “let someone go” in a professional setting means to fire that person.

48. Answer (D)The woman says that Pedro is a “nice guy.” Among the answer choices, “kindness” is the closest synonym to “niceness.”

Page 147: BASES ET RVISIONS NCESSAIRES POUR LE TOEIC · 2019. 12. 2. · BASES ET RVISIONS NCESSAIRES POUR LE TOEIC 3 CONSIGNES Ā APPLIQUER AVANT LE STAGE Révisez les temps présentés dans

147CORRECTION DÉTAILLÉE TEST 1

49. Answer (B)The man says that Pedro is “just starting out,” which implies that he is young and new to a career.

50. Answer (A)The man asks the woman for a quote to design his website. We can infer that this is her specialty.

51. Answer (B)The man is quite concerned about the price of the work and has a strict price limit that he cannot exceed.

52. Answer (C)The woman charges $40 per hour and the man has a budget of $4,000. Therefore, he can afford 100 hours of work.

53. Answer (D)The woman talks about “candidates” and “the position,” which indicates that this is likely a job interview.

54. Answer (C)The woman says that there are five candidates of which the man is one. Therefore, she had already spoken to four candidates.

55. Answer (B)The woman says that the man will get a call from her “in the next day or two.”

56. Answer (A)The man says that he is having trouble reading the Departures board.

57. Answer (C)The man’s train leaves at 1:15 and arrives at 4:15. That is a travel time of 3 hours.

58. Answer (D)The woman tells the man his train is leaving from platform 30.

59. Answer (D)Emily starts by saying, “Good evening,” and adds, “I will be your server tonight.” This indicates that the clients are in a restaurant for an evening meal, which is dinner.

60. Answer (C)Emily states that the soup is not spicy at all, so choice (B) can’t be correct; conversely, she does mention that it contains several aromatic herbs, so choice (C) is extremely likely and therefore correct. Choice (A) is a trap playing with the word “palate,” but cannot be assumed as correct, given that no one has tasted the soup yet. Aller-gies (choice (D)) are not mentioned in the conversation at all.

61. Answer (B)The woman mentions “the octopus sashimi as a starter and then the roast duck,” which means that the duck will be her main course.

62. Answer (D)This conversation between the man and the woman is clearly an aside from the main talk the man is giving. Answer choice (A) would indicate that the man and woman are alone, which is clearly not the case. There is

Page 148: BASES ET RVISIONS NCESSAIRES POUR LE TOEIC · 2019. 12. 2. · BASES ET RVISIONS NCESSAIRES POUR LE TOEIC 3 CONSIGNES Ā APPLIQUER AVANT LE STAGE Révisez les temps présentés dans

148

nothing to suggest that this takes place at a university, so answer choice (C) is incorrect. The main topic of the conversation is not how to use a computer, which makes answer choice (B) unlikely. Answer choice (D), implying that the man is giving a presentation and the woman is assisting him, is the most probable context for this conversation.

63. Answer (C)The man specifies that the graphs are “on the desktop.”

64. Answer (D)The “Projects” part of the expenses in the graph occupies a little more than a half of the pie, so choice (D) is correct.

65. Answer (A)The man has brought a packet he needs to give to Mr. Yannatos. Therefore, he is a deliveryman. The woman welcomes him and offers to take the package, so she is most likely a receptionist. Answer choice (A) is correct.

66. Answer (C)The man states that this is “hand delivery,” meaning that he must give the package to Mr. Yannatos personally: answer choice (C) is correct. The size and the contents of the package (choices (A) and (D)) are never discussed. The woman doesn’t refuse to take the package – on the contrary, she offers to take it from the man, so choice (B) can’t be correct.

67. Answer (B)The woman says that Mr. Yannatos works in the Marketing Department, so the man needs to go to the 2nd floor, where this department is located.

68. Answer (B)The man says he “finally” has the tickets, and the woman states, “we’re so lucky.” It can, therefore, be assumed that the tickets were not easy to obtain, and answer choice (B) is correct. Nothing is said about the relative value of the show, so choice (A) cannot be assumed as correct. The man will clearly not perform in the show since he mentions “our” tickets and mentions he is a “patron.” According to the man, the show will take place “tomorrow,” not “next week.”

69. Answer (C)The price on the ticket is 100 euros. However, the man states that he gets a 20% discount off “face value,” i.e. of the declared price. Therefore, the woman owes the man 80 euros.

70. Answer (A)The woman says, “I will write you a check right away.” In this context, “right away” means “right now.”

Page 149: BASES ET RVISIONS NCESSAIRES POUR LE TOEIC · 2019. 12. 2. · BASES ET RVISIONS NCESSAIRES POUR LE TOEIC 3 CONSIGNES Ā APPLIQUER AVANT LE STAGE Révisez les temps présentés dans

149CORRECTION DÉTAILLÉE TEST 1

Part 471. Answer (D)According to the speaker, “People want to know the best way to safely grow their money… I think this is a com-mon concern today.” Therefore, answer choice (B) is the best possible answer. Earning money (A), children (B), and early retirement (C) are never discussed in the talk.

72. Answer (C)The woman affirms, “Interest rates on savings accounts are at an all-time low.” Answer choice (C) is, therefore, correct. While (A) might be true, it is not stated as such by the speaker.

73. Answer (D)The speaker concludes by saying, “What I always recommend is that people sit down together and map out a detailed plan.” Therefore, answer choice (D) is the best option. Choice (C) is in direct contradiction with what the woman says. Choice (A) is not mentioned, and choice (B) is a distortion of the information from the talk.

74. Answer (C)The announcement indicates that a cruise is beginning. A cruise takes place on a ship.

75. Answer (D)The announcement says that performers will be by the main pool. This indicates that they will be doing a perfor-mance. While drinks and snacks are mentioned, they will be served on the main deck, not at the main pool, so answer choices (B) and (C) are incorrect. People might be swimming at the pool, but that is not specified in the announcement, so answer choice (A) is incorrect.

76. Answer (D)The announcement says that the forward dining room stops serving dinner at 10 p.m. and that late-night dining is available in the bars, casino, and lounge. Answer (C) is incorrect because a minibar is a small fridge with drinks located in a hotel room. It is not the same as a bar with a bartender.

77. Answer (C)The announcement is promoting moisturizing hand soap. Lotions and creams are a minor detail in the talk, and shampoo is never mentioned.

78. Answer (A)The advertisement begins by saying that winter often makes people’s hands cracked and split because of the dry air. Information from other answer choices is never mentioned in the talk.

79. Answer (D)The talk indicates that the speaker is responsible for writing the advertisement. This indicates that the speaker works in the advertising department.

80. Answer (A)The speaker says that the handbook is thick and has a gray cover.

Page 150: BASES ET RVISIONS NCESSAIRES POUR LE TOEIC · 2019. 12. 2. · BASES ET RVISIONS NCESSAIRES POUR LE TOEIC 3 CONSIGNES Ā APPLIQUER AVANT LE STAGE Révisez les temps présentés dans

150

81. Answer (C)The handbook addresses “office-related” items. The speaker gives examples of getting a computer repaired, get-ting an ID replaced, and setting up voicemail. It is unlikely that the handbook covers social issues such as salary negotiation and dealing with difficult clients (answer choices (A) and (B)). It is also unlikely that the handbook deals with getting to work (answer choice (D)) since this is something that is different for each employee.

82. Answer (B)The speaker says that the handbook will save time for the employees because they can solve their own problems.

83. Answer (B)The speaker says that the Blue and the Green metro lines stop four blocks away. That makes it two in total.

84. Answer (A)The speaker says that many people go for runs or walks along the trail during their lunch breaks.

85. Answer (C)The speaker ends by saying, “in answer to your question – yes, the office is easy to get to without a car.” This implies that the original question was about the accessibility of the building without a car.

86. Answer (A)Since Daniela Fernandez is teaching a course called Global Business Strategy, it can be inferred that she likely specializes in international business.

87. Answer (B)The message says that a person should press 1 to hear a staff and professor directory. The directory will give the extensions for the employees of the school.

88. Answer (D)One of the options listed in the message is to hear a list of “school locations.” This implies that the school has multiple locations. While the message is offered in English and Spanish, this does not indicate how many stu-dents speak languages other than English (answer choice (B)).

89. Answer (A)It’s clear from the talk that the two people are having problems in the workplace. The speaker is in a position to offer suggestions to fix these problems. Answer choice (A) is the best option in this case.

90. Answer (D)The speaker says that Anita has mentioned that she is unsure of when projects are due.

91. Answer (B)The speaker suggests implementing a shared online calendar and database. This implies that technology (answer choice (B)) will help solve the issue.

92. Answer (D)The speaker is clearly leaving the company he works for. The speaker talks about “announcing my retirement” at the beginning of the talk.

Page 151: BASES ET RVISIONS NCESSAIRES POUR LE TOEIC · 2019. 12. 2. · BASES ET RVISIONS NCESSAIRES POUR LE TOEIC 3 CONSIGNES Ā APPLIQUER AVANT LE STAGE Révisez les temps présentés dans

151CORRECTION DÉTAILLÉE TEST 1

93. Answer (B)The speaker says that the experiences were all due to the “incredible people” the speaker has worked with.

94. Answer (C)The speaker refers to Ioana as “my assistant.”

95. Answer (A)According to the graphic, the height of the photograph should be between 5,5 and 6,5 cm, with the width between 3,5 and 4,5 cm. The only answer choice that does not correspond to this requirement is (A): the height of 5 cm is below the minimum stipulated. Therefore, it is safe to assume that choice (A) represents the dimen-sions of the photograph initially submitted by Ms. Heikkinen. All the other answer choices are sizes that fit the requirements.

96. Answer (B)The message says that there is a rush processing fee for applications submitted less than a week before travel. Therefore answer choices (C) and (D) are incorrect. Given that other items need to be sent to the embassy, it is highly unlikely that Ms. Heikkinen plans to travel the following day (answer choice (A)). Answer choice (B) is the best option.

97. Answer (A)The message states that Ms. Heikkinen is traveling for business. It also states that the letter from the company must say how long Ms. Heikkinen’s work will take. This indicates that she will not be relocating permanently to the country (answer choice (B)). Answer choice (A) is the best option.

98. Answer (C)The woman asks Sven to modify the prices on the website, so choice (C) is the most plausible. Note that Sven can’t be a salesperson, as he clearly has not participated in the Sales Team’s meeting.

99. Answer (B)The woman mentions “both of our boutiques,” which means that there are 2 of them.

100. Answer (C)The woman mentions that the prices of the Butterfly models should not change. Therefore, the correct answer is $60 – choice (C).

Page 152: BASES ET RVISIONS NCESSAIRES POUR LE TOEIC · 2019. 12. 2. · BASES ET RVISIONS NCESSAIRES POUR LE TOEIC 3 CONSIGNES Ā APPLIQUER AVANT LE STAGE Révisez les temps présentés dans

152

Reading SectionPart 5101. Answer (D)The verb “to mind” is always followed by the gerund.

102. Answer (A)This is a sentence in the present tense. Answer choice (B) would be correct if the sentence were in the past tense. Answer choice (C) is used for commands or suggestions, which this clause is not. Answer choice (D) is used for conditionals or future in the past, which is not appropriate for this sentence. Answer choice (A) is in the present and, therefore, the best answer.

103. Answer (C)The question clearly is about a specific meeting, known to both speakers, and so requires the definite article “the.”

104. Answer (B)The question requires an adverb to indicate the meaning of “already” in the present perfect tense. Only the adverb “yet” fits this context. “Therefore” indicates a causal relation; “after” indicates a temporal relation; “now” can only be used with the present simple or present progressive tense.

105. Answer (B)The primary verb in the phrase is “to be” so the tag must also contain the verb “to be.” Answer choices (C) and (D) are incorrect. The phrase is positive so the tag must be negative so answer choice (A) is incorrect. Answer choice (B) is the negative form that is used for “am” when it is contracted with “not.”

106. Answer (C)A conjunction that introduces an alternative is required. Answer choice (A) does not compare the two items so is incorrect. Answer choice (B) is used in negative sentences, which this is not. Answer choice (D) indicates a contradiction, which does not exist in this sentence. Answer choice (C) compares two items so is correct.

107. Answer (C)The sentence compares Dr. Matthews to the other doctors. Answer choice (A) is illogical, because “good” is already mentioned in the first clause, and the second clause is introduced by the conjunction “but,” which would suppose a contradiction. Answer choice (B) would be correct if there were only two doctors compared and if it were followed by “one.” Answer choice (D) is not a logical choice since the sentence implies that Dr. Matthews is superior rather than inferior to the other doctors. Answer choice (C), the superlative form, is the most logical and fits grammatically.

108. Answer (A)The sentence states something that will definitely happen in the future. “Shall” is used to talk about things that are expected to happen in the future, so it is the best option.

109. Answer (D)The sentence is in the present perfect tense because “checked” is a past participle. “Anyone” is the subject of the sentence and is the third person singular, so the correct answer choice must be (D).

Page 153: BASES ET RVISIONS NCESSAIRES POUR LE TOEIC · 2019. 12. 2. · BASES ET RVISIONS NCESSAIRES POUR LE TOEIC 3 CONSIGNES Ā APPLIQUER AVANT LE STAGE Révisez les temps présentés dans

153CORRECTION DÉTAILLÉE TEST 1

110. Answer (C)The question refers to location. The question word that is used for location or place is “where,” answer choice (C).

111. Answer (B)Answer choices (A), (C), and (D) do not make sense in the sentence since they cannot be followed by the pre-position “on.” Answer choice (B) means to ask a professional for his or her opinion, which is the best fit in this sentence.

112. Answer (D)The sentence indicates that the researchers began their work many years ago and continue their work today, thus requiring a verb in the present perfect progressive tense. Answer choice (D) is conjugated in that tense and so is the best option.

113. Answer (A)The question is trying to determine the amount of money the developer costs (“charges”). Answer choice (C) is used for frequency, and answer choice (D) for time, so neither is correct. Answer choice (B) is used for countable nouns, of which there are none in this sentence, so is incorrect. Answer choice (A) is used for uncountable nouns, such as money, so is the correct choice.

114. Answer (D)The sentence indicates that the person is looking for the fastest way to do something. Neither answer choice (A) nor (B) is a superlative, so they are not correct. “Quick” is a one-syllable word and so in the superlative form uses the ending “-est” rather than the word “most”. For this reason, answer choice (D) is correct.

115. Answer (A)The sentence indicates a contrast between what has been said and what the person thinks will happen. Answer choices (B) and (D) would indicate a positive correlation between the clauses. Answer choice (C) would indicate a conditional nature of the facts, which is not the case here. Answer (A) indicates a contradiction and is the best choice.

116. Answer (B)The question compares two types of banks. The two kinds of banks that exist are commercial and investment. Therefore, answer choice (B) is correct.

117. Answer (C)The sentence is in the conditional form and indicates that if something had happened in the past (the paying of taxes), the present situation (owing late fees) would not occur. The first clause needs a verb tense that indicates a completed action in the past so answer choice (C) is the best option.

118. Answer (A)Answer choices (B) and (C) are negative and so cannot be placed in a sentence with the negative subject of the sentence, “no one.” Answer choice (D) indicates a comparison, which does not exist. Answer choice (A) indicates a time and implies that people never remember to do something.

119. Answer (D)The main verb of the sentence is “to have” so “have” must be used in the tag as well. Since the sentence is negative, the tag question must be positive. Therefore, answer choice (B) is correct.

Page 154: BASES ET RVISIONS NCESSAIRES POUR LE TOEIC · 2019. 12. 2. · BASES ET RVISIONS NCESSAIRES POUR LE TOEIC 3 CONSIGNES Ā APPLIQUER AVANT LE STAGE Révisez les temps présentés dans

154

120. Answer (B)The sentence is a conditional structure implying an “impossible condition”: one person can never be another. The verb following “I” therefore must be conjugated in the past tense. Note that in the correct conjugation of the verb “to be” in all tenses of “impossible condition” is “were,” not “was.”

121. Answer (C)The sentence is in the passive voice and refers to something that happened at a specific moment in the past. Answer choice (C) is the best option because it is the simple past version of the passive voice. Answer choice (D) cannot be used in this case, as nothing in the phrase implies that the breakage took place before the power surge.

122. Answer (A)The question asks about something that will happen in the future: the adverb “soon” is an unambiguous indica-tor of that. Answer choice (A) is the only option that refers to the future. Other answer choices do not fit the context and are ungrammatical: the modal verbs (B) and (C) are not used this way to construct questions, and the auxiliary (D) is not used with “be.”

123. Answer (B)The blank requires an object pronoun since it follows a preposition. Answer (B) is the first person object pronoun.

124. Answer (C)The sentence indicates a completed action in the past. The past simple is usually used to describe this time period. Therefore answer choice (C) is correct.

125. Answer (D)The blank space requires a pronoun that would correspond to something that can be said. Answer choice (D) is the best: “what” refers to something. Answer choice (C) refers to a person, so is not logical in this case. “That” is either a demonstrative pronoun or a conjunction and cannot be used in this context; the same can be said about “which.”

126. Answer (B)Since the person doing the announcing is not known, the sentence is in the passive voice. Therefore answer choice (B) is the correct option.

127. Answer (A)A non-disclosure agreement is a legal document stating that the person signing will not talk about the company’s business or projects. While answer choices (B), (C), and (D) all have similar meanings to “agreement,” only answer choice (A) is used when talking about contracts and legal documents.

128. Answer (B)The sentence is contrasting two things (canceling a meeting and knowing that canceling the meeting is a bad idea). Answer choices (C) and (D) are not used in comparisons in this way. Answer choice (A) is used to indicate time or order but not comparisons. Answer choice (B) is used to compare two items.

129. Answer (C)The phrase “return on investment” is used to discuss the benefits a person receives for doing a particular thing.

130. Answer (D)A “pep talk” is a positive speech to help motivate someone. The sentence requires a reflexive pronoun so answer choice (D) is correct.

Page 155: BASES ET RVISIONS NCESSAIRES POUR LE TOEIC · 2019. 12. 2. · BASES ET RVISIONS NCESSAIRES POUR LE TOEIC 3 CONSIGNES Ā APPLIQUER AVANT LE STAGE Révisez les temps présentés dans

155CORRECTION DÉTAILLÉE TEST 1

Part 6131. Answer (A)Team-building events are organized for employees of the same company, i.e. colleagues. A synonym for “collea-gues” is “coworkers.” In the context of the message, it is highly unlikely that an event manager would recommend socializing with clients (B) or partners (D). While answer choice (C) cannot be excluded, option (A) is by far the best.

132. Answer (C)The subject-matter changes from the schedule (paragraph 1) to the nature of activities (paragraph 2), and the phrase immediately following the blank starts with “Each group.” This means that the correct phrase must men-tion a group or groups: only sentence (C) does that.

133. Answer (D)Almost the entire text is in the future tense, so the context of the phrase calls for a verb conjugated in the future tense as well, which excludes choices (A) and (B). Each group will receive, or, in other words, be given a task: the passive voice is obligatory here; the correct answer is (D).

134. Answer (D)Within the logic of the phrase, the response must be received before a certain deadline in order to have a gua-ranteed place. The only preposition fitting the context is “within,” meaning here “during the period of five days from now.”

135. Answer (C)The speaker mentions “the most versatile” substance, meaning that “coming across” took place in the past but its result is important for the present: the best option is the conjugation in the present perfect, option (C).

136. Answer (B)The word “whether” starting the sentence usually introduces a list of alternatives, and therefore the conjunction that best fills in the blank is “or.”

137. Answer (B)The phrase following the blank is, “However, it is also popular in candies and various foods.” The words “however” and “also” indicate that the sentence to insert must offer an alternative to a format in which green tea is a consu-med, contrasted with “candies and various foods.” Answer choice (B) is the best, as it presents a format in which green tea is most often consumed: a drink.

138. Answer (A)The article, in general, is full of admiration for green tea, and therefore the missing word should describe something very positive. Option (A) is the most positive, and fits the context stylistically better than (B), while the latter is also used to indicate positive effects, albeit in other contexts.

139. Answer (A)A building collapse can bring to the neighborhood a feeling of anxiety that other buildings might collapse as well. The closest synonym of “anxiety” is “concern,” answer choice (A). While the word “problem” might fit logically, it doesn’t fit grammatically because of the determiner “much” preceding the blank: “problem” is a countable noun and should be used with “many” and in the plural.

Page 156: BASES ET RVISIONS NCESSAIRES POUR LE TOEIC · 2019. 12. 2. · BASES ET RVISIONS NCESSAIRES POUR LE TOEIC 3 CONSIGNES Ā APPLIQUER AVANT LE STAGE Révisez les temps présentés dans

156

140. Answer (A)The phrase preceding the blank mentions the beams as the primary mistake in the construction. The pronoun “They” in answer choice (A) supports this logic and makes it the best option.Choice (B) does not fit logically: the age of the code makes the proof neither easier nor harder. In addition, the sentence following the blank says the construction company took full responsibility, meaning that the fault was quite evident and therefore not difficult to prove. Choice (C) is ungrammatical, as there is no antecedent for the pronoun “He” anywhere in the text. Choice (D) is extremely unlikely, as it simply paraphrases the previous sentence: it is therefore redundant.

141. Answer (D)The only action that can be done with a destroyed building is “restoring” it, answer choice (D). There is nothing to return, nor restitute, and a property cannot be “retired.”

142. Answer (B)The sentence is in the passive voice (“were inspected”); therefore, the verb in the blank should be a past parti-ciple: answer choices (A) and (D) can be eliminated. Comparing (B) and (C), it is important to remember that “found” is the past participle of “to find” (meaning “to discover”), whereas “founded” is the past participle of “to found” (meaning “to establish”). Choice (B) is clearly the best option: the buildings were inspected and found to be in perfect condition.

143. Answer (C)The blank calls for an adverb indicating the addition of one item to the next. While all of the answer choices indicate addition, choices (A) and (B) are usually placed at the beginning of a sentence and (D) at the end. Only choice (C), “also,” can be placed between the subject and the verb of a clause, as is the case here.

144. Answer (C)To choose the pronoun correctly, one must identify its antecedent. Nothing in the entire text refers to specific persons, so “him” and “her” can be excluded. In the phrase itself, the closest antecedent is “the world,” and so the pronoun should be “it.” “Them” can refer to a number of other nouns, but they are not close enough and “learning more” about the “team” or the “families” doesn’t make much sense in the context of this job ad.

145. Answer (B)This blank is between the verb “are” and the particle “to,” meaning that the word that fills it should be an adjective or a participle rather than a modal verb. The only adjective among the answer choices is “unable.”

146. Answer (D)The text is clearly a want ad, and therefore it should logically be concluded with an invitation to apply for the position advertised. Answer choice (D) is the best.Choices (A) and (C) do not make much sense in the context, as the text is neither about helping everyone nor about changing one’s own life. Choice (B) shifts the focus too much: discovering the world is not the main idea of the text, and it would be extravagant to qualify the described experience as wonderful.

Page 157: BASES ET RVISIONS NCESSAIRES POUR LE TOEIC · 2019. 12. 2. · BASES ET RVISIONS NCESSAIRES POUR LE TOEIC 3 CONSIGNES Ā APPLIQUER AVANT LE STAGE Révisez les temps présentés dans

157CORRECTION DÉTAILLÉE TEST 1

Part 7147. Answer (B)In the second paragraph of the email, Mr. Ibarrola clearly asks Ms. Laval for advice as for what to do in the situation: answer choice (B) is the best. While he obviously is describing a problem, the tone of the email does not make one think that he is complaining about it, so choice (A) cannot be correct.

148. Answer (B)The email states, “As per your instructions…”, which means that Ms. Laval had provided the installation instruc-tions to Mr. Ibarrola – answer choice (B) is clearly correct. Given the nature of the questions in the second para-graph of the email, Mr. Ibarrola is unsure whether Ms. Laval can help him, so choice (A) cannot be considered right. Choice (D) is too far-fetched to be assumed as unambiguously correct.

149. Answer (A)Abdel’s phrase “Actually it is” is the reply to Radek’s question “Is it urgent?” Therefore, option (A) is the best answer choice: yes, the matter is urgent for Abdel.

150. Answer (C)In his 8:16 message, Radek says that Cyril authorized his leave. It is, therefore, logical to assume that Cyril is the supervisor of the security guards team. Note that Abdel cannot be the supervisor, as he acknowledges that he has not been informed about the absence of a member of the team.

151. Answer (C)The notice mentions that bus and metro services will be closed from Monday morning to Wednesday morning. It is therefore safe to assume that the storm will last for about 48 hours: two days.

152. Answer (D)The notice says residents should only go outside in emergency situations. The only option that could be conside-red an emergency would be a trip to the hospital.

153. Answer (A)The form is a questionnaire with the goal of measuring customers’ satisfaction. Therefore, answer choice (A) is correct.

154. Answer (D)The answer to the last question on the form mentions that customer service representatives are not always avai-lable on the phone. Answer choice (D) accurately paraphrases this complaint. The range of services (A) is not discussed in the form, and options (B) and (C) represent the aspects that Hsiu Lang is generally satisfied with.

155. Answer (C)The email says that the company has just won a major contract and that they have begun working on the new project.

156. Answer (A)The email refers to researching old marketing campaigns to assess what was missing and use this information to develop new marketing campaigns. This implies that the company is in the marketing business.

Page 158: BASES ET RVISIONS NCESSAIRES POUR LE TOEIC · 2019. 12. 2. · BASES ET RVISIONS NCESSAIRES POUR LE TOEIC 3 CONSIGNES Ā APPLIQUER AVANT LE STAGE Révisez les temps présentés dans

158

157. Answer (D)Louis ends his email by saying that Guillermo is missed around the office and that there is always a job for him if he wants to return. This implies that Guillermo used to work at Louis’s company.

158. Answer (B)The memo says that price are “closer” to what people deserve to get for their homes but that it is still a “buyer’s market.” This implies that home prices are lower than what the house is actually worth.

159. Answer (A)The email says, “The banks are starting to give loans once more,” which allows people to have the money to buy a new home.

160. Answer (C)The memo is addressed to the “team” and later thanks the “talented realtors” who have made the year’s accom-plishments possible.

161. Answer (C)The announcement states that the internship is available for scientists and researchers. This indicates that the type of research done at the company is scientific in nature.

162. Answer (A)The announcement says that there are 5 positions open. 15 people are called in for interviews, but of those 15, only 5 are selected.

163. Answer (C)The phrase to insert refers to a part of the application package. Its best position is, therefore, No. [3], immediately after the sentence, “When applying, be sure to…”

164. Answer (A)The notice starts by mentioning the “handbook that was given out at your orientation.” It can therefore be assu-med that the notice is sent out to all those who received orientation already. Answer choices (B) and (D) cannot be inferred from the text. Answer choice (C) is incorrect, as the notice mentions that only [some] part-time employees have been awarded certain healthcare benefits.

165. Answer (B)The notice says that replacement handbooks can be obtained from HR.

166. Answer (C)The memo says that, previously, some part-time employees have been able to negotiate certain healthcare benefits.

167. Answer (D)The notice says that most answers are available in the Frequently Asked Questions section of the handbook and that employees should look there before asking HR.

168. Answer (A)The flight will depart Lisbon at 9:15: “departure” is a synonym for “take-off.”

Page 159: BASES ET RVISIONS NCESSAIRES POUR LE TOEIC · 2019. 12. 2. · BASES ET RVISIONS NCESSAIRES POUR LE TOEIC 3 CONSIGNES Ā APPLIQUER AVANT LE STAGE Révisez les temps présentés dans

159CORRECTION DÉTAILLÉE TEST 1

169. Answer (B)The email says that seats can be changed “by visiting our website.”

170. Answer (A)The email says that the first bag is free, the second is 25€, and the third is an additional 50€. This means that three bags cost 75€.

171. Answer (D)The phrase to insert talks about hand luggage. Positions [1] and [2] are clearly unsuitable, as the sentences around them talk about the seat choices. Position [3] is located between two sentences dedicated to checked luggage, not hand luggage. This leaves position [4] as the only possible choice.

172. Answer (B)The word “constraint” means “limitation” or “impossibility.” Therefore, the first question that Ms. Kwambo asks is when the moving would not be possible. Answer choice (B) is the best. Answer choice (C) is inexact, as there might be events not really interfering with the moving.

173. Answer (B)The inventory is mentioned in Ying Kong’s message at 15:04. She says, “We will need two days […] right before the holidays.” “Holidays” are mentioned in Yves Kerlouann’s message at 15:02 as starting on December 20. The-refore, the inventory has to be done on December 18 and 19.

174. Answer (D)Mr. Zimmerman says at 15:05, “we’ll have to finalize the shipments to overseas clients.” It is therefore safe to assume that he manages the shipping department or is a member of that department.

175. Answer (C)Mr. Fitzpatrick replies to a question about “sales orders” and later talks about “hot prospects.” It is therefore clear that he deals with sales in the company. Note that his role as CEO is unlikely, since it is Ms. Kwambo who leads and manages the conversation.

176. Answer (A)The letter says that the company hires financial planners, instructional designers, and analysts.

177. Answer (C)The letter says that the teams are made up of new graduates and experienced professionals. This indicates that the team members have different levels of experience. They are also made up of people from different fields. Therefore, the teams can be described as “diverse.”

178. Answer (D)The letter says that the company provides a gym, recreational room, and lunches on Friday. It makes no mention of health insurance.

179. Answer (B)The letter says that the company is trying to hire on to teams in industries where the company is growing its presence. According to the chart, the Government/NGO sector has been growing steadily each year. Therefore, it is likely that this is the area where the company would like to hire more people.

Page 160: BASES ET RVISIONS NCESSAIRES POUR LE TOEIC · 2019. 12. 2. · BASES ET RVISIONS NCESSAIRES POUR LE TOEIC 3 CONSIGNES Ā APPLIQUER AVANT LE STAGE Révisez les temps présentés dans

160

180. Answer (C)According to the chart, the company has the most contracts in the Technology sector.

181. Answer (B)The regulator is concerned that the merger will create a monopoly. A “monopoly” is a situation where one com-pany has control over a particular market or product. Answer choice (D), while plausible, is not mentioned as the primary concern in the correspondence, especially in the light of its overly affirmative nature and the word “extremely.”

182. Answer (A)Given that Mr. Jacobs is writing to inform Ms. Alvarez about the government investigation and that he uses the term “our investigators,” it is likely that he works for a government office that is in charge of regulating companies.

183. Answer (A)Ms. Alvarez says in her email that she has had his lawyers work with the legal team from the other company to see if a monopoly would occur.

184. Answer (D)Neither letter makes any mention of hardware production: the companies produce software only, and so (D) is the correct answer.

185. Answer (B)In his email, Mr. Jacobs says that the investigation will take less time if both companies cooperate. As Ms. Alvarez is clearly interested in the merger to go through as soon as possible, she is ready to cooperate with his investigation.

186. Answer (B)Debra says in her email that she has already left for the conference so it can be inferred that she is no longer at the office and can’t take care of the supplies.

187. Answer (A)Debra tells Amanda to first check the supply closet for the items before going to another department or a store.

188. Answer (D)Debra says in her email that the company is tax-exempt and asks Amanda to make sure the store will “remove the tax from the grand total of the purchase.” This means that the company does not have to pay taxes like sales tax on a purchase.

189. Answer (C)Mr. Gomes says that double the number of extension cords should be brought. This means that Debra and Amanda will need to bring 8 extension cords instead of 4 currently on the list.

190. Answer (C)Given that there are 75 giveaway bags and 75 pads of paper, as well as 75 USB drives, it is logical to conclude that there will be 75 participants. Pens and flyers are always provided in larger quantities than the numbers planned, and microphones are not necessarily given to all participants.

Page 161: BASES ET RVISIONS NCESSAIRES POUR LE TOEIC · 2019. 12. 2. · BASES ET RVISIONS NCESSAIRES POUR LE TOEIC 3 CONSIGNES Ā APPLIQUER AVANT LE STAGE Révisez les temps présentés dans

161CORRECTION DÉTAILLÉE TEST 1

191. Answer (B)Sanjay says that he has lived his entire life in Los Angeles prior to London. This indicates that he grew up there.

192. Answer (D)Sanjay says that the weather in Seattle is similar to the weather in London – overcast and drizzly. “Drizzle” is another word for light rain.

193. Answer (D)Sanjay says that it takes about 2 hours to get from the plane to the train station after clearing immigration and customs. Since Marianne arrives at 7:45 a.m., she will likely arrive at the train platform at 9:45 a.m. According to the schedule, the next train is at 10:00 a.m., so it is logical that she was supposed take this one.

194. Answer (B)Sanjay says that he will meet Marianne at the train station, after which they will walk together to the office.

195. Answer (D)In her 9:45 a.m. text message, Marianne says that she will be “only half an hour late,” implying that she will take the 10:30 a.m. train and therefore arrive in London at 10:50 a.m.

196. Answer (C)The newsletter says that Natasha was a managing director in the credit risk department.

197. Answer (A)The bank’s press release says that Natasha has “risen through the ranks.” This implies that she has been promoted several times and has held ever more powerful positions.

198. Answer (D)The press release states that Natasha has an “innate understanding for managing risk.” Something that is “innate” is something a person is born with or that comes naturally to a person.

199. Answer (A)Giulia’s email says that Natasha is 40 years old, while the press release indicates that she has worked for the bank for 6 years. This means that she was 34 when she started working there.

200. Answer (D)While all the answer choices are probable, the question asked is about the main purpose of Giulia’s email. As stated in the Subject of the email, the main purpose is to obtain an interview with Natasha.

Page 162: BASES ET RVISIONS NCESSAIRES POUR LE TOEIC · 2019. 12. 2. · BASES ET RVISIONS NCESSAIRES POUR LE TOEIC 3 CONSIGNES Ā APPLIQUER AVANT LE STAGE Révisez les temps présentés dans

162

TOEIC L&RPRACTICE TEST

Listening Section

Part 1190. Answer (C)1. Answer (C)In the picture, you can see coffee with dessert served for two, but no people: answer choices (A) and (D) are incorrect. The forks are next to the plates with cakes, not to the cups, so answer choice (B) is also wrong. Answer (C) is correct: each of the two cups is on a saucer.

2. Answer (C)In the photograph, one can see a man and a woman in the back of a car looking at the screen of a laptop: answer choices (A) and (B) are incorrect, and choice (C) is correct. It’s the man, not the woman, who is wearing sun-glasses, so choice (D) is wrong.

3. Answer (D)The main element of the photograph is a clock tower, which is indeed decorated with banners, so answer choice (D) is the best. All the other statements contradict the picture: no woman is crossing the street, no parade in sight, and the second tower is clearly on the other side of the road.

4. Answer (B)In the photograph, one can see neither a man nor a microscope: answer choices (C) and (D) can be excluded. A doctor is examining a patient’s eyes with a special machine, so answer choice (B) is correct, while statement (A) is wrong.

5. Answer (A)In the photograph, one can see a swimming stadium with two pools, but only one of them seems open to the public, so answer choices (B) and (D) are wrong. All the seats in the stadium are empty: there are no spectators, meaning choice (C) is also wrong. The only correct statement is the most general one – answer (A): the weather is indeed sunny, and there are people bathing in one of the pools.

6. Answer (C)In the photograph, one can see a man standing and bringing his pass (or ticket) to a validating machine, while on board of a bus. This means that answer choice (C) is correct, whereas all the other choices contradict the scene.

Page 163: BASES ET RVISIONS NCESSAIRES POUR LE TOEIC · 2019. 12. 2. · BASES ET RVISIONS NCESSAIRES POUR LE TOEIC 3 CONSIGNES Ā APPLIQUER AVANT LE STAGE Révisez les temps présentés dans

163CORRECTION DÉTAILLÉE TEST 2

Part 27. Answer (B)The question (“how”) calls for a response describing a method. Response (C) does not suggest any, and is there-fore incorrect. Answer choices (A) and (B) both offer a method, but it’s highly unlikely that a password can be changed by sending an email, especially given the word “instead,” so choice (A) can be eliminated. Response (B) is correct: the man needs to talk to the IT team in order to have his password changed.

8. Answer (A)The question (“how many”) calls for a number in the response. Therefore, choice (B) can be eliminated. Response (C) talks about the number of questionnaires (“print them”), not questions in a questionnaire, and is therefore wrong. Response (A) is the only correct one: not more than 10 questions.

9. Answer (C)Response (A) talks about staples, not staplers. Response (B) indicates the location of the supply closet, without answering the question. Answer choice (C) is correct: the woman believes there are some staplers in the closet since she saw them there recently.

10. Answer (C)Responses (A) and (B) are out of the context: they talk about the product and the campaign, respectively, rather than the scandal, which is the central point of the question. Answer choice (C) is correct: the speaker believes that the scandal has actually been a good thing.

11. Answer (B)Response (A) is incorrect because the choice “dark gray” is not offered. Response (C) asks for options, whereas they are clearly outlined in the question. Choice (B) is the only answer that fits the context: neither dark red nor light gray.

12. Answer (A)In response (A) the woman agrees to call back right away (“in a second”) on a different line – this is the correct answer. Response (B) is contradictory: in the first part the woman agrees to call again, but in the second asks the man to call back instead. Answer choice (C) is wrong, as it answers a question about the purpose of the call, not its method.

13. Answer (B)Response (A) is incorrect, because, while theoretically going to Sydney every year can help with the logistics, this is not a direct response to the question asked. Response (C) talks about the guest speaker, not the logistics coordinator. Answer choice (A) is correct, as it provides an unambiguous answer: Leonidas and his team.

14. Answer (C)The question is about a location: where. Response (C) provides a location and is therefore correct. Response (A) talks about a timespan, and response (B), about a person.

15. Answer (B)Response (A) talks about seeing rather than hearing something, and so it is wrong. Responses to a want ad (res-ponse (C)) are highly unlikely to cause any strange noise. Answer choice (B) is the only plausible explanation for the noise: it’s Mrs. Zhang’s dog who is doing something.

Page 164: BASES ET RVISIONS NCESSAIRES POUR LE TOEIC · 2019. 12. 2. · BASES ET RVISIONS NCESSAIRES POUR LE TOEIC 3 CONSIGNES Ā APPLIQUER AVANT LE STAGE Révisez les temps présentés dans

164

16. Answer (C)Response (A) is contradictory: the first speaker is clearly impatient, while the second suggests waiting more while stating “I agree.” Response (B) talks about the past while the question is clearly about the future. Response (C) is correct, as it provides a deadline: the end of this week.

17. Answer (B)“Would care for” is a synonym for “would like.” In this sense, answer choice (C) is incorrect, since it misuses the word from the question. Answer (A) is nonsensical, as a shrimp cocktail is an appetizer, not a dessert. Response (B) is a polite way of refusing the offer (some dessert) and therefore the correct answer.

18. Answer (B)The question asks for a description of a person. Answer choice (A) speaks about the building and (C) about the hallway. Only response (B) fits the context: the woman doesn’t remember much and therefore can’t provide a description.

19. Answer (A)The question is about the necessity or obligation of driving a car in order to get to a certain point. Response (A) establishes that there is no such obligation, as other options are available. Response (B) is talking about driver’s license, not driving per se. Response (C) uses the word “have” in its primary meaning “possess” and does not talk about an obligation.

20. Answer (B)The woman is being sarcastic in response (B), but this is the only answer choice that provides an indication of time, as the question asks “when.” The two remaining responses are general statements that do not mention any timeframe.

21. Answer (C)Response (C) provides information about Mr. Severski previous job and gives an answer to the question: no. Response (A) is a trap misusing the word “judge.” Response (B) is nonsensical, as it talks about the memory of one’s previous job rather than the fact of it as such.

22. Answer (B)The question starts with “Which department” – therefore, the correct answer should contain a name or an indi-cation of a department. The only response to do this is answer choice (B): Cybersecurity.

23. Answer (A)Speakers are discussing the mode of transportation for arriving to work in the morning. “Tube” is a synonym for “metro” or “underground.” The only answer choice indicating transportation is (A): a scooter.

24. Answer (C)The question asks for a reason. The only reason for an absence can be response (C): a business lunch. Answer choices (A) and (B) are not reasons as such.

25. Answer (B)The conversation is clearly taking place in a hotel, between a female guest and a male receptionist. The woman would like to know by which time she must leave the hotel. Response (B) provides a clear answer: before noon. Response (A) doesn’t make sense unless it is a rude joke, which is unimaginable in the context. Response (C) confuses the word “check” for a payment method.

Page 165: BASES ET RVISIONS NCESSAIRES POUR LE TOEIC · 2019. 12. 2. · BASES ET RVISIONS NCESSAIRES POUR LE TOEIC 3 CONSIGNES Ā APPLIQUER AVANT LE STAGE Révisez les temps présentés dans

165CORRECTION DÉTAILLÉE TEST 2

26. Answer (A)Response (B) is clearly incorrect, as the question is about a number, and (B) doesn’t provide any. Response (C) mentions a number, but can’t be logical, since CVs are not counted in halves. Response (A) is correct: the job opening is very popular, and several hundred CVs have been received. The speakers will have a lot of work sorting through them.

27. Answer (C)Response (A) does not answer the question, as it talks about Etienne’s departure, not his sales results. Response (B) doesn’t now allow to infer whether Etienne is good at sales or not. Response (C) is correct: the woman not only confirms but also admires Etienne’s sales skills.

28. Answer (A)The man asks in which currency or currencies a bank account can be nominated. Response (A) specifies that in the case of a checking account (most probable account type), two options are available: euros and Danish Krones. Answer choice (B) also mentions currencies but fails to address the other important point of the question: the account. Response (C) mentions the account but not the currencies.

29. Answer (A)The question asks about a payment method. None of the responses indicates a method, but only choice (A) gives a plausible explanation for it: Stu didn’t pay at all; Rebecca paid instead of him.

30. Answer (C)Response (A) can be tempting, but in fact, it doesn’t provide an unambiguous answer. For instance, if there had been a huge loss in the first quarter, a modest loss in the second quarter could still be considered “a lot better.” Response (B) does not provide a result. Choice (C) is the only plausible response: it should be clear to both speakers that the answer is “loss,” due to heavy fixed costs.

31. Answer (C)The question (“who?”) requires a response indicating a person. Only answer choice (C) provides that: the man mentions a woman with whom he is not acquainted. Responses (A) and (B) are general statements not indicating any person specifically.

Part 332. Answer (D)The two speakers are discussing who will go to their next convention: answer choice (D) is therefore correct. The paperwork (choice (B)) is a minor detail. The sales team is not on vacation (and won’t be), only Adam is, so answer choice (A) cannot be correct. The speakers are not discussing the organizational aspects of the convention, so choice (C) cannot be assumed as correct.

33. Answer (D)The man suggests John, Adam, and Taylor. As Adam cannot go due to his vacation, Paul will replace him. The-refore, the team will consist of John, Paul, and Taylor: answer choice (D).

34. When suggesting Paul instead of Adam, the woman says, “Paul has been greatly improving, so he would be a good substitute,” which means that Paul has made good progress lately – answer choice (C) is correct.

Page 166: BASES ET RVISIONS NCESSAIRES POUR LE TOEIC · 2019. 12. 2. · BASES ET RVISIONS NCESSAIRES POUR LE TOEIC 3 CONSIGNES Ā APPLIQUER AVANT LE STAGE Révisez les temps présentés dans

166

35. Answer (C)Answering the man’s question, “Should I place it online or in the newspaper?” the woman says, “Both would be better.” Therefore, it can be assumed that the man will follow her advice/order and place the advertisement both online and in a newspaper.

36. Answer (B)The man has already placed the ads for cooks and cleaners, so answer choices (A) and (C) are wrong. He has yet to place the ad for “the upkeep of the grounds,” which is a synonym for “maintenance” – therefore, answer (B) is correct. Builders (choice (D)) are never mentioned in the conversation.

37. Answer (C)The man asks the woman for advice or opinion in the first line of the dialog, whereas the woman says, “before I make any decisions” in the last line; therefore, answer choice (A) has no grounds. Choice (C), on the contrary, is quite plausible. Nothing is said about the woman’s dislike of advertising, so answer choice (B) cannot be correct. Finally, the woman doesn’t sound particularly “alarmed,” and not even concerned, by the lack of personnel: they just need to hire more people, and that’s it; for this reason, answer choice (D) cannot be considered correct either.

38. Answer (A)The woman says that she can’t wait to see Courtney in their department. “Can’t wait” is a synonym for “look forward to,” so answer choice (A) is correct. Later, the woman says that Courtney needs to be trained first, which contradicts answer choice (B). The woman says nothing about Courtney’s speed or ability to learn (C) and doesn’t provide any emotional judgment about her personality (D).

39. Answer (B)Answer choices (A) and (D) are plausible but too extreme to be true: the department is not exceptionally unders-taffed, and even if the man would like to see the work “better distributed,” it doesn’t automatically mean that the work is badly organized. While the speakers will have to train Courtney, nothing really makes one think that this is specifically what the department deals with, so choice (C) is incorrect. Finally, as the man mentions “five insurance cases” and believes that others could do this work too, it would be safe to assume that the department deals, at least partially, with insurance.

40. Answer (A)The only thing that the woman warns the man about is, “it may be more work than it’s worth at first,” which is correctly paraphrased in answer choice (A). The man will clearly benefit from Courtney presence, albeit not right away, so choice (B) is incorrect. The woman never says that Courtney is a slow learner or that the work will be poorly distributed, so choices (C) and (D) are also wrong.

41. Answer (D)The man clearly comes to the woman for advice in wealth management, so answer choice (D) is correct. Employ-ment (answer choice (B)) is never discussed in the conversation. Choices (A) and (C) are probable but not at all supported by the dialog.

42. Answer (C)The woman states clearly, “I will start by checking if your portfolio is well-balanced in terms of stock and bonds” – answer choice (C) says exactly that. The other answer choices distort the information from the conversation.

43. Answer (B)The man says, “I would be willing to invest a small percentage into higher-risk securities,” which means that (B) is the best answer choice. All the other answer choices are too extreme and contradict the man’s words regarding high-risk investments.

Page 167: BASES ET RVISIONS NCESSAIRES POUR LE TOEIC · 2019. 12. 2. · BASES ET RVISIONS NCESSAIRES POUR LE TOEIC 3 CONSIGNES Ā APPLIQUER AVANT LE STAGE Révisez les temps présentés dans

167CORRECTION DÉTAILLÉE TEST 2

44. Answer (B)The meeting is “scheduled for 12 p.m.,” which is noon.

45. Answer (A)The man says, “I have to meet with a client then, and I can’t miss that.” This means that choice (A) is the correct answer. At the moment of the dialog, the man doesn’t know yet if the meeting called by the boss is important or not, and, in any case, he does not express his opinion regarding that meeting’s importance, so answer choice (B) cannot be assumed as correct. Choice (C) simply distorts the information from the conversation. As for choice (D), the man has indeed been away all morning, but this is not a reason for his being unable to attend the mee-ting: it’s a reason for not having received the memo about the meeting.

46. Answer (C)The woman clearly doesn’t consider the meeting important either for the man or for herself: “Just between us, it’s really only to tell us about the equipment upgrades.” That makes answer choice (C) correct and choices (A) and (B) incorrect. Nothing in the dialog is said about the woman’s expectations of the boss’s feelings in case of their absence, so choice (D) cannot be considered right.

47. Answer (C)The woman says, “I need to find the terminal for my next flight,” and the man later confirms, “You’re going to be looking for terminal 3A.”

48. Answer (D)The man says, “You’re going to be looking for terminal 3A, which is just at the end of the southeast corridor behind you,” meaning that the woman needs to turn around.

49. Answer (B)The man says, “you do have 2 hours until your flight,” so clearly the woman needn’t hurry: answer choice (D) is incorrect. The man never suggests that the woman accompany him: choice (A) is also wrong. On the other hand, he mentions “many shops and restaurants,” implying that the woman can do some shopping or have a meal, so answer choice (B) is the best answer. “Browsing the Internet” is never mentioned in the conversation and is only a distortion of the man’s suggestion to browse shops and restaurants.

50. Answer (C)Sophia spells her name as W-H-I-T-E-L-Y.

51. Answer (A)Giovanni and Sophia are discussing the hiring manager, whom Sophia is trying to reach. Giovanni says, “Our hiring manager isn’t working today,” meaning that the latter is absent from the office. Nothing is said about whether the hiring manager takes calls from applicants, so choices (B) and (C) cannot be assumed as correct. Email is never mentioned in the dialog; on the other hand, Giovanni suggests leaving a message with him, so choice (D) is clearly incorrect.

52. Answer (D)Sophia mentions her “application for the assistant manager position,” which is option (D).

53. Answer (C)The man states at the beginning of the conversation that the reason for rescheduling is, “Half of our team will not be able to make it.” In other words, half of the team supposed to be at the meeting will be unavailable: answer choice (C). The clients are not the reason for rescheduling, so choices (A) and (B) are wrong, and the meeting room (D) comes up only later in the conversation.

Page 168: BASES ET RVISIONS NCESSAIRES POUR LE TOEIC · 2019. 12. 2. · BASES ET RVISIONS NCESSAIRES POUR LE TOEIC 3 CONSIGNES Ā APPLIQUER AVANT LE STAGE Révisez les temps présentés dans

168

54. Answer (A)The man suggests/wonders “if they would like to meet here at 4 p.m.,” and before that, the woman says the client called and suggested meeting today. Answer choice (A) is therefore correct. Answer choice (D) indicates the original meeting time. The other two answer choices simply distort that information.

55. Answer (B)Answer choices (C) and (D) are nonsensical, as the client has already made a suggestion for the meeting, and all the man and woman need to do is to confirm it – what the question asks about. Cleaning the meeting room is never mentioned in the dialog. This leaves only answer choice (B) as correct: before confirming the new time with the client, the speakers should make sure their own team will be available at that time.

56. Answer (A)The man says that his employee ID is 9384.

57. Answer (D)The man says, “I … never got a check for my unused vacation days,” and this is the problem with which he comes to see the woman. Therefore, choice (D) is the correct answer. While the man has been “terminated,” according to the conversation, he never mentions it as a problem, so answer choice (A) is wrong. The same logic applies to choice (C). As for choice (B), the man says, “I came with questions about my last paycheck,” meaning that he did receive his paycheck (and therefore the salary), except that it didn’t contain the amount due for the unused vacation.

58. Answer (C)The woman assures the man that there is basically no problem here: she just needs to print the check, and the man will get what he has come for. So, answer choice (C) is correct. All the other answer choices distort minor details from the dialog.

59. Answer (A)The three speakers do not need to introduce themselves to each other and are discussing a subject that is clearly familiar to the three of them; therefore, it can be assumed that they’ve met before – perhaps many times. Answer choice (A) is the best option. Answer choice (B) can’t be correct, as Ms. Tanaka is a real estate agent, and it’s unlikely that Mr. Dubois would work in real estate as well yet use her services. Nothing in the conversation sug-gests the two women are friends: choice (C), while theoretically can’t be excluded, finds no confirmation in the conversation. The same can be said about choice (D): Mr. and Mrs. Dubois clearly have children, but nothing is said about Ms. Tanaka in this aspect.

60. Answer (B)Comparing the two apartments in question, Mr. and Mrs. Dubois mention that Russell Gardens apartment is closer to Mr. Dubois’s work (so answer choice (A) is wrong) and that it is smaller than the Overfield Drive one, making answer choice (B) right. Apparently, there is a park near the Russell Gardens apartment – and therefore it is clear that there isn’t any near the Overfield Drive apartment, so choice (C) is wrong. For the same reason, Mrs. Dubois mentions that the Russell Gardens apartment would be better for her children: answer choice (D) is also wrong.

61. Answer (A)When Ms. Tanaka suggests that the Dubois fill in the forms, Mr. Dubois asks if they can do it “now.” Therefore, answer choice (A) is correct. They are not going to the office, it is up to Ms. Tanaka to contact the owner, and the decision about the apartment is already made: choices (B), (C), and (D) are incorrect.

Page 169: BASES ET RVISIONS NCESSAIRES POUR LE TOEIC · 2019. 12. 2. · BASES ET RVISIONS NCESSAIRES POUR LE TOEIC 3 CONSIGNES Ā APPLIQUER AVANT LE STAGE Révisez les temps présentés dans

169CORRECTION DÉTAILLÉE TEST 2

62. Answer (A)The applications and processor are mentioned by the woman, not the man, so choices (B) and (C) are not confir-med in the dialog. Choice (D) contradicts the man’s words, as he mentions touchscreen as a feature of a computer he would recommend. Which leaves (A) as the only correct answer choice: the man says that his computer “works well for travel.”

63. Answer (B)The woman mentions that applications are unimportant, so choice (A) is wrong, but good memory is, so choice (B) is right. Touchscreen and presentations are mentioned by the man and not specifically commented on by the woman.

64. Answer (D)At the end of the conversation with the woman, the man highly recommends the Steel 300 model for her, saying, “[It] has the best design for what you want to do.” It is, therefore, safe to assume that the woman will buy this model. In the price list, Steel 300 is at $499 – answer choice (D).

65. Answer (D)While explaining the procedure in place, the woman asks the man to bring the form to her and to “pay here.” In other words, she processes the payments for the orders: she is at the cash register.

66. Answer (B)The woman says, “You take one of the order forms at the entrance.” In this context, “pick up” is a synonym of “take.”

67. Answer (A)The man says that he is allergic to dairy products, i.e. to products with a milk base. The only listed dairy ingredient is “sour cream” in the Fast and Fresh sandwich. The man can’t eat it, or he would need to request on the form not to add sour cream if he wanted to order this sandwich. Therefore, answer choice (A) is correct.

68. Answer (D)The woman believed that her Stretching class was about to start, but the man says that the class has been moved: “class schedule change” in choice (D) indicates that this is the best answer. She did not actually miss this class, so choice (A) can’t be considered correct. Choice (B) is also incorrect, in the sense that the woman clearly doesn’t want to go to Body Pump, but this is not a problem as she is given another choice.

69. Answer (C)The woman says, “usually there is a sizeable group of people in by now,” meaning that the class is quite popular and well attended: answer choice (C) is the best option. The class, as follows from the conversation, takes place on Thursdays, not on Tuesdays, so choice (A) can’t be correct. Choice (B) is also incorrect, as the man states exactly the opposite. Finally, nothing is said about the level of participants in the Stretching class: only the Yoga class is mentioned, which is open to everyone.

70. Answer (B)The Stretching class, which has been moved, starts at noon, and the Yoga class the woman would like to attend is scheduled at 12:15. Therefore, she has between 15 and 20 minutes before the class starts and can use this time to warm up.

Page 170: BASES ET RVISIONS NCESSAIRES POUR LE TOEIC · 2019. 12. 2. · BASES ET RVISIONS NCESSAIRES POUR LE TOEIC 3 CONSIGNES Ā APPLIQUER AVANT LE STAGE Révisez les temps présentés dans

170

Part 471. Answer (D)The man says that the hours were “odd,” which is a synonym for “strange” or “unusual.” Therefore, answer choice (D) is correct. Choice (A) states exactly the opposite, while choices (B) and (C) refer to the job itself rather than the schedule.

72. Answer (A)None of the options listed is cited in the talk, but “accommodation” is a synonym for “lodging,” which is men-tioned by the man.

73. Answer (B)All answer choices are plausible, but (A) and (D) can be eliminated right away, as the speaker mentions them only as initial factors in his decision. Choosing between (B) and (C), note the last phrase of the talk: “I would recommend this job to anyone who enjoys the company of others” – this is a direct indication that (B) is the cor-rect answer. Being adventurous per se would mean a certain affinity for travel, but not necessarily as a tour guide.

74. Answer (A)The message states that the office hours are “8 am to 6 pm Monday through Saturday, and 9 am to 5 pm on Sunday.”

75. Answer (C)The message invites everyone wishing to track an order (which is the situation of the caller mentioned in the question) to dial 4.

76. Answer (D)The Accounting department is not listed among the options directly; therefore, it falls under “all other questions” category, which is option 6. Note that choice (B) cannot be correct since an employee directory would not help the caller identify the Accounting department’s extension.

77. Answer (D)The man states, “we provide a comprehensive course,” which indicates that answer choice (D) is correct. All the other answer choices are not the direct objects of the man’s talk.

78. Answer (B)The man says nothing about the frequency of mistakes and accidents in deep sea diving; therefore, choice (A) cannot be considered correct. Answer choice (B), on the other hand, is cited almost directly in the talk: “among many of the pastimes quickly growing in popularity.” Answer choices (C) and (D) refer to the courses offered by the man, not the activity itself.

79. Answer (A)When talking about the Association, the man simply states that the diving safety is becoming more prevalent: this is something that answer choice (A) claims as well. None of the remaining answer choices is confirmed by the talk.

80. Answer (A)The woman is mainly promoting her product: the solar panel roofing, so answer (A) is the best choice. While she does endorse “green energy,” this is not the main purpose of the talk (as called for in the question), as the

Page 171: BASES ET RVISIONS NCESSAIRES POUR LE TOEIC · 2019. 12. 2. · BASES ET RVISIONS NCESSAIRES POUR LE TOEIC 3 CONSIGNES Ā APPLIQUER AVANT LE STAGE Révisez les temps présentés dans

171CORRECTION DÉTAILLÉE TEST 2

woman is not talking about green energies in general and does not promote any kind of them other than the solar panels; besides, she clearly represents a business (“our company”): answer choice (B) cannot be considered correct. Answer choices (C) and (D) are minor details from the talk and clearly not the main purpose of the woman’s speech.

81. Answer (D)The installation and the cost are not mentioned in the talk, so answer choices (A) and (B) are incorrect. The outage protection is mentioned, but it is not the roofing that guarantees it: it’s the battery that comes with it, so choice (C) is also wrong. Answer choice (D) is correct, as it paraphrases the portion “while maintaining both its form and function” referring to the existing roof.

82. Answer (C)The woman says that the panels provide “additional protection against wind, rain, hail, and various debris,” mea-ning that they resist well these phenomena, so answer choices (A) and (D) are incorrect. She also says that the panels are “much sturdier than standard roof tiles,” which means that answer choice (B) is wrong and (C) can be assumed as correct.

83. Answer (B)The woman mainly talks about her job, describing her activities and background. Therefore, answer choice (B) is correct. The woman is not selling any services, so choice (C) is incorrect. Choice (D) almost contradicts the woman’s claim that the experience is as important for this job as education. Choice (A) is a minor detail from the speech.

84. Answer (D)The correct answer is an option that is not mentioned in the talk. As a directory of photography, the woman “oversees the lighting … work” (choice B), establishes the intentions of the clients (choice A) and “chooses the equipment” (choice C). Teaching anyone is not mentioned as part of her responsibilities, and therefore (D) is the right answer.

85. Answer (D)The woman states, “a lot of the experience required for the job, like I have, will be gained as a camera operator.” It can, therefore, be safely assumed that this was her previous job: answer choice (D). Director of photography (choice B) is her current job: “I am a director of photography.” Teaching photography and organizing fairs are never mentioned as the woman’s current nor past functions.

86. Answer (A)The woman starts by speaking about her passions and then mentions “an opening for a full-time position” as well as her “background and experience.” All this makes one believe that she is being interviewed for a job.

87. Answer (B)All the answer choices seem plausible at first sight, but their detailed analysis allows to eliminate wrong answers one by one. Engineering education (A) is just a part of woman’s background: while it might indeed be a source of inspiration for her, it is not presented as such in the talk. Landscaping is most probably what the company in question does, but not necessarily what the woman is inspired by, so choice (C) is also wrong. Response (D) states the opposite of what the woman says, “I love seeing others with similarly well-intentioned philosophies…” Answer choice (B) is cited directly from the talk: “giving back to the community is a huge part of the motivation in my work” and is, therefore, the correct answer.

Page 172: BASES ET RVISIONS NCESSAIRES POUR LE TOEIC · 2019. 12. 2. · BASES ET RVISIONS NCESSAIRES POUR LE TOEIC 3 CONSIGNES Ā APPLIQUER AVANT LE STAGE Révisez les temps présentés dans

172

88. Answer (C)The woman says, “I know I can provide just what the position requires, and more.” This indicates that answer choice (C) is unambiguously correct. Answer choice (B) is wrong, as the woman is being interviewed for a full-time position after having been volunteering for some time. Choice (D) is incorrect because the woman affirms having enough mechanical engineering experience already. Answer choice (A) might be considered correct, were it not for the word “completely,” which is too extreme to adequately describe the woman’s intentions.

89. Answer (A)The advertisement lists the branches of law in which Northbrook Associates specialize: labor, immigration, civil rights, and consumer law. While businesses (B) can indeed take advantage of Northbrook’s expertise in labor and consumer law, individuals seem a more direct target audience for all the four branches cited.

90. Answer (C)Answer choices (A) and (D) are never mentioned in the talk. Answer choice (B) is something that the firm claims to strive for, not the actual basis of its reputation. Answer choice (C) is a correct paraphrase of the claim “Our expert legal representatives have been certified by the highest-ranking law schools.”

91. Answer (B)The correct answer – choice (B) – is indicated by the last phrase of the talk, “…provides you not only with top-rate legal representation but with a family built on the foundation of always lending a helping hand.” Information from the other answer choices cannot be found in the talk.

92. Answer (A)According to the talk, the speaker’s seminars are intended for smaller audiences. The speaker says that it is pre-ferable to not have over 50 people in the audience. From this it can be inferred that the typical audience for the speaker is between 1 and 50.

93. Answer (B)The speaker says that work in the seminar produces creativity and independence.

94. Answer (C)The speaker refers to the seminars and then says that the skills learned are all part of “leadership training.” This indicates that he is a leadership trainer. Note that “motivational talks” (answer choice (B)) are not mentioned anywhere in the speech.

95. Answer (C)The advertisement states that the construction started in 2015. According to the graphic, it was finished in 2018. Therefore, 3 years would be the safest assumption for the answer.

96. Answer (D)The correct answer is the choice that is not mentioned in the advertisement. Choices (B) and (C) are mentioned directly. Choice (A) can be deduced as advertised from “family-friendly environment” in the beginning of the talk. Answer choice (D) is the only one remaining: there is no mention that the resort organizes visits of any kind.

97. Answer (A)The last sentence of the advertisement mentions “our rewards system,” so answer choice (A) is true. All the other answer choices are too far-fetched to be assumed as necessarily true about the enterprise.

Page 173: BASES ET RVISIONS NCESSAIRES POUR LE TOEIC · 2019. 12. 2. · BASES ET RVISIONS NCESSAIRES POUR LE TOEIC 3 CONSIGNES Ā APPLIQUER AVANT LE STAGE Révisez les temps présentés dans

173CORRECTION DÉTAILLÉE TEST 2

98. Answer (B)Stephen says he is calling from “Outward Events, regarding the setup for our upcoming cocktail,” implying that he works in a company that organizes events – in this particular case, a cocktail.

99. Choice (D)In his message, Stephen mentions “the gorgeous view from your hall.” Therefore, choice (D) is the best option. Nothing is said about the size of the hall, so choice (B) can’t be correct. Stephen asks to “place the bar” in the hall, so it’s unlikely that the bar already exists in the hall, not to mention its popularity. Technology is not really mentioned in the talk, except for two microphones, which are clearly not “cutting-edge technology” these days.

100. Answer (C)Stephen says, “We’re gonna need 8 cocktail tables.” In the plan, there are only 7 of them.

Reading Section

Part 5101. Answer (B)The context of the sentence calls for an indefinite article. As the word “honest” starts with a vowel sound, the article should be “an,” not “a.”

102. Answer (A)Answer choice (B), “grow,” cannot be used transitively in this context. The remaining choices fit grammatically, but only “start” fits contextually: a business/company can be started, but neither generated nor made.

103. Answer (D)The word “tomorrow” indicates that the modal verb should be conjugated in the future tense. The modal construc-tion expressing ability or permission in the future is “will be able to,” or, in this case, “won’t be able to”: answer choice (D) is correct. Choices (A) and (B) are grammatically impossible in English, and choice (C) lacks the particle “to.”

104. Answer (D)The idiomatic expression describing anticipation is “look forward to,” always followed by the gerund.

105. Answer (C)In the context, the blank must be filled with a preposition indicating the time of an event’s start. Only the pre-position “since” can be used with “when.”

106. Answer (B)The conjugated verb in the main clause is “couldn’t”; therefore, the tag must contain “could” (in the positive form). The subject of the main clause is plural: “Gina and Alfredo”; therefore, the subject in the tag must be “they.”107. Answer (A)A conjunction that indicates a contradiction between two clauses is required: “but” is the best choice. None of the remaining answer choices indicates a contradiction: “if ” is an indicator of a condition (not possible because

Page 174: BASES ET RVISIONS NCESSAIRES POUR LE TOEIC · 2019. 12. 2. · BASES ET RVISIONS NCESSAIRES POUR LE TOEIC 3 CONSIGNES Ā APPLIQUER AVANT LE STAGE Révisez les temps présentés dans

174

of “instead” in the second clause); “so” is an indicator of a consequence (also impossible because of “instead”); “or” is an indicator of an alternative, which would be applicable if a possibility were expressed in both clauses.

108. Answer (A)“Stationery” is a term used to describe writing or printing paper. In the context of the sentence, “company sta-tionery” most probably means paper with the company’s logo, which, logically, should not be used for personal correspondence. Answer choice (B) is a distortion of “letterhead,” which also means “company stationery.”

109. Answer (B)As there are more than 2 items that are being compared, superlative degree, rather than comparative, is required: this eliminates choices (A) and (C). When speaking of distances in the literal sense, the form “farther/farthest” is preferable to “further/furthest,” the latter being reserved for abstract concepts.

110. Answer (B)The sentence states that something happened seconds ago. In this context, the present perfect test is the best choice: “has” is the correct answer.

111. Answer (C)All the answer choices are modal verbs expressing possibility, obligation, or advice and therefore fit the context. However, only “ought” can (and must) be followed by “to” and fits grammatically.

112. Answer (D)“Jetlag” is a noun, and “jetlagged” is a participle; in this context, the latter is clearly preferable.

113. Answer (A)The context of the question requires a word that would describe someone who is not an employee but could be considered as (or confused with) such. The only viable option is “an external consultant” – someone who works on a temporary project without actually being hired under an employment contract.

114. Answer (A)The verb in the blank must be conjugated in the subjunctive mood because the clause expresses a suggestion: in the main clause, one sees “I suggest that…” The correct form of the subjunctive in English is the bare infinitive: “discuss.”

115. Answer (D)In English, the word “advice” is uncountable, meaning that it can be only singular and is never used with nume-rals. Therefore, the correct construction is “two pieces of advice.” Note that “advise” is a verb.

116. Answer (A)In an English clause, only one negation can be used. In the sentence, this negation is already present outside the blank: the negative pronoun “no.” Therefore, the conjugated verb should be in its positive form, and the advert “ever” should be used instead of “never”: the correct answer choice is (A).

117. Answer (B)The blank calls for a conjunction indicating a condition: “you can pay by check only if you are a local resident.” Among the answer choices, only the word “provided” (or sometimes “provided that”) can function as a conjunction.

118. Answer (C)The answer choice must be in comparative form, not superlative, as Igor’s situation today is compared only with his situation yesterday. “Ill” being an irregular adjective when it comes to comparisons, its correct comparative form is “worse.”

Page 175: BASES ET RVISIONS NCESSAIRES POUR LE TOEIC · 2019. 12. 2. · BASES ET RVISIONS NCESSAIRES POUR LE TOEIC 3 CONSIGNES Ā APPLIQUER AVANT LE STAGE Révisez les temps présentés dans

175CORRECTION DÉTAILLÉE TEST 2

119. Answer (B)The sentence describes a situation when one action happened in the past “the phone rang,” while another action was happening at the same moment. Therefore the verb in the blank must be conjugated in past progressive tense: “was sorting.”

120. Answer (A)The general question containing a negative form of a modal verb is usually formed by placing the modal verb, contracted with the particle “not,” in front of the subject: “Shouldn’t we.” Note that the form “Should we not” is also possible, but it is not offered as an answer choice.

121. Answer (B)The preposition “within” indicates that the limits have been set, and the word “expenses” suggests that these limits are financial. The financial limits of a department are usually defined in a budget, which is the word fitting the present context the best.

122. Answer (D)The sentence is a past conditional structure (“yesterday”) containing an “impossible condition”: Pekka actually did break his phone. In this case, the correct conjugation of the verb “to be” is the past perfect tense: “had been.”

123. Answer (D)The correct structure employed to indicate a habit is “to be used to doing”; therefore, answer choice (D) is correct.

124. Answer (A)The fixed expression for “sue” is “press charges.”

125. Answer (C)The verb of the sentence is conjugated in the past: “took”; therefore, the time indicator should also refer to the past. The only answer choice referring to the past is (C).

126. Answer (D)Note that the sentence is not a question: there is no need to use the inversion, so answer choice (C) is wrong. “Let us do” or “let’s do” is a structure used to formulate a suggestion referring to the speaker and the listeners: (D) is the correct answer.

127. Answer (C)The sentence is in the past (“last Friday”), so answer choices (A) and (D) can be excluded. “Celine” is singular, and the sentence is in the indicative mood, so “were to” is grammatically incorrect. The only possible modal verb is, therefore, “had to” – choice (C).

128. Answer (C)The person who serves drinks and food in a café or restaurant is called a “waiter” or “server.”

129. Answer (C)The structure of the sentence requires a verb conjugated in the passive voice. The only answer choice using passive voice is (C).

130. Answer (D)The context calls for an adverb indicating a sequence of events, with the meaning of “afterwards.” The only pos-sible option is “then.” Answer choice (A) is a conjunction, choices (B) and (C) are pronouns, and none of these conveys a meaning of sequence.

Page 176: BASES ET RVISIONS NCESSAIRES POUR LE TOEIC · 2019. 12. 2. · BASES ET RVISIONS NCESSAIRES POUR LE TOEIC 3 CONSIGNES Ā APPLIQUER AVANT LE STAGE Révisez les temps présentés dans

176

Part 6131. Answer (C)The text is clearly an advertisement for a product. The most logical way to start an ad is to name the product advertised. Both choices (A) and (C) do that. However, choice (A) does not fit contextually: it talks about food for dogs, while the ad is dedicated mostly to a dog bed.

132. Answer (C)The context of the phrase calls for a conjunction that would indicate opposition: “other pet beds will tear, [but ours] has a lifetime warranty.” Choices (A) and (D) are adverbs, not conjunctions. Choice (B) would indicate a logical consequence rather than opposition. Only choice (C) fits the context: “while” here means “although” or “whereas.”

133. Answer (B)The subject of the sentence is “each,” not “items.” Therefore, the verb should be conjugated in the third person singular, which excludes choices (A) and (D). Choosing between the present simple (B) and the present perfect (C), note that the sentence speaks of a habitual action rather than something accomplished, especially in the light of “may include”: for this reason, choice (B) is largely preferable over (C).

134. Answer (D)According to the context of the phrase, the blank should be filled with an option that is likely to give a discount (“30% off ”) on the placed order. Opening a pet club would be too much of hassle for obtaining a discount; on the contrary, simply opening a package wouldn’t be enough. The advertisement is clearly that of a store, not a bank, and therefore choice (C) doesn’t fit the context. The only plausible option is “membership account,” which is usually a way of obtaining discounts in stores.

135. Answer (C)The correct expression in this context is “in regards to,” which is basically a fancy way of saying “about” in an official context. Choice (B) would be correct if it read simply “regarding.”

136. Answer (D)The blank calls for a preposition meaning “before.” Only the option “until” conveys this meaning.

137. Answer (A)The context of the phrase calls for an adjective that would positively qualify a business partner. This excludes choices (B) and (D). While “understanding” is a positive quality, it doesn’t fit the context of a very formal letter. Therefore, choice (A) is the best option: “reliable” describes someone one can count on.

138. Answer (A)The tone of the letter, quite menacing, and the promise to take “legal action” indicate that the letter is written by a lawyer or, at the minimum, someone experienced in recovering debts. Answer choice (A) is, therefore, the most plausible. Choice (C) is unlikely, as accountants rarely deal with customers directly, especially in so serious a situation.

139. Answer (D)Whenever a verb follows a preposition, the verb must be in its gerund form.

140. Answer (A)The context of the phrase calls for a word that would mean “new stories.” The word “updates” means exactly that: news.

Page 177: BASES ET RVISIONS NCESSAIRES POUR LE TOEIC · 2019. 12. 2. · BASES ET RVISIONS NCESSAIRES POUR LE TOEIC 3 CONSIGNES Ā APPLIQUER AVANT LE STAGE Révisez les temps présentés dans

177CORRECTION DÉTAILLÉE TEST 2

141. Answer (B)In this context, the blank should be filled with a word meaning “far away.” This is the case of the word “remotest.” Choices (A) and (C) indicate exactly the opposite meaning, and choice (D) is clearly off topic.

142. Answer (B)The phrase following the blank is, “It will help get our ideas through to your friends and acquaintances.” The-refore, the correct answer choice should feature a proper antecedent for “It” and some reference to “friends and acquaintances.” Only option (B) does that fully.

143. Answer (A)The phrase is about the physical capacity of the parking lot to contain cars: the best answer choice is the modal verb “can,” expressing the physical possibility.

144. Answer (D)The phrase following the blank is, “However, accidents […] outside the company’s property are not something we will be able to assist with.” This means that the sentence to fill in the blank must introduce an idea contrary to the fact that the company is unable to help with problems occurring outside the company’s premises. Answer choice (D) expresses that idea precisely: The company will be able to help with accidents occurring on the company’s premises. All the other answer choices lack the contrast necessary for a perfect fit.

145. Answer (C)The context of the required answer choice is set by the expression “accidents and similar _______”: in other words, the correct answer choice is a close synonym of “accidents.” “Causes” and “circumstances” are not close synonyms, as they do not have a connotation of “events.” “Phenomena” is quite a close synonym but does not fit stylistically, since it is used mostly in scientific or esoteric contexts. The best answer choice is “incidents,” especially since the phrase deals with insurance.

146. Answer (B)The correct preposition in this context is “to”: one reports to someone.

Part 7147. Answer (D)The ticket is for traveling with NeueHansa Ferries, and it mentions “cabin class.” Therefore, the trip will take place on a ship.

148. Answer (C)The departure is at 14:00 (2 p.m.), and the arrival is at 10:00 (10 a.m.). Therefore, the journey will last 20 hours: answer choice (C).

149. Answer (A)This automatic email states that Ms. Sauliunaite is currently “on vacation,” which is a synonym for “on holiday.”

150. Answer (C)The subject of Mr. McCarran’s initial email is titled “URGENT,” and it is unlikely that he would be willing to wait for 12 days ( June 20th – July 2nd) for a matter he considers urgent. Therefore, he will most probably follow Ms. Sauliunaite’s suggestion and contact her colleague Mr. Field.

151. Answer (B)The instructions read, “Before using your Instant Pressure Cooker, make sure that there are no flaws.”

Page 178: BASES ET RVISIONS NCESSAIRES POUR LE TOEIC · 2019. 12. 2. · BASES ET RVISIONS NCESSAIRES POUR LE TOEIC 3 CONSIGNES Ā APPLIQUER AVANT LE STAGE Révisez les temps présentés dans

178

152. Answer (C)The text refers to the device several times as “pot,” e.g. “no flaws in the composition of the pot” and “center the pot on top of a larger burner.” The latter statement also confirms that choice (D) is wrong. Since more than a cup of water fits in it, it can’t be in a form of a cup: choice (A) is incorrect. A lid is only a part of the device, so choice (B) cannot be correct.

153. Answer (C)In his first message, Ryu asks, “Are you far from the office already?”, implying that Danuta left recently and actually might be not far. Answer choice (C) is therefore the best. Other answer choices are not supported by the conversation.

154. Answer (C)In the same message, Danuta says, “I forgot to tell you,” meaning that she does believe Ryu, that she is not surprised at his message, and that she knows what is “wrong” with the wi-fi. The only possible answer choice is, therefore, (C), which is also indirectly confirmed by Danuta’s message at 5:03 p.m.

155. Answer (B)The notice discusses the renovation of some of the park’s installations, which will take place soon, so answer choice (B) is correct. Options (A) and (C) are only details in the notice and are the consequences of the renova-tion. Nothing in the notice concerns the past, so choice (D) cannot be correct.

156. Answer (C)The second sentence of the notice lists the installations that will be closed: “five of our roller coasters, two child-ren’s rides, and three restaurants,” which is 10 in total.

157. Answer (A)Only tickets purchased between March 28th and May 28th can be reimbursed or exchanged, so choices (B) and (C) are incorrect. 50% is a discount on food purchases, not on the ticket price, so choice (D) is also incorrect. Answer choice (A) is correct: the notice states, “we will also be offering two-day passes at the same price as a single day pass.”

158. Answer (D)At the stage of this letter, Kim Choi offers only a job interview to Mr. Anderson: “to request a preliminary inter-view with our team.” Therefore, choice (D) is the only possible answer.

159. Answer (B)As part of the responsibilities, the job requires training interns, but it is never mentioned in the text that recruit-ment of any kind would be part of it.

160. Answer (A)The remuneration is discussed in the first sentence of the second paragraph of the letter: “The starting salary for our current position…” Therefore, the phrase should be placed in positions [1] or [2]. Choosing between them, note the word “also” in the phrase to be inserted: it indicates that the idea continues from the previous sentence, rather than starts a new topic. Therefore, position [2] is the best choice.

161. Answer (B)The ad mentions “being more customized … than any other tax service,” so choice (B) is correct. Choice (A) contradicts the text, as the stated goal is to “eliminate the stress.” The services are provided year-round and 24/7, so (C) cannot be correct, in spite of the “tax season.” It is never mentioned that services are free, so (D) is wrong.

Page 179: BASES ET RVISIONS NCESSAIRES POUR LE TOEIC · 2019. 12. 2. · BASES ET RVISIONS NCESSAIRES POUR LE TOEIC 3 CONSIGNES Ā APPLIQUER AVANT LE STAGE Révisez les temps présentés dans

179CORRECTION DÉTAILLÉE TEST 2

162. Answer (B)Regarding the clients who choose not to file online, the ad states, “we are more than happy to reimburse the extra money needed for first class shipping.” Options (A) and (D) find no confirmation in the text. And while (C) might be true, it is never stated or implied in the ad.

163. Answer (D)According to the ad, the discount is given to “individuals in their first two years of owning a business.” Only answer choice (D) satisfies those conditions.

164. Answer (C)In his 17:05 message, Zoltan Petofi says that the client would like to order “porcelain plates, saucers, and porce-lain trays.” “Porcelain” is a synonym for “china” in this context, meaning that the company produces china dishes.

165. Answer (D)In his 17:02 message, Zoltan says, “I need more information regarding the delays,” which means that answer choice (D) is correct. The same message means that choice (C) is wrong.

166. Answer (B)In this context, the word “scale” means “volume,” or “size,” of the order. This meaning is also confirmed by Zoltan’s reply following Lola’s question in this message. Answer choice (B) is therefore the best.

167. Answer (D)The expression “You tell me” in this context means “The choice is up to you.” Benjamin offers two options: either pack the order in 3 days using current personnel only or hire temporary personnel, thus packaging the order faster. He doesn’t have a preference and leaves the choice to his colleagues.

168. Answer (A)Ms. Lee is clearly complaining about the services in general, so answer choice (A) is correct. She is not claiming reimbursement (B) and describes more than one visit (C). The purpose of her letter is not to evaluate the perfor-mance of the company as such, so (D) is also incorrect.

169. Answer (C)The letter reads, “I walked in two minutes late to my scheduled time,” meaning that Ms. Lee was late for her appointment – answer choice (C). She was not refused the service (“before I could attend my appointment”); the appointment had indeed been scheduled; the purchase of an unsatisfactory product took place “this week” and not “last month.”

170. Answer (B)According to the letter, “the new hairspray was half the size, but twice the cost of the old one.” This means that it is exactly 4 times more expensive than the previous one, so answer choice (B) is correct. This also means that choice (C) contradicts the text. The quality of the spray is not discussed, so (A) cannot be correct. The advisor didn’t know about other sizes of the same product, but it doesn’t necessarily mean that they don’t exist: on the contrary, Ms. Lee implicitly suggests that they do, so choice (D) cannot be correct either.

171. Answer (A)The word “However” starting the phrase to insert indicates that there must be a contradiction between the ideas expressed before and after this phrase. The only such transition – from a positive to a negative review – can be observed around position [1]: Ms. Lee had appreciated Spa Z’s services since 2003, but the staff no longer seems adequate. These two ideas are connected by the inserted phrase, which serves as a link between them.

Page 180: BASES ET RVISIONS NCESSAIRES POUR LE TOEIC · 2019. 12. 2. · BASES ET RVISIONS NCESSAIRES POUR LE TOEIC 3 CONSIGNES Ā APPLIQUER AVANT LE STAGE Révisez les temps présentés dans

180

172. Answer (D)Choice (A) contradicts directly the first phrase of the memo. The registration procedure sounds rather simple, so (B) is not the right answer. Nothing is said about the price, in either absolute or relative terms, so answer choice (C) cannot be correct. As Mr. Nilssen mentions “accommodation” that the company will pay for, it is safe to assume that the Expo will take place in a city relatively far from where most of the employees reside: choice (D) is correct.

173. Answer (D)The memo says that in order to register, one can “respond to the Expo newsletter with [one’s] name and workplace information,” so answer choice (D) is correct. Choices (A) and (B) are items that registrants will receive after the registration. The newsletter (C) in itself is not indispensable since one can register by phone.

174. Answer (C)Mr. Nilssen mentions “our offices, such as ophthalmology and disease.” It is therefore clear that his organization is a medical one.

175. Answer (D)According to the memo, “Employees chosen to train multiple individuals will be compensated with a bonus at the end of the month,” so only answer choice (D) can be correct.

176. Answer (B)While never stated directly, “Marketing” is easy to infer: Donald talks about customer satisfaction, polls, and brand recognition. All these work aspects are clearly marketing-related.

177. Answer (A)It is clear from both the text, “the marketing event in late 2015 was extremely successful and resulted in higher customer satisfaction ratings the following year,” and the graphs that the number of clients satisfied with the company has increased dramatically from 2015 to 2016. Donald attributes it to the marketing event, which took place in late 2015.

178. Answer (C)Donald writes, “your intuition regarding the proportion of unhappy customers has proved to be correct.” There-fore, Jessica’s guess was about the unsatisfied customers. As one can see from the graphs, the combined number of unsatisfied and very unsatisfied clients has actually increased between 2015 and 2016.

179. Answer (C)“To be stable” means “not to change” in this context. As it is visible in the chart, the only client category not to change its proportion between the two graphs is “Unsatisfied.” “Satisfied” and “Very unsatisfied” have increased while “Not sure” has diminished.180. Answer (A)In the conclusion of his email, Donald writes, “We will organize an internal meeting as soon as possible” in order to address the unusually high number of “very unsatisfied” clients. Organizing an event is also a goal, but not as immediate as holding a meeting, so answer choice (A) is clearly correct.

181. Answer (D)Mr. Grady does complain about certain points in his message, but finally requests only explanations (answer choice (D)); a new order (A), a change to an existing order (C), and a reimbursement (B) are never mentioned in his email.

Page 181: BASES ET RVISIONS NCESSAIRES POUR LE TOEIC · 2019. 12. 2. · BASES ET RVISIONS NCESSAIRES POUR LE TOEIC 3 CONSIGNES Ā APPLIQUER AVANT LE STAGE Révisez les temps présentés dans

181CORRECTION DÉTAILLÉE TEST 2

182. Answer (A)Mr. Aguillar explains that “it was the only way to get [the goods] to your facility without delay when coming from our multiple warehouses” when talking about multiple trucks delivering the supplies; therefore, choice (A) is correct. All the other answer choices provide details from the email exchange that do not give a reason per se for multiple deliveries.

183. Answer (B)Both emails mention that Mr. Aguillar’s company has recently changed the locations of its warehouses, and the second email mentions a change of a manager at Mr. Grady’s company. Answer (B) is therefore correct. Office location changes are never discussed in the texts.

184. Answer (C)Mr. Aguillar’s email states, “Regarding the excessive plasticware, we will gladly allow you to keep the surplus.” This means that the plastic cups, lids, and straws, delivered in excess of 30 items each, will not be invoiced and that Mr. Grady’s company can keep them for free.

185. Answer (D)Mr. Aguillar says, “your order of the cleaning products will be reviewed …, and I will inform you of the results.” This is a polite way of saying, “We don’t know what happened, but we will investigate and let you know.” There-fore, answer (D) is correct.

186. Answer (C)The email addressed to the counselors says, “You will be placed into 4 groups of 5,” which means that there will be a total of 20 counselors.

187. Answer (A)According to the list, “camp equipment,” of which a tent is definitely a part, needn’t be brought to the camp; choice (A) is, therefore, the correct answer. All the other items fall under the “Bring your own” category: toiletries (B), clothes (C), and entertainment items (D).

188. Answer (B)“Nocturnal” means “taking place in the evening or at night, i.e. after dark.” The only activity that is announced as starting in the evening – at 8 pm – is Fire Building.

189. Answer (B)The schedule reads: June 2nd – Camp Start. It means that this is the date when children will arrive at the camp, so answer choice (B) is correct. The counselors arrive on May 24th, and their training starts on June 25th. Nothing is specifically stated about the policy review, except perhaps “Mid-Season Review,” which will take place on June 12th.190. Answer (D)The email states, “The payment for your service will be given at the end of the 20-day camp session.” The camp session lasts indeed for 20 days, from June 2nd to June 22nd; therefore, June 22nd is payday.

191. Answer (B)Mr. Schlossberg starts his email with, “We have at least one confirmed case of fraudulent use of the company’s credit card. The transaction originated in Singapore.” Answer choice (B) is correct. He doesn’t need a list of all expenses made by his colleagues while in Singapore, but only of those paid with credit cards; therefore, answer choice (D) cannot be correct.

Page 182: BASES ET RVISIONS NCESSAIRES POUR LE TOEIC · 2019. 12. 2. · BASES ET RVISIONS NCESSAIRES POUR LE TOEIC 3 CONSIGNES Ā APPLIQUER AVANT LE STAGE Révisez les temps présentés dans

182

192. Answer (A)Ms. Van der Merwe mentions in her email that she is “working at the convention,” and her expense list fea-tures a “Convention Pass.” It is, therefore, safe to assume that the primary goal of her trip to Singapore was this convention. While she might be meeting with clients (“Business Dinner” expense on the list), this is clearly not the main purpose of the trip. She is not investigating any fraud, and advice given to her colleagues is a minor detail in her email.

193. Answer (B)In her email, Ms. Van der Merwe says she will return on February 10th, and in her list “Taxi from the airport” (expense nominated in Singapore dollars) is on February 6th, meaning she arrived in Singapore on this date. Thus, she spends 4 nights in Singapore.

194. Answer (A)Ms. Van der Merwe says, “I will be able to provide a finished list upon my return.” She later adds that the payment for the taxi on her return trip will be an additional expense. All this allows concluding that answer choice (A) is correct. Every team member has his or her own card, so choice (B) cannot be correct. Choice (C) contradicts directly Ms. Van der Merwe’s email. Choice (D) is a distortion of the information from the email.

195. Answer (D)Hotel is by far the largest expense, and the Business Dinner is the second largest, with 120 SGD.

196. Answer (D)The advertisement starts by saying, “What would a trip to Ireland be without…” It is therefore safe to assume that the website is dedicated to people visiting Ireland from abroad: foreign tourists. This answer is also indi-rectly confirmed by Mildred’s review, as it is clear that she is not quite familiar with the country and its culinary traditions.

197. Answer (B)The advertisement says that the Market is open every Saturday. Mildred Bernstein visited it on December 9th, so December 9th was a Saturday. She posted her review on December 15th, which then was, logically, a Friday.

198. Answer (C)Mildred visited Offaly Market on December 9th. Ciaran says in his comment that the trout can only be fished in Ireland from February through October: in other words, there was no fresh trout on the market because it was not the season.

199. Answer (A)Mildred says that she got a basket for a “very reasonable price.” The meaning “low” can therefore be easily deduced from the context.

200. Answer (A)Ciaran O’Healy’s signature says, “President, Offaly County Farmers Market Cooperative.” Therefore, it is clear that he is directly involved in Offaly Market’s organization and operations. Other answer choices either contra-dict his comment, like (B), or do not find enough support in his comment, like (C) and (D).

Page 183: BASES ET RVISIONS NCESSAIRES POUR LE TOEIC · 2019. 12. 2. · BASES ET RVISIONS NCESSAIRES POUR LE TOEIC 3 CONSIGNES Ā APPLIQUER AVANT LE STAGE Révisez les temps présentés dans

183GRILLE CONVERSION DU SCORE

TOEIC L&R PRACTICE TESTS 1 & 2

Score Conversion TableNumber of correct answers Converted Listening score Converted Reading score

0 5 51 5 52 5 53 5 54 5 55 5 56 10 57 15 58 20 109 25 15

10 30 2011 35 2512 40 3013 50 3514 55 4015 60 4516 65 5017 70 5518 75 6019 80 6520 90 7021 95 7522 100 8023 105 8524 110 9025 115 9526 120 10027 130 105

Page 184: BASES ET RVISIONS NCESSAIRES POUR LE TOEIC · 2019. 12. 2. · BASES ET RVISIONS NCESSAIRES POUR LE TOEIC 3 CONSIGNES Ā APPLIQUER AVANT LE STAGE Révisez les temps présentés dans

184

28 135 11029 140 11530 145 12031 150 12532 155 13033 160 13534 170 14035 175 14536 180 15037 185 15538 190 16039 195 16540 200 17041 210 17542 215 18043 220 18544 225 19045 230 19546 235 20047 240 20548 250 21049 255 21550 260 22551 265 23052 270 23553 275 24054 280 24555 290 25556 295 26057 300 26558 305 27059 310 27560 315 28061 320 28562 330 29563 335 30064 340 305

Page 185: BASES ET RVISIONS NCESSAIRES POUR LE TOEIC · 2019. 12. 2. · BASES ET RVISIONS NCESSAIRES POUR LE TOEIC 3 CONSIGNES Ā APPLIQUER AVANT LE STAGE Révisez les temps présentés dans

185GRILLE CONVERSION DU SCORE

65 345 31066 350 31567 355 32068 360 32569 370 33570 375 34071 380 35072 395 36573 400 37074 410 38075 415 38576 420 39077 425 39578 430 40079 440 41080 450 42081 455 42582 460 43083 465 43584 470 44585 475 45086 480 45587 485 46088 490 46589 490 47090 490 47591 495 48092 495 48593 495 48594 495 49095 495 49096 495 49097 495 49598 495 49599 495 495

100 495 495